SlideShare a Scribd company logo
1 of 90
Tải tài liệu tại sividoc.com
Viết đề tài giá sinh viên – ZALO:0973.287.149-TEAMLUANVAN.COM
ĐẠI HỌC THÁI NGUYÊN
TRƯỜNG ĐẠI HỌC KHOA HỌC
--------------

-------------
NGUYỄN VĂN NGHĨA
KỸ THUẬT BIẾN ĐỔI TÂM TỶ CỰ
VÀ ỨNG DỤNG VÀO GIẢI TOÁN
LUẬN VĂN THẠC SĨ TOÁN HỌC
THÁI NGUYÊN - 2017
Tải tài liệu tại sividoc.com
Viết đề tài giá sinh viên – ZALO:0973.287.149-TEAMLUANVAN.COM
ĐẠI HỌC THÁI NGUYÊN
TRƯỜNG ĐẠI HỌC KHOA HỌC
--------------

-------------
NGUYỄN VĂN NGHĨA
KỸ THUẬT BIẾN ĐỔI TÂM TỶ CỰ
VÀ ỨNG DỤNG VÀO GIẢI TOÁN
LUẬN VĂN THẠC SĨ TOÁN HỌC
Chuyên ngành: Phương pháp Toán sơ cấp
Mã số: 60 46 01 13
NGƯỜI HƯỚNG DẪN KHOA HỌC
PGS.TS. NGUYỄN VIỆT HẢI
THÁI NGUYÊN - 2017
Tải tài liệu tại sividoc.com
Viết đề tài giá sinh viên – ZALO:0973.287.149-TEAMLUANVAN.COM
i
Lời cảm ơn
Tôi xin chân thành cảm ơn phòng Đào tạo bộ phận sau đại học,
quý thầy cô giảng dạy lớp Cao học K9B (2015 - 2017) Trường Đại Học
Khoa Học - Đại Học Thái Nguyên đã tận tình truyền đạt những kiến thức
quý báu cũng như tạo điều kiện cho tôi hoàn thành khóa học.
Để hoàn thành được luận văn một cách hoàn chỉnh, tôi luôn nhận
được sự hướng dẫn và giúp đỡ nhiệt tình của PGS.TS. Nguyễn Việt
Hải, Giảng viên cao cấp Trường Đại Học Hải Phòng. Tôi xin chân
thành bày tỏ lòng biết ơn sâu sắc đến thầy và xin gửi lời tri ân nhất
của tôi đối với những điều thầy đã dành cho tôi.
Tôi xin gửi lời cảm ơn chân thành nhất tới gia đình, bạn bè, những
người đã luôn động viên, hỗ trợ và tạo mọi điều kiện cho tôi trong suốt
quá trình học tập và thực hiện luận văn.
Xin trân trọng cảm ơn!
Hải Phòng, tháng 6 năm 2017
Người viết Luận văn
Nguyễn Văn Nghĩa
Tải tài liệu tại sividoc.com
Viết đề tài giá sinh viên – ZALO:0973.287.149-TEAMLUANVAN.COM
i
Danh mục hình
1.1 Quy tắc Archimedes . . . . . . . . . . . . . . . . . . . . . . . 7
1.2 Tọa độ diện tích . . . . . . . . . . . . . . . . . . . . . . . . . . 11
2.1 Chọn tâm tỷ cự . . . . . . . . . . . . . . . . . . . . . . . . . . 19
2.2 Quĩ tích là đường tròn . . . . . . . . . . . . . . . . . . . . . . . 21
2.3 I là đỉnh thứ tư hình bình hành . . . . . . . . . . . . . . . . . . 23
2.4 Trực tâm H . . . . . . . . . . . . . . . . . . . . . . . . . . . . 27
2.5 Tọa độ tỷ cự điểm đồng quy . . . . . . . . . . . . . . . . . . . . 29
2.6 Tính tỷ số . . . . . . . . . . . . . . . . . . . . . . . . . . . . . 32
2.7 Tính diện tích . . . . . . . . . . . . . . . . . . . . . . . . . . . 34
2.8 Hình chóp tam giác đều . . . . . . . . . . . . . . . . . . . . . . 35
3.1 P;Q;R thẳng hàng . . . . . . . . . . . . . . . . . . . . . . . . . 43
3.2 MOP2006............................. 53
3.3 USAMO2001#2......................... 54
3.4 USAMO2008........................... 55
Tải tài liệu tại sividoc.com
Viết đề tài giá sinh viên – ZALO:0973.287.149-TEAMLUANVAN.COM
i
Mục lục
Lời cảm ơn i
Mở đầu 1
1 Tâm tỷ cự của hệ chất điểm 4
1.1 Hệ chất điểm và tâm tỷ cự . . . . . . . . . . . . . . . . . . . . 4
1.2 Các tính chất cơ bản của tâm tỷ cự . . . . . . . . . . . . . . . . 7
1.3 Tâm tỷ cự và diện tích đại số . . . . . . . . . . . . . . . . . . . 9
1.3.1 Diện tích đại số . . . . . . . . . . . . . . . . . . . . . . 9
1.3.2 Tọa độ tỷ cự trong mặt phẳng . . . . . . . . . . . . . . 12
1.4 Công thức Lagrang và công thức Jacobi . . . . . . . . . . . . . 15
2 Các kỹ thuật biến đổi tâm tỷ cự và ứng dụng 18
2.1 Kỹ thuật chọn tâm tỷ cự . . . . . . . . . . . . . . . . . . . . . . 18
2.2 Kỹ thuật diện tích hóa và tọa độ hóa. . . . . . . . . . . . . . . . 26
2.3 Kỹ thuật giao hoán-kết hợp. . . . . . . . . . . . . . . . . . . . . 31
2.4 Kỹ thuật quán tính. . . . . . . . . . . . . . . . . . . . . . . . . 36
3 Các vấn đề liên quan 41
3.1 Chứng minh một số định lý nổi tiếng . . . . . . . . . . . . . . . 41
3.2 Một số bài toán thi học sinh giỏi và thi Olympic . . . . . . . . . 50
3.2.1 Véc tơ chuyển chỗ . . . . . . . . . . . . . . . . . . . . 51
3.2.2 Đường thẳng vuông góc . . . . . . . . . . . . . . . . . 51
3.2.3 Phương trình đường tròn . . . . . . . . . . . . . . . . . 52
Tài liệu tham khảo 59
Tải tài liệu tại sividoc.com
Viết đề tài giá sinh viên – ZALO:0973.287.149-TEAMLUANVAN.COM
1
Mở đầu
1. Mục đích của đề tài luận văn
Khái niệm tâm tỷ cự đã được các nhà toán học đề cập đến từ lâu, chẳng
hạn xem ([1], [5], [6]). Tuy nhiên việc ứng dụng khái niệm này còn rất hạn chế vì
ngoài định nghĩa thông qua véc tơ, các tính chất và các biểu diễn khác của tâm tỷ
cự chưa được nêu trong các tài liệu truyền thống. Mục đích của đề tài này là
nghiên cứu đề xuất các tính chất đăc trưng của tâm tỷ cự từ đó đề ra các kỹ thuật
biến đổi tâm tỷ cự để giải các loại toán hình học phẳng. Cụ thể là:
- Nghiên cứu các tính chất đặc trưng của tâm tỷ cự của hệ chất
điểm. Đưa ra các kỹ thuật biến đổi tâm tỷ cự nhằm ứng dụng có hiệu
quả vào việc giải toán Hình học.
- Ứng dụng các kỹ thuật biến đổi tâm tỷ cự vào giải các bài toán
tính toán, chứng minh, tìm tập hợp điểm và các vấn đề khác nhằm
khắc sâu phương pháp giải các bài toán liên quan đến tâm tỷ cự.
- Các kiến thức được nâng cao: Xây dựng một lý thuyết chặt chẽ và có hệ
thống về tâm tỷ cự, các kỹ thuật biến đổi tâm tỷ cự, tính chất mô men quán
tính,. . . dựa vào khái niệm véc tơ. Bổ sung thêm một phương pháp hiệu quả
khi giải các bài toán hình học sơ cấp. Đặc biệt áp dụng được vào việc giải các
bài toán thi olympic Quốc gia và Quốc tế. Có thể nói đây một sáng tạo mới để
giải các bài toán hình học, một phương pháp giải toán có hiệu quả.
2. Nội dung của đề tài, những vấn đề cần giải quyết
Đề tài sẽ giải quyết các vấn đề sau: Hệ thống, chứng minh các tính chất
của tâm tỷ cự, trình bày các kỹ thuật biến đổi tâm tỷ cự để ứng dụng vào giải
Tải tài liệu tại sividoc.com
Viết đề tài giá sinh viên – ZALO:0973.287.149-TEAMLUANVAN.COM
2
các bài toán hình học có liên quan. Nêu ra được các bài toán mẫu, điển hình
minh họa cho các kỹ thuật biến đổi, giải được các bài toán khó, thể hiện được
tính hơn hẳn so với cách giải thông thường. Nội dung chia làm 3 chương:
Chương 1. Tâm tỷ cự của hệ chất điểm
Định nghĩa và nêu các tính chất của tâm tỷ cự chủ yếu là trên
mặt phẳng, các kiến thức cần thiết để xây dựng một số kỹ thuật biến
đổi tâm tỷ cự, chuẩn bị cho chương hai. Các tính chất được xây dựng
và chứng minh chặt chẽ, đầy đủ. Chương 1 gồm 4 mục sau.
1.1. Định nghĩa và ký hiệu.
1.2. Các tính chất cơ bản của tâm tỷ cự.
1.3. Các ví dụ mở đầu.
1.4. Công thức Lagrang và công thức Jacobi.
Chương 2. Các kỹ thuật biến đổi tâm tỷ cự và ứng dụng
Lần lượt trình bày các kỹ thuật biến đổi dựa vào các tính chất của tâm
tỷ cự trên mặt phẳng. Mỗi kỹ thuật được nêu thành các bước vận dụng, các ví
dụ và các bài toán mẫu. Hình thành các kỹ năng " chọn tâm tỷ cự, biến đổi
tâm tỷ cự, coi diện tích là tọa độ tâm tỷ cự,. . . " để giải các loại toán hình học
phẳng: chứng minh, tính toán, tìm quỹ tích,. . . Chương 2 trình bày 4 mục sau:
2.1. Kỹ thuật chọn tâm tỷ cự.
2.2. Kỹ thuật diện tích hóa.
2.3. Kỹ thuật giao hoán và kết hợp.
2.4. Kỹ thuật quán tính.
Chương 3. Các vấn đề liên quan
Tải tài liệu tại sividoc.com
Viết đề tài giá sinh viên – ZALO:0973.287.149-TEAMLUANVAN.COM
Trình bày các bài toán liên quan đến tâm tỷ cự ở mức độ khó
hơn, gồm hai nội dung:
Tải tài liệu tại sividoc.com
Viết đề tài giá sinh viên – ZALO:0973.287.149-TEAMLUANVAN.COM
3
3.1. Chứng minh một số định lý nổi tiếng của hình học sơ cấp.
3.2. Một số bài toán thi học sinh giỏi và thi Olympic.
- Mặc dù đã rất cố gắng nhưng luận văn không tránh khỏi những
hạn chế, khiếm khuyết. Tác giả rất mong sự góp ý, bổ sung của
các đồng nghiệp và các thầy cô giáo nhằm làm cho kết quả
nghiên cứu hoàn chỉnh và có ích hơn. Xin chân thành cảm ơn.
Tác giả.
Tải tài liệu tại sividoc.com
Viết đề tài giá sinh viên – ZALO:0973.287.149-TEAMLUANVAN.COM
4
Chương 1
Tâm tỷ cự của hệ chất điểm
Các khái niệm ở đây được xét trong mặt phẳng hoặc trong không gian.
Thuật ngữ "barycentric" được nhiều tác giả dịch là "tâm tỷ cự" hoặc "khối
tâm",. . . Thực ra sử dụng các từ này chỉ đúng nghĩa một phần bởi
"barycentric" chỉ liên quan đến đoạn thẳng và các khái niệm quen thuộc trong
cơ học. Đến nay "barycentric" đã được toán học hóa dựa vào khái niệm không
gian véc tơ thì các cách Việt hóa như trên có những hạn chế nhất định. Trong
luận văn này chúng tôi vẫn sử dụng chữ "tâm tỷ cự" do tính chất lịch sử của
khái niệm và phù hợp với các tài liệu hiện hành (xem [1]). Các ký hiệu cũng
được tham khảo và vận dụng vào việc trình bày cho thuận tiện nhất.
1.1 Hệ chất điểm và tâm tỷ cự
Mệnh đề 1.1. Cho hai điểm A;B và hai số thực m1;m2 không đồng thời bằng
0. Khi đó
i. Nếum + m = 0 thì không có Z sao cho ! ! ~
1 2
m ZA + m ZB=0:
1 2
ii. Nếu m1 + m26= 0 thì tồn tại duy nhất điểm Z sao cho
! !
m1ZA + m2ZB =~0:
Khi Z thỏa mãn đẳng thức trên thì với mọi điểm O ta luôn có:
!
!!
m OA + m OB
OZ =
1 2
:
m1 + m2
Tải tài liệu tại sividoc.com
Viết đề tài giá sinh viên – ZALO:0973.287.149-TEAMLUANVAN.COM
5
Chứng minh.
! ! ! ! !
i. Ta có
!
~ () ~ ()
1 2 1 2 1 2
ZB = 0 ZA (ZA+AB) = 0 )ZA+
!
m ZA+m m +m (m +m
~ Nếu thì không có .
= 0: m + m = 0 Z
m AB
2
2 1
m2
m + m = AZ = AB; chứng tỏ Z xác
ii. Nếu 1 2 6 0 thì đẳng thức trên là
!
!
(m1 + m2)
định và duy nhất.
m OA + m OB ta có:
Với O tùy ý, xen điểm Z vào ! !
! ! ! ! 1 ! 2 !
m OA + m OB = m (OZ + ZA) + m (OZ + ZB)
1 2 1
! !
2
!
= (m
!
+ m )OZ + (m ZA + m ZB) = (m + m )OZ.
1 2 1 2 1 2
Mệnh đề 1.2. Cho ba điểm A;B;C và ba số thực m1;m2;m3 không
đồng thời bằng O. Khi đó,
i. Nếu m1 + m2 + m3 = 0 thì không có Z sao cho
!
+
!
+
!
=~: m1ZA m2ZB
m3ZC 0
ii. Nếu m1 + m2 + m3 6= 0 thì tồn tại duy nhất điểm Z sao cho
!
+
!
+
!
=~ m1ZA m2ZB
m3ZC 0
Khi Z thỏa mãn đẳng thức trên thì với mọi điểm O ta luôn có:
!
! ! !
m OA + m OB + m ZC
OZ =
1 2 3
: (1.1)
m1 + m2 + m3
Chứng minh. Chứng minh tương tự mệnh đề 1.1.
Nhận xét 1.1.
! !
Trong trường hợp m1 = m2 = m3 6= 0 thì đẵng thức (1.1) trở thành ZA + ZB +
! ~
()
ZC=0 Z G trọng tâm tam giác ABC.
Mệnh đề 1.3. Cho n điểm A1;A2; ::;An và n số thực m1;m2; :::;mn
không đồng thời bằng O . Khi đó
i. Nếu m1 + m2 + ::: + mn = 0 thì không có Z sao cho
!
+
!
+:::+
!
=~:
Tải tài liệu tại sividoc.com
Viết đề tài giá sinh viên – ZALO:0973.287.149-TEAMLUANVAN.COM
m1ZA1 m2ZA2mnZAn 0
Tải tài liệu tại sividoc.com
Viết đề tài giá sinh viên – ZALO:0973.287.149-TEAMLUANVAN.COM
6
ii. Nếu m1 + m2 + ::: + mn 6= 0 thì tồn tại duy nhất điểm Z sao cho
!
+
!
+:::+
!
=~:
m1ZA1 m2ZA2mnZAn 0
Khi có Z thỏa mãn (1.2) thì với mọi điểm O ta luôn có :
!
m
!
1OA1 + m
!
2OA2 + ::: + m
!
nOAn
OZ = :
m1 + m2 + ::: + mn
(1.2)
(1.3)
m + m + ::: + m =
~
Chứng minh. i. Ta có
!
1ZA1
!
2ZA2
!
nZAn 0
()
m + m ( + ) + ::: + m ( + ) =
~
!
1ZA1
!
2ZA1
!
A1A2
!
nZA1
!
A1An 0
() ( 1 +:::+ n ) 1 + m 2 1 2 + ::: + m n 1 n =
~
:
Từ đây ta có:
m m!ZA !A A !A A 0
Nếu m1 + m2 + ::: + mn = 0 thì không có Z. với A Z =
ii. Nếu
m + m + ::: + m =
0 thì đẳng thức trên tương đương
12 n
6 !
1
m + m + ::: + m
!
2A1A2
!
3A1A3
!
nA1An
; chứng tỏ Z xác định và duy nhất. Với mọi
(m1 + m2 + ::: + mn)
điểm O, từ m
!
1OA1 + m
!
2OA2 + ::: + m
!
nOAn ta có:
m
!
1OA1 + m
!
2OA2 + ::: + m
!
nOAn =
! !
= m
! ! ! !
) + ::: + m )
(OZ + ZA ) + m (OZ + ZA (OZ + ZA
1 1 2 2 n n
= (m + m
! ! !
+ ::: + m )OZ + (m ZA + m ZA 2
1 2 n 1 1 2
= (m + m
!
+ ::: + m )OZ. Từ đó suy ra (1.3).
1 2 n
!
+ ::: + mnZAn)
Định nghĩa 1.1. Giả sử P là tập hợp điểm trên mặt phẳng, tích
Decasterte R P được gọi là "một hệ chất điểm" trong mặt phẳng. Mỗi
chất điểm có hai thành phần, được viết là mA hoặc m:A hoặc (m;A) 2
R P, thành phần thứ nhất là số, thành phần thứ hai là điểm.
Định nghĩa 1.2. Điểm Z xác định duy nhất từ hệ thức (1.3) với các số
thực m1;m2; :::;mn thoả mãn điều kiện m1 + m2 + ::: + m3 6= 0 được
gọi là tâm tỷ cự của hệ chất điểm fmiAig
n
i=1, với S
n
i=1mi 6= 0 và viết
Z [m1A1;m2A2; :::;mnAn] hay ký hiệu tắt là Z [miAi]1 i n:
Ký hiệu I [1A;1B], tức I là tâm tỷ cự của hệ chất điểm f1A;1Bg, đó là
trung điểm của đoạn AB. Khi G là trọng tâm tam giác ABC, ta viết G
[1A;1B;1C] (trọng tâm là tâm tỷ cự của 3 đỉnh tam giác).
Tải tài liệu tại sividoc.com
Viết đề tài giá sinh viên – ZALO:0973.287.149-TEAMLUANVAN.COM
7
1.2 Các tính chất cơ bản của tâm tỷ cự
Trước hết ta xét 3 tính chất sau của tâm tỷ cự.
Tính chất 1.1. Mỗi hệ hữu hạn các chất điểm fm1A1; :::;mkAng với m1 + :: +
mn 6= 0 đều xác định duy nhất tâm tỷ cự của hệ sai khác một hằng số khác
không, tức là tồn tại duy nhất Z sao cho Z [miAi]1 i n [kmiAi]1 i n;k 6= 0:
Chứng minh. Thật vậy, chọn O tùy ý khi đó Z xác định duy nhất theo
đẳng thức véc tơ (1.3). Ta có thể viết (1.3) ở dạng sau với k 6= 0:
!
km
!
1OA1 + km
!
2OA2 + ::: + km
!
nOAn
OZ = :
km1 + km2 + ::: + kmn
Tính chất 1.2. (Quy tắc Archimedes) Tâm tỷ cự của hệ hai chất điểm
fm1A1;m2A2g nằm trên đoạn thẳng (hoặc đường thẳng) nối hai điểm
A1;A2. Vị trí tâm tỷ cự xác định theo "quy tắc cân bằng của đòn bẩy" của
Archimedes (gọi là quy tắc Archimedes): jm1jd1 = jm2jd2.
Hình 1.1: Quy tắc Archimedes
Chứng minh. Giả sử Z là tâm tỷ cự của hệ hai chất điểm fm1A1;m2A2g. Khi
đó: !
1ZA1
!
2ZA2 0 () !
1ZA1
=
!
2ZA2
:
Nếu 1
;m
2 cùng dấu (có
m + m =
~
m m m
thể coi
m ;m >
0) ta thấy các véc
tơ ZA ;ZA ngược hướng nên điểm Z nằm
1 2 ! ! 2
1
trên đoạn thẳng A A, hơn nữa, m
!
ZA
1
j= m
!
ZA
2
j; tức là m d
1
= m d . Nếu
m ;m 1 2 1
j 2
j 1 2 2
tơ ZA ;ZA cùng hướng nên điểm Z nằm ngoài
1 2
trái dấu ta thấy các véc ! !
2
j 1jj 1 j 2jj 2
j
j 1
j
j 2
j
2
đoạn thẳng
1 2 1
j 1
.
A A, ngoài ra,m
!
ZA = m
!
ZA ; tức làm d = m d
Tải tài liệu tại sividoc.com
Viết đề tài giá sinh viên – ZALO:0973.287.149-TEAMLUANVAN.COM
Từ đây ta cũng thấy tâm tỷ cự của hệ hai điểm ở gần điểm có
"trọng lượng" lớn hơn trong hai "trọng lượng" của hai chất điểm.
Tải tài liệu tại sividoc.com
Viết đề tài giá sinh viên – ZALO:0973.287.149-TEAMLUANVAN.COM
8
Hệ quả 1.1. Điểm M chia đoạn thẳng AB theo tỷ số
b
khi và chỉ khi M
a
[aA;b B].
Tính chất 1.3. (Tính chất kết hợp) Giả sử ta lấy ra k chất điểm fm1A1;m2A2; :::;mkAkg
trong hệ n chất điểm fm1A1;m2A2; :::;mnAng và gọi C là tâm tỷ cự của hệ k chất điểm
đó. Khi đó hệ chất điểm ban đầu có cùng tâm tỷ cự với hệ chất điểm là:
f(m1 + m2 + ::: + mk)C;mk+1Ak+1; :::;mnAng:
Chứng minh. Gọi Z là tâm tỷ cự của hệ n chất điểm, ta có:
!
+:::+
!
+
!
+:::+
!
=~:
m
1
ZA
1
m
k
ZA
k
m
k+1
ZA
k+1
m
n
ZA
n
0
Vì C là tâm tỷ cự của hệ fm1A1;m2A2; :::;mkAkg nên ta có:
! m + ::: + m
ZC =
!
1ZA1
!
kZAk
:
m1+ ::: + mk
Từ hai đẳng thức trên ta có
! ! ! ~
(m + ::: + m )ZC + m ZA + ::: + m ZA
k+1 k+1 n
= 0:
1 k n
Đó là điều cần chứng minh.
Từ tính chất kết hợp ta có các hệ quả hiển nhiên sau.
Hệ quả 1.2. Nếu Z là tâm tỷ cự của hệ 3 điểm là đỉnh tam giác ABC.
Khi đó đường thẳng AZ cắt cạnh BC ở điểm A
0
là tâm tỷ cự của hệ
hai chất điểm đặt tại B và C.
Hệ quả 1.3. Giả sử tại các đỉnh A,B,C của tam giác ABC theo thứ tự đặt các
trọng lượng m1;m2;m3. Nếu B
0
là tâm tỷ cự của hệ fm1A;m3Cg; C
0
là tâm tỷ
cự của hệ fm1A;m2Bg thì Z = BB
0
 CC
0
là tâm tỷ cự của hệ ba điểm
fm1A;m2B;m3Cg.
Ký hiệu thu gon
Trong công thức (1.3), O là điểm tùy ý trong không gian nên có thể quy ước
bỏ điểm O và không dùng ký hiệu véc tơ. Như thế (1.3) được ký hiệu thu gọn là
Z =
m1A1 + m2A2 + ::: + mnAn
(1.4)
m1 + m2 + ::: + mn
Tải tài liệu tại sividoc.com
Viết đề tài giá sinh viên – ZALO:0973.287.149-TEAMLUANVAN.COM
Tải tài liệu tại sividoc.com
Viết đề tài giá sinh viên – ZALO:0973.287.149-TEAMLUANVAN.COM
9
hay (m1 + m2 + ::: + mn)Z = m1A1 + m2A2 + ::: + mnAn (1.5)
Mỗi ký hiệu thu gọn nói trên khẳng định điểm Z là tâm tỷ cự của hệ chất điểm
m1A1;m2A2; :::;mnAn. Khi viết P =
2A + 3B + 8C
nghĩa là điểm P là tâm tỷ cự
13
của hệ điểm f2A;3B;8Cg. Từ tính chất kết hợp ta có thể sử dụng ký
hiệu thu gọn linh hoạt hơn. Chẳng hạn, ký hiệu
P = (2A+3B)+8C = 5D+8C =) P 2 CD
13 13
diễn tả bằng lời:" Giả sử P là tâm tỷ cự của hệ chất điểm f2A;3B;8Cg, nếu
tâm tỷ cự của hệ f2A;3Bg là D thì điểm P là tâm tỷ cự của hệ f5D;8Cg".
1.3 Tâm tỷ cự và diện tích đại số
1.3.1 Diện tích đại số
Để xét tính chất quan trọng của tâm tỷ cự liên quan đến diện tích,
ta giới thiệu diện tích đại số thông qua khái niệm tích ngoài như sau.
Định nghĩa 1.3. Trong mặt phẳng định hướng diện tích đại số của tam
giác định hướng ABC, ký hiệu là ABC, là một số thực mà trị tuyệt đối
của nó là diện tích (hình học) của tam giác đó với dấu + hay tùy theo
tam giác ABC có hướng thuận hay nghịch: ABC = S(ABC):
Trường hợp DABC suy biến thì ABC = 0 , C 2 AB:
Định nghĩa 1.4. Tích ngoài (hay tích phản vô hướng) của hai véc tơ ~u;~v, ký
~ ~
hiệu là ~u ^~v là một số thực bằng 0 khi ~u = 0 hoặc~v = 0, bằng j~ujj~vjsin(~u;~v) khi
~
~u =~v 6= 0;.
Ta có nhận xét ngay:
! ^
!
ABC
= 1
AB AC(từ đó tam giác ABC định hướng thuận khi và chỉ khi
! !
2
AB^AC > 0).
~v ^~u = ~u ^~v; ~u và ~v cùng phương ()~u ^~v = 0:
Dạng tọa độ. Xét mặt phẳng tọa độ Oxy, gọi
~
i;
~
j là hai véc tơ chỉ phương đơn
Tải tài liệu tại sividoc.com
Viết đề tài giá sinh viên – ZALO:0973.287.149-TEAMLUANVAN.COM
vị của hai trục Ox, Oy, góc định hướng (
~
i;
~
j) =
p
[mod2p]. Khi đó nếu ~u
= 2
Tải tài liệu tại sividoc.com
Viết đề tài giá sinh viên – ZALO:0973.287.149-TEAMLUANVAN.COM
10
(u1;u2);~v = (v1;v2) thì
u1 v1
~u ^~v = u1v2 u2v1 = :
u 2 v 2
~
Thật vậy, khi ~u hay ~v bằng 0 thì rõ ràng u1v2 u2v1 = 0.
~
Khi ~u và ~v khác 0 thì ta có:
sin(~u;~v) = sin((
~
i;~v) (
~
i;~u)) = sin(
~
i;~v)cos(
~
i;~u) cos(
~
i;~v)sin(
~
i;~u) =
v2 u1 v1 u2
nên ~u ^~v = u1v2 u2v1.
~v
j
~u ~v
j
~u
j j
Từ đó dễ kiểm tra được
Hệ quả 1.4. ( ~) ^~ = ~ ^ ~ = (~ ^~); (~ +
~0
) ^~ = ~ ^~ +
~0
^~: ku v u kv k
u v u u v u v u v
Diện tích đại số ABC có các tính chất sau:
a. Với mọi 4 điểm A, B, C, M ta có hệ thức Chasles:
ABM + BCM + CAM = ABC:
b. Cho DABC. Với 2 điểm B
0
;C
0
2 BC ta có:
BC ABC
B0C0
=
AB0C0
c. Cho tam giác ABC và điểm O. Giả sử các đường thẳng AO;BC cắt nhau
tại M (khác B,C). Ta có:
MB = OBA
MC OCA
d. Cho tam giác ABC và M thuộc đường thẳng BC. Ta có:
MB = MBA
MC MCA
Chứng minh.
! !!!!!
a. Thật vậy, AB ^AM + BC ^BM + CA ^CM
! !!!!!!!
= AB^(AC+CM)+BC^(BC+CM)+CA^(CM
! !!!!! !!
= AB^(AC+(AB+BC+CA)^CM = AB^(AC:
Các đẳng thức b., c., d. là hiển nhiên.
Tải tài liệu tại sividoc.com
Viết đề tài giá sinh viên – ZALO:0973.287.149-TEAMLUANVAN.COM
Tải tài liệu tại sividoc.com
Viết đề tài giá sinh viên – ZALO:0973.287.149-TEAMLUANVAN.COM
11
Tính chất sau đặc biệt quan trọng khi ta chuyển sang xét tọa độ tâm tỷ
cự của một điểm đối với tam giác ABC (không suy biến). Chính vì tính
chất này mà tọa độ tỷ cự còn được gọi là "tọa độ diện tích".
Tính chất 1.4. DABC và điểm M tùy ý trong mặt phẳng. Khi đó
(1.6)
M [MBC:A;MCA:B;MAB:C]:
Chứng minh.
Hình 1.2: Tọa độ diện tích
Trong các đường thẳng AM,BM,CM phải có ít nhất một đường
thẳng không song song với BC, CA, AB theo thứ tự đó vì chẳng hạn
có BM k AC;CM k AB thì tứ giác ABMC là hình bình hành nên AM
phải cắt BC. Bởi vậy, ta có thể coi AM cắt BC ở điểm A
0
.
! ! !
Bước 1. Ta có BCMA0
= (A0
C A0
B) = A0
CMA0
A0
BMA0
!MB BA0 !MC CA0
A0C A0B
= ( !
+ ) ( !+ )
! ! !
=
A0
! 0 0 0
! 0 0 0
! 0
! 0 0
C
C:MB+BA:A C A B:MC CA :A B. = A C:MB A B:MC+BA :A
! ! A0
C A0
B
CA0:A0B. Từ đây ta suy ra: MA0 = ! MB ! MC.
BC BC
Bước 2. Theo tính chất về độ dài đại số và diện tích đại số:
A
0
C = AA
0
C = MA
0
C = MCA . Suy raA
0
C = A
0
C
A
0
B ABA
0
MBA
0
MAB BC A
0
C A
0
B
= MCA . Tương tự, A0
B = MAB .
BC
MCA MAB ! MCA + MAB
Thay vào đẳng thức tính MA
0
ở trên ta được:
! MCA !
:MB +
MAB !
:MC:
MA0 =
MCA + MAB MCA + MAB
Bước 3. Vì MA0
= MCA0
=MA0
B = MCA0
+ MA0
B nên
Tải tài liệu tại sividoc.com
Viết đề tài giá sinh viên – ZALO:0973.287.149-TEAMLUANVAN.COM
MA MCA MAB MCA + MAB
Tải tài liệu tại sividoc.com
Viết đề tài giá sinh viên – ZALO:0973.287.149-TEAMLUANVAN.COM
12
! MBC
!
MCA
!
MAB
!
MA0 = MA = MB + MC. Suy ra
MCA + MAB MCA + MAB MCA + MAB
~
tương đương (1.6).
MBC:MA + MCA:MB + MAB:MC = 0
! ! !
Từ tính chất này ta suy ra các hệ quả.
Hệ quả 1.5. Nếu M nằm trong miền tam giác ABC thì
M [S
(MBC)
A;S
(MCA)
B;S
(MAB)
C]:
(1.7)
(M là tâm tỷ cự của hệ chất điểm fS(MBC)A;S(MCA)B;S(MAB)Cg)
Hệ quả 1.6. Nếu M ở ngoài miền tam giác ABC và ở trong BACd thì
M [ S
(MBC)
A;S
(MCA)
B;S
(MAB)
C]:
(1.8)
(M là tâm tỷ cự của hệ chất điểm f S(MBC)A;S(MCA)B;S(MAB)Cg). Tương tự như thế nếu M ở ngoài tam
giác nhưng ở trong
[
hay ở trong
[
.
ACB CBA
1.3.2 Tọa độ tỷ cự trong mặt phẳng
Ta cố định tam giác ABC, gọi nó là tam giác cơ sở (không suy
biến). Khi đó theo 1.6, mọi điểm M trong mặt phẳng đều có M
[MBC:A;MCA:B;MAB:C]. Từ đó ta định nghĩa
Định nghĩa 1.5. Giả sử ABC là tam giác cơ sở. Tọa độ tỷ cự của điểm
M đối với tam giác ABC là bộ ba số (x : y : z) sao cho
x : y : z = MBC : MCA : MAB
Ta có tọa độ tỷ cự của một số điểm đặc biệt trong tam giác ABC: Cho 4 ABC,
Gọi G, I, O, H, Oa lần lượt là trọng tâm, tâm đường tròn nội tiếp, tâm đường
tròn ngoại tiếp, trực tâm, tâm đường tròn bàng tiếp trong góc A. Khi đó ta có:
a. G(1A: 1B: 1C),
b. I(aA: bB: cC),
c. O [sin2A.A, sin2B.B, sin2C.C] hay dưới dạng:
O [a
2
(b
2
+ c
2
a
2
)A,b
2
(c
2
+ a
2
b
2
)B,c
2
(a
2
+ b
2
c
2
)C].
Tải tài liệu tại sividoc.com
Viết đề tài giá sinh viên – ZALO:0973.287.149-TEAMLUANVAN.COM
13
d.O
a
[-S
(OaBC)
A,S
(OaCA)
B,S
(OaAB)
C] [-aA, bB, cC]
e.H [tan A.A, tan B.B, tanC.C] hay dưới dạng:
H [
1
A,
1
B,
1
C].
b2
+ c2
a2
c2
+ a2
b2
a2
+ b2
c2
(Xem chứng minh trong ví dụ 2.9)
Khi M có tọa độ tỷ cự là (x : y : z) mà x + y + z 6= 0 thì ta nói (x : y : z) là tọa
độ tỷ cự tuyệt đối của M, nếu x + y + z = 1 thì (x : y : z) được gọi là tọa độ tỷ cự
chuẩn của M. Từ tính chất của tâm tỷ cự ta rút ra: nếu M và N có tọa độ tỷ cự
tuyệt đối, điểm X chia đoạn MN theo tỷ số MX : XN = m : n sẽ có tọa độ tỷ cự
tuyệt đối là
n:M + m:N
. Tuy nhiên để thuận tiện trong tính toán ta sẽ tính tọa độ m
+ n
của X theo cách tương đương như sau: Nếu M = (u : v : w);N = (u0
: v0
: w0
) mà
u + v + w = u0
+ v0
+ w0
thì điểm X sẽ có tọa độ tỷ cự là (nu + mu0
: nv + mv0
:
nw + mw
0
).
Các sự kiện cơ bản trong tọa độ tỷ cự
(a) Phương trình đường thẳng đi qua 2 điểm P, Q:
Khi P(a1 : b1 : c1); Q(a2 : b2 : c2) thì PQ được xác định bởi
Dax + Dby + Dcz = 0; trong đó,
Da = c 1 c 2; Db
b b
1 2
= a1 a2; Dc = b 1 b 2; hoặc định thức cấp 3:
c c a 1 a 2
1 2
x y z = 0:
a1 b1 c1
a2 b2 c2
Từ đó suy ra rằng điểm R(x
0
: y : z
0
) nằm trên PQ khi và chỉ khi
0
x0 y0 z0 = 0:
a1 b1 c1
a2 b2 c2
(b) Giao điểm của 2 đường thẳng a
1
x + by + c
1
z = 0 và a
2
x + b
2
y + c
2
z = 0
1
Tải tài liệu tại sividoc.com
Viết đề tài giá sinh viên – ZALO:0973.287.149-TEAMLUANVAN.COM
xác định bởi công thức M(b1c2 b2c1 : c1a2 c2a1 : a1b2 a2b1): Từ đó suy ra
Tải tài liệu tại sividoc.com
Viết đề tài giá sinh viên – ZALO:0973.287.149-TEAMLUANVAN.COM
14
điều kiện để ba đường thẳng có phương trình aix + biy + ciz = 0;i =
1;2;3 đồng quy là
a
1
b
1
c
1 = 0:
a2 b2 c2
a
3
b
3
c
3
(c) Điều kiện để hai đường thẳng aix + biy + ciz = 0;i = 1;2 song song:
(a1 b1 : b1 c1 : c1 a1) = (a2 b2 : b2 c2 : c2 a2)
(d) Trung điểm của đoạn thẳng BC là điểm M(0 : 1 : 1); Tiếp điểm với cạnh
BC của đường tròn nội tiếp DABC là D(0 : 1 : 1 ) và tương tự với các
p b p c
cạnh còn lại.
(d) Phương trình đường tròn ngoại tiếp tam giác cơ sở ABC: a
2
yz +
b
2
zx + c
2
xy = 0:
(e)Hai điểm P(x1 : y1 : z1);Q(x2 : y2 : z2) liên hợp đẳng giác với nhau
khi và chỉ khi tồn tại số k 2 R để có x1x2 = ka
2
;y1y2 = kb
2
:z1z2 = kc
2
:
Diện tích tam giác
Lấy ABC là tam giác cơ sở. Giả sử P(p1 : p2 : p3);Q(q1 : q2 :
q3);R(r1 : r2 : r3) có tọa độ tỷ cự chuẩn hóa theo ABC. Khi đó
=
p
1
q
1
r
1
: : (1.9)
PQR p2 q2 r2
ABC
p3 q3 r3
Chứng minh.
Với mọi điểm O,
!
! ! ! ! !
+
OP = p OA + p OB + pOC; OQ = q OA
! ! 1 2 3 1
q OB + q OC. Từ đó,
2 3
!
= (q
1 1 ! 2 2!
+ (q
3 3!
PQ p )OA + (q p )OB p)OC:
Lấy O
PQ = (q p )CA + (q p )CB. Tương tự, PR = (r
C ta có:!
1 1
!
2
! !
1
! ! ! 2 !
p )CA + (r
2
p )CB. Ta nhận được: PQR = 1
PQ
^
PR =
1
1
2 ! ! 1
2 ! !
= (q 1 p 1 )(r 2 q )CA ^ CB + (q 2 p )(r 1 p)CB ^ CA:
Tải tài liệu tại sividoc.com
Viết đề tài giá sinh viên – ZALO:0973.287.149-TEAMLUANVAN.COM
2 2 2 2 1
Tải tài liệu tại sividoc.com
Viết đề tài giá sinh viên – ZALO:0973.287.149-TEAMLUANVAN.COM
15
Vì ! ! ! !
ABC 2 ^
CB =
2 ^
CA nên ta tìm được:
=
1
CA
1
CB
PQR = ((q1 p1)(r2 p2) (q2 p2)(r1 p1))ABC =
= [(p1q2 p2q1) + (q1r2 q2r1) + (r1 p2 r2 p1)]ABC:
Cuối cùng sau khi nhân biểu thức (...)thứ nhất trong ngoặc vuông với r1 +
r2 + r3 = 1, biểu thức (...)thứ hai trong ngoặc vuông với p1 + p2 + p3 = 1, biểu
thức (...)thứ ba trong ngoặc vuông với q1 + q2 + q3 = 1 ta nhận được (1.9).
1.4 Công thức Lagrang và công thức Jacobi
Định nghĩa 1.6. Giả sử có hệ chất điểm m1A1;m2A2; :::;mnAn. Mô men
quán tính của hệ đối với điểm S được Euler định nghĩa là đại lượng
n
JS = m1jSA1j
2
+ m2jSA2j
2
+ ::: + mnjSAnj
2
= å mkjSAkj
2
(1.10)
k=1
Chẳng hạn, ABCD là hình vuông với cạnh 2a, tại mỗi đỉnh đặt trọng
lượng 1. Gọi S là trung điểm của AB, Z là tâm hình vuông. Ta hãy tính
các mô men quán tính: JS và JZ. Ta có
p
JS = 1:jSAj
2
+ 1:jSBj
2
+ 1:jSCj
2
+ 1:jSDj
2
= 2a
2
+ 2(a 5)
2
= 12a
2
.
p
JZ = 1:jZAj
2
+ 1:jZBj
2
+ 1:jZCj
2
+ 1:jZDj
2
= 4(a 2)
2
= 8a
2
.
Công thức sau đây được nhà toán học Pháp Lagrang tìm ra thể hiện mối liên
hệ giữa mô men quán tính JS của hệ chất điểm đối với điểm S tùy ý và mô
men quán tính JZ cũng trong hệ chất điểm đó đối với tâm tỷ cự Z của nó.
Tính chất 1.5. Giả sử Z[m1A1;m2A2; :::;mnAn]. Khi đó đối với điểm S tùy ý
ta có:
JS = JZ + mjSZj
2
trong đó,m = m1 + m2 + ::: + mn: (1.11)
n ! n ! !
Chứng minh. Ta có, J = å å )
2
=
S m SA 2
= m (SZ + ZA
k k k k
k=1 k=1
n ! n ! ! n !
å å å ):
m ZA 2
+( m )SZ2
+ 2SZ: m (ZA
k k k k k
k=1 k=1 k=1
n ( ) =
~
;
ta thu được (1.11).
Vì Z là tâm tỷ cự nên å m
!
kZAk 0
Tải tài liệu tại sividoc.com
Viết đề tài giá sinh viên – ZALO:0973.287.149-TEAMLUANVAN.COM
k=1
Tải tài liệu tại sividoc.com
Viết đề tài giá sinh viên – ZALO:0973.287.149-TEAMLUANVAN.COM
16
Một công thức khác được nhà toán học Đức K.G. Jacobi tìm ra
biểu diễn mô men quán tính JZ của hệ chất điểm (với tổng trọng
lương khác không) đối với tâm tỷ cự Z của nó qua trọng lượng của
các chất điểm và khoảng cách AiAj giữa chúng.
Tính chất 1.6. Giả sử Z[m1A1;m2A2; :::;mnAn]. Khi đó,
JZ =
1
å mim jjAiAjj2
: (1.12)
m
1 i< j n
Chứng minh. Xét n = 3 (trường hợp tổng quát làm tương tự). Ta có công
thức Lagrang: JA1 = JZ + mjA1Zj
2
. Từ đó, sau khi nhân với m1 nhận được
m1m2jA1A2j
2
+ m1m3jA1A3j
2
= m1JZ + mm1jA1Zj
2
Tương tự ta có (mô men quán tính đối với các điểm A2;A3):
m2m1jA2A1j2
+ m2m3jA2A3j2
= m2JZ + mm2jA2Zj2
m3m1jA3A1j2
+ m3m2jA3A2j2
= m3JZ + mm3jA3Zj2
:
Cộng vế với vế các đẳng thức trên ta nhận được tổng các tích mim
jjAiAjj
2
với i 6= j. Mỗi tích như thế ta gặp 2 lần. Nếu ta chỉ viết các số
hạng với i < j thì ta nhận được
2 å mim jjAiAjj2
= mJZ + mJZ
1 i< j 3
tương đương với công thức (1.12) với n = 3:
Kết hợp các công thức 1.11 và 1.12 ta có công thức tính khoảng
cách từ một điểm bất kỳ đến tâm tỷ cự của hệ điểm.
n
Hệ quả 1.7. Giả sử Z [m1A1;m2A2; :::;mnAn] với å mi = 1. Khi đó đối với
điểm S tùy ý ta có công thức khoảng cách
i=1
n
å mimjAiAj
2
SZ
2
= JS JZ = å miSAi
2
i=1 1 i<j n
Tải tài liệu tại sividoc.com
Viết đề tài giá sinh viên – ZALO:0973.287.149-TEAMLUANVAN.COM
17
3
Chẳng hạn với Z [m1A1;m2A2;m3A3]; å mi = 1 thì SZ
2
=
i=1
m1SA1
2
+ m2SA2
2
+ m3SA3
2
(m1m2A1A2
2
+ m1m3A1A3
2
+
m2m3A2A3
2
):
Ví dụ 1.1. Cho tứ giác ABCD, cạnh và đường chéo có các độ dài
a;b;c;d;e; f . Tại mỗi đỉnh của tứ giác đặt trọng lượng 1. Tính mô men
quán tính của hệ chất điểm này đối với trung điểm Z của đoạn thẳng
PQ nối trung điểm các đường chéo.
Lời giải. Điểm Z là tâm tỷ cự của hệ chất điểm đang xét. Theo công
thức Jacobi ta có:
JZ =
1
(a2
+ b2
+ c2
+ d2
+ e2
+ f2
):
4
Đó chính là mô men quán tính của hệ chất điểm này đối với trung
điểm Z của đoạn thẳng PQ.
Nội dung của chương một là nhắc lại khái niêm hệ chất điểm, tâm tỷ cự
của hệ chất điểm và hệ thống các tính chất cần thiết của tâm tỷ cự. Các
phép chứng minh đều dựa vào công cụ véc tơ. Trong chương tiếp theo của
luận văn ta sẽ xây dựng các kỹ thuật dựa vào các tính chất đó và áp dụng
khái niệm mô men quán tính vào giải các bài toán hình học. Đây cũng là
những kỹ thuật khá điển hình khi khai thác khái niệm tâm tỷ cự.
Tải tài liệu tại sividoc.com
Viết đề tài giá sinh viên – ZALO:0973.287.149-TEAMLUANVAN.COM
18
Chương 2
Các kỹ thuật biến đổi tâm tỷ cự và
ứng dụng
Từ các tính chất cơ bản ta nêu ra các kỹ thuật được sử dụng thường
xuyên và có hiệu quả khi giải các bài toán hình học. Ta lần lượt trình bày các
kỹ thuật biến đổi dựa vào các tính chất của tâm tỷ cự trên mặt phẳng. Mỗi kỹ
thuật được nêu thành các bước vận dụng, các ví dụ và các bài toán mẫu.
2.1 Kỹ thuật chọn tâm tỷ cự
Kỹ thuật chọn tâm tỷ cự dựa vào tính chất 1.1 và 1.2. Chọn tâm tỷ cự
thích hợp đi liền với đặt trọng lượng tương ứng làm cho lời giải bài toán
đơn giản hơn. Ở đây lưu ý đến quy tắc Archimedes (tính chất 1.2).
Ví dụ 2.1. Trên cạnh AC của tam giác ABC lấy điểm M sao cho AM =
1
AC. 3
Trên phần kéo dài của tia CB lấy điểm N sao cho BN = BC. Đường thẳng
MN cắt cạnh AB tại điểm P. Điểm P chia các đoạn thẳng AB và NM theo
tỷ số nào? Lời giải. Rõ ràng tại N và C cần đặt các trọng lượng bằng
nhau vì khi đó tâm tỷ cự của hai điểm này là B, bởi vậy, ta sẽ đặt tại mỗi
điểm N và C các chất điểm 1N, 1C. Tiếp theo vì CM=2AM nên theo quy
tắc Archimedes, M [1C, 2A]. Do đó Z [1N, 1C, 2A] [1N, 3M] (theo tính
chất kết hợp), tức là Z 2 MN. Vậy Z = AB  MN hay Z P. Vì Z [1N, 3M]
[2A, 2B] nên theo quy tắc Archimedes ta có NP : PM = 3:1; AP:PM = 1:1.
Chú ý. Ta có thể đưa ra cách giải bằng phương pháp véc tơ thông
qua cách giải trên.
Tải tài liệu tại sividoc.com
Viết đề tài giá sinh viên – ZALO:0973.287.149-TEAMLUANVAN.COM
19
Tức là cách giải thuần túy véc tơ không nói gì đến ”chất điểm” hay ”tâm tỷ cự”:
Cố định một điểm O nào đó, từ giả thiết ta có:
! ! !! ! !
2MA + MC = 0 ; BC + BN = 0 và tương đương với các đẳng thức véc tơ sau:
!
OM= ! !
!
;OM= ! ! :
2OA + OC 2OA + OC
3 3
Hình 2.1: Chọn tâm tỷ cự
! ! !
! 2OA+OC+ON
Xét điểm Z cho bởi điều kiện OZ =
! ! 4
Ta có:
! 3OM + ON ) ! ! ! ) 2
4
OZ =
!
3ZM +ZN= 0 Z MN:
! ! ! ! ! )
!
! ! 2
2OA 4
+ON) 4
OZ =
+ (OC 2OA + 2OB
ZA+ZB= 0;Z AB. Do đó, Z
=
là giao điểm của các đoạn thẳng AB và MN, tức là Z trùng với P và
bởi vậy, PN:PM = 3; PA = PB.
Rõ ràng cách giải bằng véc tơ là hay và chặt chẽ (ý tưởng của nó xuất phát từ
!
cách sử dụng tâm tỷ cự). Tuy nhiên xét đẳng thức OZ là không tự
nhiên và khó giải thích vì sao lại chọn như vậy.
Kỹ thuật chọn tâm tỷ cự đặc biệt có lợi khi gặp các bài toán tìm
điểm hoặc tìm quỹ tích điểm thoa mãn đẳng thức véc tơ. Ta thường
dùng kỹ thuật này để biến đổi một vế hoặc cả hai vế của một đẳng
thức, tâm tỷ cự nếu tồn tại sẽ đóng vai trò điểm cố định.
Ví dụ 2.2. Cho hình vuông ABCD. Tìm điểm M thỏa mãn
! ! ! ! !
MA +4MB +MC +4MD = 5AD:
Tải tài liệu tại sividoc.com
Viết đề tài giá sinh viên – ZALO:0973.287.149-TEAMLUANVAN.COM
20
Lời giải.
Cách 1. Ký hiệu G là tâm hình vuông ABCD. Ta có:
!MA!!!!!!!!!
()
+4MB + MC+4MD = 5AD () MA+MC+ 4MB + 4MD = 5AD
2
!
! ! ! !
:4MG + 84MG = 5AD
()
MG =
2
1
AD: Vậy M là trung điểm của AB.
~
Cách 2. Chọn 1A
;
4B
;
1C
;
4D
]; GA + 4GB + GC + 4GD = 0: Khi đó
[
G tức
!!
! !
điều kiện của M tương đương với:
! ! ! () !
! ! !!!!
(GA GM) + 4(GB GM) + (GC GM) + 4(GD GM) = 5:AD 10GM
=
!
! :OA + 4:OB + 1:OC + 4:OD
5AD: Cần xác định vị trí của G: Với mọi O, OG =
1
!
! ! !
:
!
IA 10 ! !
!
Từ đẳng thức đó chọn O I (tâm hình vuông) thì ta
có: IG = + 4:IB IA 4:IB
10
!
!
= 0 : Tức G I là tâm hình vuông và M là trung điểm của AB.
Cách 3. Chọn G [1A;4B;1C;4D] [[1A;1C];[4B;4D]] [2I;8I] [10I] I
với I là tâm hình vuông.
Cách 1 ngắn vì rất may ta chọn G đúng là tâm hình vuông. Cách 2 chính
là áp dụng kỹ thuật chọn tâm tỷ cự của hệ chất điểm 1A, 4B, 1C, 4D. Cách 3
ngắn nhất do sử dụng kỹ thuật kết hợp một cách hợp lý. Các bài toán tiếp theo
sẽ khẳng định ưu thế của kỹ thuật này.
Ví dụ 2.3. Cho hình vuông ABCD. Tìm điểm M thỏa mãn
! ! ! ! !
MA +2MB +3MC +4MD = 5AD:
Lời giải. Chọn G là tâm tỷ cự của hệ chất điểm f1A;2B;3C;4Dg. Khi đó:
!!!!!
GA + 2GB +3GC+4GD = 0 .
Điều kiện đề
bài MA + 2MB + 3MC + 4MD = 5AD
! ! ! ! !
! () !
() ! ! ! ! ! ! ! !
(GA GM)+2(GB GM)+3(GC GM)+4(GD GM) = 5AD 10GM
= ! ! !
5AD
()
GM =
2
1
AD:
! ! ! !
Xác định vị trí của G:
!
! !
GA + 2GB + 3GC+ 4GD = O
! !
() ! ! ! ! !
OG = 1(OA + 2OB + 3OC + 4OD):
OG)= O
(OA OG)+2(OB OG)+3(OC OG)+4(OD
()
!
! ! ! !
10
Lấy O I ( tâm hình vuông) thì:
2 ! ! !
! ! ! ! !
IG = 1
(OA + 2OB + 3OC + 4OD) = (OA + OB) = 2
OE với E là trung
5 5
10
điểm của AB.
Tải tài liệu tại sividoc.com
Viết đề tài giá sinh viên – ZALO:0973.287.149-TEAMLUANVAN.COM
21
Hình 2.2: Quĩ tích là đường tròn
mãn IG =
2
IG; I là tâm hình
Quỹ tích các điểm M là đường tròn tâm G (thỏa ! !
1
5
vuông), bán kính GM = IE = AD.
2
Ví dụ 2.4. Cho tam giác ABC. Tìm quỹ tích điểm M sao cho
! ! ! !
;
a. MA + 2MB + 3MC = 0
! ! ! !
;
b. MA + 2MB 3MC= 0
! !!!!!
c. 2 j MA + MB + MC j=j MA + 2MB + 3MC j :
Lời giải.
a. Vì 1 + 2 + 3 = 6 6= 0 nên tồn tại duy nhất điểm M là tâm tỷ cự của hệ chất
! ! !
f g OA +2OB + 3OC
6
điểm 1A
;
2B
;
3C
: có: OM = : Ta
Suy ra với mọi điểm O ta
!
xác định vị trí của M bằng cách cho O vào một trong các vị trí đặc biệt A, B,
!
2OA +3AC = ! !
1 1
C,... . Chẳng hạn O A, ta có AM = ! !
AB + AC: Như vậy, quỹ
6 3 2
!
!
tích điểm của M là một điểm, đỉnh của hình bình hành với hai cạnh =
a 1AB;
= ! 3
!
b 1
AB.
2
b. Vì 1 + 2 - 3 = 0 nên không có M thỏa mãn đẳng thức. Quỹ tích rỗng.
c. Gọi G [1A;1B;1C] và I [1A;2B;3C]. Theo định nghĩa với mọi điểm M
ta ! ! ! ! ! ! ! ! !
có: MG =1 (MA+MB+MC);MI =1 (MA + 2MB +3MC). Từ đó, 2 MA+
6
!
MB!
MC 3 ! ! ! ! j
j !
MA j () j j ()
M cách đều hai
+ = + 2MB + 3MC 2:3 MG =6MI
Tải tài liệu tại sividoc.com
Viết đề tài giá sinh viên – ZALO:0973.287.149-TEAMLUANVAN.COM
22
điểm G và I. Ở đây G là trọng tâm 4ABC còn I xác định như điểm M trong
phần a. Vậy quỹ tích các điểm M cần tìm là đường trung trực của đoạn IG.
Ví dụ 2.5. Cho tam giác ABC và đường thẳng d. Tìm điểm M trên d sao cho
!
jMA
!
MB ! j
đạt giá trị nhỏ nhất.
+ + 3MC
Lời giải. Chọn G [1A;1B;3C] khi đó với mọi điểm M ta có:
!MA!MB !
+ + 3MC
thiết MA + MB + 3MC =5MG = 5MG.
:
!
MG = 5 Theo giả j
! ! j !j ! ! !
j j
Suy !j
nhỏ nhất nếu và chỉ nếu MG nhỏ nhất, tức M là hình
ra MA+MB + 3MC
chiếu của G trên d.
!
MA!
MB !
Xác định vị trí của G: Từ đẳng thức MG = + 5+ 3MC ;
chọn M C
!
thì
! ! ! !
CA+CB 2
CG = = CE với E là trung điểm của AB. Từ đó ta dựng được G
5 5
và hình chiếu của G xuống d.
Ví dụ 2.6. Cho tứ giác ABCD và số k 2 R. Tìm quỹ tích các điểm M thỏa:
!
jMA ! ! j !
j j
+ 3MB MC = k MD :
Lời giải. Chọn G [1A;3B; 1C], nghĩa là với mọi điểm O ta có
! :OA +3:OB 1:OC ! ! ! !
OG = 1!!!
= 1:OA + 3:OB 1:OC =3OG; O.
3 ! ) j j !
j j 8 j j j 8
Với O M thì
!
jMA !
+3MB MC =3MG, M. Bởi vậy đẳng thức đã cho
3 MG = k MD hay 3MG = kMD. Từ đó ta có
tương đương với
!
j j !j j
Nếu k < 0, quỹ tích là 0/.
Nếu k = 3, quỹ tích là đường thẳng trung trực của đoạn GD (G sẽ
được xác định ngay sau đây).
Nếu 0 < k 6= 3, đẳng thức đã cho tương đương với
MG
=
k
, quỹ tích là
MD 3
đường tròn Aponolius dựng trên đoạn GD. ! !
1
Điểm G được xác định: Cho O
BG = BA BC CA.
3 = 3
B thì
!
!
Ví dụ 2.7. Cho tam giác ABC. Tìm quỹ tích điểm M sao cho
!MA ! ! j ! ! ! j
j
+ 3MB 2MC =
j
MB :
2MA MC
Tải tài liệu tại sividoc.com
Viết đề tài giá sinh viên – ZALO:0973.287.149-TEAMLUANVAN.COM
Lời giải. Ta phải biến đổi cả hai vế: Chọn I [1A, 3B, -2C], nghĩa là với
mọi điểm O ta có
Tải tài liệu tại sividoc.com
Viết đề tài giá sinh viên – ZALO:0973.287.149-TEAMLUANVAN.COM
23
1
!
! ! ! ! ! !
!OI =
:OA +3:OB 2:OC ) j j j j 8
O. Cho O M
2
! !
= OA + 3:OB 2:OC =2OI;
ta có vế ! j !
j
+ 3MB 2MC
j j
trái MA =2MI.
Hình 2.3: I là đỉnh thứ tư hình bình hành
Vế phải
!
! ! ! ! ! ! ! ! !
!
2MA MB MC = (MA MB)+(MA MC) = BA+CA = (AB+
!
AC) = 2AJ với J là trung điểm BC. Do đó điều kiên của M tương đương với
2!
! j ()
MI = AJ = const. Quỹ tích điểm M là đường tròn tâm I,
j j j
MI =2AJ
bán kính AJ. Ngay từ khi chọn I là tâm tỷ cự ta đã biết I tồn tại và duy nhất
vì 1 + 3 - 2 = 2 6= 0 (vế phải không làm thế này được). Vì I [1A, 3B, -2C]
!!!! ()!! !! ! ()! !
=
nên IA + 3IB 2IC = 0 (IA + IB) 2(IB IC) = 0 2IE +2CB
!0
!
IE !
() = CB (E là trung điểm của AB). Như vậy I là đỉnh thứ tư của hình
bình hành BCEI.
Ví dụ 2.8. Cho 4 ABC. Tìm quỹ tích những điểm M sao cho
! ! ! !
(2MA 3MB)(MA +2MB) =~0:
Lời giải. Gọi M [2A, -3B]; G
0
[1A, 2B] thì G và G
0
tồn tại duy nhất và cố
định. Với mọi M trong mặt phẳng:
! !
! ! ! ! ! ! 0 0
. Do
= (1 = 3MG
2MA 3MB = (2 3)MG = MG; MA+ 2MB + 2)MG
đó,
!
! ! ! () ! ! 0 ()  0
= 0
0
=90 ,
(2MA 3MB)(MA + 2MB) = 0 3MG:MG GMG
quỹ tích của M là đường tròn đường kính GG’.
Tải tài liệu tại sividoc.com
Viết đề tài giá sinh viên – ZALO:0973.287.149-TEAMLUANVAN.COM
24
;
G0
! 2 M,GM=
Xác định vị trí dựng điểm G : Vì G [2A, -3B] nên với mọi !
! ! ! ! 0 !
3
3MB 2MA. Chọn M A thì AG = 3AB. Tương tự, AG = AB.
Quỹ tích điểm M là đường tròn đường kính GG0
với G;G0
được dựng theo
các đẳng thức véc tơ:
! ! 0
2
! ; ! .
AG = 3AB AG = AB
3
Các bài toán khác
Bài toán 2.1.
Trên các cạnh tam giác
ABC
lấy các điểm
L, M !
sao cho CL =
CA; CM = CB,0<
a
<1;0<
b
< 1. Giả sử P = AM BL, tính tỷ số
AP
.
a! ! b!
a
 AM
AP 1
HD. = .
AM 1 ab
Bài toán 2.2. Trên các cạnh AB, AC cua tam giác cân BAC lấy các
điểm P, Q sao cho BP = nPA;AQ = nQC. Đường thẳng PQ chia đường
cao AM của tam giác theo tỷ số nào?
Bài toán 2.3. Trên các cạnh AC, BC cua tam giác ABC lấy các điểm L, M sao
! ! ! !
cho CL = aCA;CM = b CB. Giả sử P = AM  BL. Tính các tỷ số AP : AM,
BP : BL.
Bài toán 2.4. Diện tích hình bình hành ABCD bằng 1. Điểm M chia cạnh
BC theo tỷ số 3 : 5. Tính diện tích tứ giác CMPD, trong đó P = AM BD.
Bài toán 2.5. Chứng minh định lý Ce
0
va bằng phương pháp chọn tâm tỷ cự.
HD. Giả sử các đường thẳng AA1 và CC1 cắt nhau tại O và giả sử AC1 : C1B
= p, BA1 : A1C = p. Ta phải chứng minh BB1 đi qua O () CB1 : B1A = 1 : pq.
Đặt 1, p, pq tương ứng tại A, B và C. Ta có C1 [1A;pB]; A1 [pB;pqC]. Do đó
tâm tỷ cự của 1A, pB, pqC là điểm O = CC1 AA1. Mặt khác O thuộc đoạn
thẳng nối B với tâm tỷ cự của A và c. Nếu B1 [1A;pqC] thì AB1 : B1C = pq : 1.
Còn lại phải chứng minh có một điểm trên AC chia AC theo tỷ số AB1 : B1C.
Bài toán 2.6. Trên các cạnh AB, BC, CD và DA của tứ giác lồi ABCD đặt các
điểm K, L, M, N tương ứng và thoa mãn: AK : KB = DM : MC = a;BL : LC = AN
: ND = b . Gọi P = KL  MN. Chứng minh rằng NP : PL = a; KP : PM = b .
Bài toán 2.7. Trong tam giác ABC tìm điểm O sao cho với mọi đường thẳng đi
Tải tài liệu tại sividoc.com
Viết đề tài giá sinh viên – ZALO:0973.287.149-TEAMLUANVAN.COM
25
qua O giao với AB tại K và giao với BC tại L thỏa mãn
p
AK
+ q
CL
= 1;
KB LB
trong đó p;q là các số dương cho trước.
Bài toán 2.8. Trên các cạnh BC, CA, AB của tam giác ABC đặt các
điểm A1;B1;C1 tương ứng và thỏa mãn: các đường thẳng
CC1;AA1;BB1 đồng quy tai O. Chứng minh rằng:
a.
CO
=
CA
1 +
CB
1: OC1 A1B B1A
b. AO:BO:CO AO + BO + CO +2>8:
OA1 OB1 OC1 OA1 OB1 OC1
Bài toán 2.9. Trên các cạnh BC, CA, AB của tam giác ABC đặt các điểm
A1;B1;C1 tương ứng và thỏa mãn:
BA1
=
CB1
=
AC1
. Chứng minh rằng tâm A1C
B1A C1B
tỷ cự của tam giác ABC và tam giác A1B1C1 trùng nhau.
Bài toán 2.10. Trên các cạnh BC, CA, AB của tam giác ABC đặt các điểm
A1;B1;C1 tương ứng sao cho các đường thẳng B1C1;BB1;CC1 cắt đường thẳng
AA1 tương ứng tại các điểm M, P, Q. Chứng minh rằng:
A1M A1P A1Q
a. = + .
MA PA QA
BC1 CB1
b. Nếu P Q thì MC1 : MB1 = : :
AB AC
Bài toán 2.11. Lấy P;P1 trên đường thẳng BA, Q;Q1 trên đường thẳng
AC. Đường thẳng nối A với giao của các đường thẳng PQ và P1Q1
cắt đường thẳng BC tại D. Chứng minh rằng
BP BP1
BD = PA P1A
CD CQ CQ1
QA Q1A
Bài toán 2.12. Cho tam giác ABC, hai điểm M và N thay đổi thỏa !
MN
=
! ! ! mãn
4MA + MB 2MC. Chứng minh rằng đường thẳng MN luôn đi qua một điểm
cố định.
Tải tài liệu tại sividoc.com
Viết đề tài giá sinh viên – ZALO:0973.287.149-TEAMLUANVAN.COM
26
Bài toán 2.13. Cho tư giác ABCD. Tìm quỹ tích điểm M sao cho:
a.!
MA!
MB!
MC !
MD j !
MA!
MB ! j
j
+ + + =
j
+ 2MC ;
!!!!!
b. MA 2MB + 3MC 4MD= AB;
c.
!
! !
=
!
2MA + 3MB MD 2AC.
j
!
Bài toán 2.14.
Cho
4 ABC
. Tìm điểm
M
để
! ! j
đạt giá trị
3MA+ 2MB MC
bé nhất.
Bài toán 2.15. Cho 4 ABC và số k 2 R. Tìm quỹ tích các điểm M thỏa mãn:
! ! ! !
MA + 2MB + kMC = 0 :
Bài toán 2.16. Cho hình bình hành ABCD. Tìm quỹ tích những điểm
M sao cho:
! ! ! !
a.jMA + MBj = jMC + MDj,
! !!!!
b. j2MA MB MCj = jMC + 2MDj.
Bài toán 2.17. Cho hình vuông cạnh a. Tìm quỹ tích những điểm M sao cho:
! !!!2
a. MA:MB + MB:MD = a ,
b. MA
2
+MB
2
+MC
2
= 3MD
2
:
2.2 Kỹ thuật diện tích hóa và tọa độ hóa.
Kỹ thuật diện tích hóa là áp dụng trực tiếp các hệ quả của tính chất
1.4 với mục đích chuyển việc tính tâm tỷ cự trong mặt phẳng về tính
diện tích đại số các tam giác. Áp dụng tọa độ tỷ cự trong mặt phẳng
cho phép giải khá nhiều các bài toán khó bằng tính toán và biến đổi
đại số. Vì khuôn khổ luận văn có hạn nên chúng tôi chỉ xét các ví dụ
điển hình. Vấn đề này sẽ được đề cập sâu hơn vào một dịp khác.
Ví dụ 2.9. Cho DABC. Gọi G, I, O, H, Oa lần lượt là trọng tâm, tâm
đường tròn nội tiếp, tâm đường tròn ngoại tiếp, trực tâm, tâm đường
tròn bàng tiếp trong góc A. Khi đó ta có tọa độ tỷ cự đối với DABC:
a. G(1.A: 1.B:1.C),
Tải tài liệu tại sividoc.com
Viết đề tài giá sinh viên – ZALO:0973.287.149-TEAMLUANVAN.COM
b. I (a.A: b.B: cC),
Tải tài liệu tại sividoc.com
Viết đề tài giá sinh viên – ZALO:0973.287.149-TEAMLUANVAN.COM
27
c. O(sin2A.A: sin2B.B: sin2C.C) hay dưới dạng:
O(a
2
(b
2
+ c
2
a
2
).A: b
2
(c
2
+ a
2
b
2
).B: c
2
(a
2
+ b
2
c
2
).C)
d.O
a
(-S
(OaBC)
.A:S
(OaCA)
.B:S
(OaAB)
.C) (-a.A: b.B: c.C),
e. H(tan A.A: tan B.B: tanC.C) hay dưới dạng:
H(
1
.A:
1
.B:
1
.C).
b
2
+ c
2
a
2
c
2
+ a
2
b
2
a
2
+ b
2
c
2
Chứng minh. a. Vì G ở trong tam giác và S(GBC) =S(GCA) =S(GAB)
nên G [1A,1B,1C]
b. Ta có I nằm trong tam giác và S(IBC) = 1a.r; S(ICA) = 1b.r; S(IAB) = 1c.r nên
2
1 1 1 2 2
I [ ar.A, br.B, cr.C] [aA, bB, cC].
2 2 2
c. Nếu Ab 90
0
thì O và A khác phía nhau đối với BC. Khi đó Sa =S(OBC) =
1
OB.OC.sin BOC =
1
R
2
sin 2A.
2 2
Nếu A > 90
0
thì A và O khác phía nhau đối với BC. Khi đó, Sa = S(OBC) =
1OB:
b
OC:sin(360
0
2A = 1R
2
sin 2A. Tương tự, Sb = 1R
2
sin 2B;Sc =1R
2
sin 2C:
2 2 2 2
Bởi vậy, O [sin 2A:A;sin 2B:B;sin 2C:C]. Vì sin 2A = 2 sin A cos A
b
2
+ c
2
a
2
c
2
+ a
2
b
2
b
2
+ a
2
c
2
= 2a: ; sin 2B = 2b: ; sin 2C = 2c: nên ta
2bc 2ca 2ab
cũng có O [a
2
(b
2
+ c
2
a
2
).A,b
2
(c
2
+ a
2
b
2
):B;c
2
(a
2
+ b
2
c
2
).C].
d. Áp dụng (1.7).
e. Cách 1.
Hình 2.4: Trực tâm H
Tải tài liệu tại sividoc.com
Viết đề tài giá sinh viên – ZALO:0973.287.149-TEAMLUANVAN.COM
28
Khi ABC là tam giác nhọn (trực tâm H ở miền trong tam giác). Ta cần chứng
minh: tan A
!
! ! ~ Trước hết ta vẽ hình bình hành
= 0:
:HA + tan B:HB + tan C:HC HB
0
A1C
AB
0
CA
0
. Trong tam giác BB
0
C ta có HA1 k B
0
C, suy ra: = .
HB A1B
Lại có A1C = AA1 cot C; A1B = AA1 cotB. Do đó, HB
0
= A1C =AA1 cot C =
HB
tan B
HB ! ! A1B AA1 cot B
!
0
tan B ! 0 ! 0
tan C ) tan C
= = HB (Vì HB;HB ngược chiều nhau). Tương tự, HA =
! !
!
! ! ! !
tan A 0 tan A tan B ()
tan C tan C tan C
HA. Từ đó: HA+HB = HA HB=HC tan A.HA +
tan B
!
. +
!
. ~
HB tan C HC = 0:
+ Khi tam giác ABC là tam giác tù. Chứng minh hoàn toàn tương tự với chú ý
rằng
!
! 0
cùng chiều.
HB và HB
Cách 2. (dùng diện tích)
+ Trường hợp 4 ABC không có góc tù (trực tâm H ở trong tam giác):
S
(HBC) =HA
0
=tanHBC =cotACB = 1
S
(ABC) AA0
tanABC tanABC tanB:tanC
tanB
=) Sa =
tanA
S(ABC). Tương tự, Sb =
S
(ABC)
. S
c
=
tanA:tanB:tanC tanA:tanB:tanC
tanC S ~ = Sa
!
+ ! +
!
c = ! +
. Từ (1.2.3):0
tanA:tanB:tanC (ABC)
HA S HB S HC tanA:HA
b
tanB
!
!
:HB + tanC:HC hay H [tan A.A, tan B.B, tan C.C].
+ Khi H ở ngoài tam giác (tam giác ABC tù): Ta vẫn có kết quả
trên. Nhờ định lí hàm tan ta còn có cách viết dạng khác :
H [
1
:A;
1
:B;
1
:C]:
b
2
+ c
2
a
2
c
2
+ a
2
b
2
a
2
+ b
2
c
2
Đương nhiên với ký hiệu tọa độ tỷ cự trong tam giác ABC ta có thể viết: G =
(1:1:1); I = (a: b: c); O = (sin 2A: sin 2B: sin 2C) = (a
2
(b
2
+ c
2
a
2
) : b
2
(c
2
+
a
2
b
2
) : c
2
(a
2
+ b
2
c
2
));
Oa = (-a: b: c), Ob = (a: -b: c), Oc = (a: b: -c);
1
1 1
H = (tan A: tan B: tan C) = ( : : ).
b
2
+ c
2
a
2
c
2
+ a
2
b
2
a
2
+ b
2
c
2
Ví dụ 2.10. Với các ký hiệu như ví dụ trên, hãy chứng minh
S
DOIH
= 1(a b)(b c)(c a):
8r
Lời giải. Theo (1.8) diện tích 4 OIH là
Tải tài liệu tại sividoc.com
Viết đề tài giá sinh viên – ZALO:0973.287.149-TEAMLUANVAN.COM
29
: R : 1:R.
a cos a b cos b c cos g
ABC a b c
a cos b cos g b cos g cos a c cos a cos b
2rp 2p rp
= R2
abc
cos a cos b cos g
1 1 1
4r p 2
R2
cos g cos a
cos b cos g cos a cos b
= (cos b cos a)(cos g cos b )(cos a cosg): Sử dụng công thức cos b
p
cos a =
c
2
+ a
2
b
2
b
2
+ c
2
a
2
(a b)(a + b + c)(a + b c) (a b)(p c)
= =
2ca 2bc 2abc 2rR
thu được:
1
1
(p - a)(p - b)(p - c)(a - b)(b - c)(c - a) = (a - b)(b - c)(c - a).
8pr2
8r
Chú ý rằng rp =
p
.
p(p a)(p b)(p c)
Ví dụ 2.11. Cho tam giác ABC có đường tròn ngoại tiếp (O) và đường tròn
nội tiếp (I). Các đường thẳng AO, BO, CO cắt (O) tương ứng tại D, E, F các
đường thẳng ID, IE, IF cắt BC, CA, AB tương ứng tại X, Y, Z. Chứng minh
rằng AX, BY, CZ đồng quy. Tìm tọa độ tỷ cự của điểm đồng quy.
Lời giải. Nếu tam giác ABC vuông chẳng hạn tại A, khi đó BY, CZ
BC, bài toán hiển nhiên. Bởi vậy ta chỉ cần xét tam giác ABC không
vuông, khi đó SA;SB;SC 6= 0.
Hình 2.5: Tọa độ tỷ cự điểm đồng quy
Tải tài liệu tại sividoc.com
Viết đề tài giá sinh viên – ZALO:0973.287.149-TEAMLUANVAN.COM
30
Phương trình đường thẳng AD: c
2
SCy = b
2
SBz =) z =
c2SC
y. Suy ra tọa
độ b2
SB
của D là nghiệm của hệ
8 z =
b2SB
y
>
c2
S
C
2 yz 2 2xy = 0
< a +b zx + c
>
Cho x = 1 ta có hệ: : z = c
2
S
8 b2 SB y
>
C
2yz 2 2 y = 0
< a +bz + c
2 2 c
2
SC c
>
2SC 2 c
2
SC
= y a : + y: :
+ yc : = 0
b
2
SB SB b
2
SB
) b
2
c
2
(S + S ) b
2
B
() y
2
a
2
c
2
SC + yb
2
c
2
(SB + SC) = 0 () y =
C
= : Tương
a2
c2
S S
c2
b2
c2 C C
tự ta có: z = =) D 1 : : = ( SBSC : b
2
SB : c
2
SD): Suy ra
SB SC SB
đường thẳng ID có phương trình
bc(cSC bSB)x cSC(SB + ca)y + bSB(SC + ab)z = 0
Kết hợp với phương trình BC: x = 0 cho ta
X=BCID= 0:
bS cS
Bằng cách hoán vị vòng quanh a, b, c
B
:
C
SB + ca SC + ab
ta thu được tọa độ của Y, Z:
; Z = SA +
A
bc : SB +
B
ca : 0 :
Y = SC +
C
ab : 0 : SA +
A
bc
cS aS aS bS
Suy ra AX, BY, CZ đồng quy tại H có tọa độ tỷ cự là
H= SA +A
bc
:
SB +B
ca
:
SC +
C
ab :
aS bS cS
Các bài toán khác
Bài toán 2.18. Chứng minh phương trình các đường đặc biệt trong tam giác:
a. Phương trình các cạnh BC, CA, AB của tam giác cơ sở ABC tương ứng
là: x = 0; y = 0; z = 0.
b. Trung trực của cạnh BC: (b
2
c
2
)x + a
2
(y z) =0. Tương tự cho trung
Tải tài liệu tại sividoc.com
Viết đề tài giá sinh viên – ZALO:0973.287.149-TEAMLUANVAN.COM
31
trực hai cạnh kia.
c. Phương trình đường thẳng Euler
(SAB SCA)x + (SBC SAB)y + (SCA SBC)z = 0
d. Phương trình đường thẳng OI nối tâm ngoại tiếp O = (a
2
S
A
: b
2
S
B
: c
2
S )
C
với tâm nội tiếp I = (a:b:c):å(b
2
SBc c
2
SCb)x
= åbc(bSB cSC)x = 0
Vì bSB cSC = ... = -2(b - c)p(p - a) nên phương trình này có thể viết lại là
åbc(b c)p(p a)x= 0 hay å(b c)(p a) x = 0.
a
Bài toán 2.19. Cho 4 ABC không vuông tại C. AD, BE là các đường
cao còn AP, BQ là các phân giác trong ( D, P 2 BC, E, Q 2 CA). Gọi I,
O lần lượt là tâm đường tròn nội tiếp, ngoại tiếp tam giác ABC. chứng
minh rằng: D, E, I thẳng hàng () P, Q, O thẳng hàng.
Bài toán 2.20. Cho DABC. Gọi D, E, F là ảnh của A, B, C qua phép đối xứng
trục SBC;SCA;SAB theo thức tự đó. Gọi (O) và H là đường tròn ngoại tiếp và
là trực tâm của tam giác. Chứng minh rằng D, E, F thẳng hàng () OH = 2R.
Bài toán 2.21. Cho 4 ABC không cân tại C. Gọi M là trung điểm của
AB. AH, CK lần lượt là đường cao, phân giác trong của tam giác ABC.
K, L là trung điểm của CH, CD còn P = CD  MK. Chứng minh rằng P
và L là hai điểm liên hợp đẳng giác trong tam giac ABC.
2.3 Kỹ thuật giao hoán-kết hợp.
Kỹ thuật giao hoán-kết hợp dựa vào tính chất 1.3:
Nếu s(1);s(2); :::;s(n) là một hoán vị của tập hợp f1;2; :::;ng thì [miAi]1 i n
[m
s(i)
A
s(i)
]
1 s(i) n
.
Nếu hai hệ chất điểm fmiAign
i=1;fn jB jgn
j=1 thỏa mãn: Sn
i=1mi + Sn
j=1k j 6= 0
thì
Tải tài liệu tại sividoc.com
Viết đề tài giá sinh viên – ZALO:0973.287.149-TEAMLUANVAN.COM
[m1A1; :::;mnAn;k1B1; :::;knBn] [[m1A1; :::;mnAn];[k1B1; :::;knBn]]:
Tải tài liệu tại sividoc.com
Viết đề tài giá sinh viên – ZALO:0973.287.149-TEAMLUANVAN.COM
32
Khi áp dụng kỹ thuật này ta hay nhóm lại và kết hợp một biểu thức tỷ cự theo
những kiểu khác nhau, giống như cách nhóm kết hợp một biểu thức đại số.
Kỹ thuật này thường được sử dụng trong các bài toán tính tỷ số hoặc chứng
minh tính đồng quy, thẳng hàng. Ta minh họa kỹ thuật này qua các ví dụ sau:
Ví dụ 2.12. Trong tam giác ABC, điểm F nằm trên cạnh BC, chia BC
theo tỷ số 3:1, kể từ B. Các điểm M và P tương ứng nằm trên các
cạnh AB, AC chia chúng theo tỷ số 1:6 kể từ A và C. Giao điểm của
hai đoạn thẳng MP và AF chia các đoạn thẳng này theo tỷ số nào?
Lời giải.
Hình 2.6: Tính tỷ số
Ta sẽ đặt tại B và C các trọng lượng sao cho tâm tỷ cự của hệ là điểm F: Chỉ
cần đặt tại B trọng lượng 1 và tại C trọng lượng 3 (theo quy luật Archimedes),
tức là xét hệ f1B, 3Cg. Ta thấy 1B và xA thỏa mãn 1.BM = x.MA, suy ra, x =
BM : MA = 5. Cuối cùng, xét hệ chất điểm f3C, yAg sao cho P là tâm tỷ cự.
Theo quy tắc Archimedes, ta có: 3CP = yPA, suy ra, y = 3CP:PA = 0,6 =
3
.
5
Tới đây xuất hiện sự kiện mới: tại A có hai trọng lượng khác nhau: 5 và 0,6.
Áp dụng kỹ thuật giao hoán-kết hợp, gọi Z [1B,5A,3C,0,6A] =
[[1B,5A],[3C,0,6A]] = [6M,3,6P]=) Z 2 MP.
Lại xét Z [1B,5A,3C,0,6A] = [[1B,3C],[5,6A]] = [4F,5,6A]=) Z2AF. Vì
Z [5,6A,4F] nên 5,6 AZ = 4FZ hay AZ : FZ = 5:7. Tương tự, 6MZ =
3,6PZ hay MZ:ZP = 3:5. Ta có kết quả cần tìm.
Dùng ký hiệu thu gọn và kỹ thuật giao hoán-kết hợp có thể trình bày lời giải
bài toán ở ví dụ 2.3.1 như sau: Từ giả thiết ta có 4F = 1B + 3C; 6M = 1B + 5A;
Tải tài liệu tại sividoc.com
Viết đề tài giá sinh viên – ZALO:0973.287.149-TEAMLUANVAN.COM
33
1B+5A+3C+0;6A
6P = 3C + 0,6A. Giả sử Z = : Khi đó,
Z =
(1B + 5A) + (3C + 0;6A)
=
6M+3;6P
=)Z2MP:
9;6 9;6
Z =
(1B + 3C) + (5A + 0;6A)
=
4F+5;6A
=)Z2FA:
9;6 9;6
Ta thấy Z = MPFA và theo quy luật Archimedes, 6MZ = 3,6ZP; 4FZ = 5,6ZA.
Từ đó, MZ:ZP = 3:5; AZ:FZ = 5:7.
Ví dụ 2.13. Các điểm M, N, P, Q, R, S theo thứ tự là trung điểm các
cạnh AB, BC, CD, DE, EF, FA của lục giác ABCDEF. Chứng minh
rằng DMPR;DNQS có cùng trọng tâm.
Lời giải. Chọn G [1A, 1B, 1C, 1D, 1E, 1F] thì G duy nhất. Dùng kỹ thuật
giao hoán-kết hợp: G [[1A, 1B], [1C, 1D], [1E, 1F]] [2M, 2P, 2R]. Từ đó, G
là trọng tâm DMPR: Lại áp dụng kỹ thuật giao hoán-kết hợp kiểu khác thì
G [[1B, 1C], [1D, 1E], [1F, 1A]] [2N, 2Q, 2S].
Từ đó, G là trọng tâm DNQS. Vậy hai tam giác MPR và NQS có cùng trọng tâm.
Ví dụ 2.14. Qua điểm P ở trong hình bình hành ABCD kẻ các đường
thẳng song song với các cạnh, chúng cắt các cạnh AB, BC, CD, DA
tại K, L, M, N. Gỉa sử Q là giao các đường trung bình của tứ giác
KLMN còn S là tâm hình bình hành ABCD. Chứng minh rằng Q nằm
trên đoạn thẳng PS. Điểm này chia đoạn PS theo tỷ số nào?
Lời giải. Thoạt tiên ta sẽ đặt các trọng lượng tại 4 đỉnh K, L, M, N sao cho tâm
tỷ cự của hệ 4 điểm đó là Q. Để có được điều đó chỉ cần đặt tại mỗi đỉnh các
trọng lượng bằng 1:Q [1K, 1L, 1M, 1N]. Bây giờ chú ý rằng KLMN là hình bình
hành nên ta có thể thay hệ f1K, 1Lg bằng hệ f1B, 1Pg, tức là Q [1B, 1P, 1D,
1P]. Suy ra Q [1S, 2P] (S là trung điểm của BD). Nhưng khi đó theo quy tắc
Archimedes điểm Q nằm đoạn thẳng PS và chia đôi đoạn thẳng này.
Tải tài liệu tại sividoc.com
Viết đề tài giá sinh viên – ZALO:0973.287.149-TEAMLUANVAN.COM
Ví dụ 2.15. Tứ giác ABCD ngoại tiếp đường tròn, các tiếp điểm là M, N, P, Q.
Biết độ dài các đoạn thẳng tiếp tuyến xuất phát từ A, B, C, D tương ứng bằng
a, b, c, d. Giao điểm của MP và NQ chia mỗi đoạn thẳng đó theo tỷ số nào?
Tải tài liệu tại sividoc.com
Viết đề tài giá sinh viên – ZALO:0973.287.149-TEAMLUANVAN.COM
34
Lời giải.
Đặt các trọng lượng m1;m2 tại các điểm A, B sao cho M [m1A, m2B].
Theo quy tắc Archimedes, am1 = bm2 bởi vậy có thể lấy m1 =
1
;m2 =
1
. Từ
a b
cách làm tương tự, bây giờ nếu tại các đỉnh A, B, C, D ta đặt các trọng lượng
1
;
1
;
1
;
1
thì M [
1
A,
1
B], N [
1
B,
1
C], P [
1
C,
1
D], Q [
1
D,
1
A]. Áp
a b c d a b b c c d d a
dụng kỹ thuật kết hợp:
1 1 1 1 1
1 1 1
Gọi Z [ A, B, C, D] [( + )M, ( + )P] =) Z 2 MP. Tương tự, ta
a b c d a b c d
khảng định được Z thuộc đoạn NQ. Do đó, Z là giao của các đoạn thẳng MP và
NQ. Theo quy tắc Archimedes, ( 1 + 1 )MZ=( 1 +1 )ZP. Từ đó nhân đượcMZ
a c ZP
1 1 1 1 (c + d)ab
b d
= ( + ): ( + ) = .
c d a b (a + b)cd
Tương tự:
NZ = ( 1 +1): (1 +1 ) = (a + d)bc .
ZQ a d b c (b + c)ad
Ví dụ 2.16. Trên các cạnh AC, BC của tam giác ABC ta lấy các điểm M và P
sao cho: AM : MC = 3 : 1; BP : PC = 1 : 2. Các đoạn thẳng AP và BM cắt
nhau ở Q. Biết diện tích tam giác BPQ bằng 1m
2
hãy tính diện tích tam giác
ABC.
Lời giải.
Hình 2.7: Tính diện tích
Dễ thấy, SABC = 3SBAP = 3SBPQ:AP. Bởi vậy ta chỉ cần tính tỷ số AP . Ta đặt các
QP QP
trọng lượng m1;m2;m3 lần lượt vào các đỉnh A, B, C sao cho M [m1A;m3C]
còn P [m3C;m2B]: Theo giả thiết bài toán và quy tắc Archimedes ta sẽ đặt
trọng lượng m1 = 1 vào A, trọng lượng m3 = 3 vào C và trọng lượng m2 = 6 vào
Tải tài liệu tại sividoc.com
Viết đề tài giá sinh viên – ZALO:0973.287.149-TEAMLUANVAN.COM
B. Giả sử Z [1A, 3C, 6B], Khi đó
Tải tài liệu tại sividoc.com
Viết đề tài giá sinh viên – ZALO:0973.287.149-TEAMLUANVAN.COM
35
1A +3C+6B (1A+3C)+6B 4M+6B
Z = = = =)Z2MB
10 10 10
1A + (3C + 6B) 1A+9P
Z = = =)Z2PA.
10 10
Từ đó, Z = MB  PA, tức Z Q. Từ đẳng thức trên theo quy tắc Archimedes:
1.AQ = 9.QP; AP = AQ + QP = 10QP. Nghĩa là
AQ
= 10 =) SABC = 30m2
.
AP
Ví dụ 2.17. Mặt phẳng a cắt các cạnh bên PA, PB, PC của chóp tam giác đều
P.ABC tương ứng theo tỷ số
2
;
3
;
4
. Mặt phẳng a cắt đường cao PM của
hình 5 5 5
chóp theo tỷ số nào?
Lời giải.
Hình 2.8: Hình chóp tam giác đều
Gọi A1;B1;C1 là giao điểm của các cạnh SA, SB, SC với mặt phẳng a. Đặt các
đỉnh A, B, C các trọng lượng bằng 1. VìAP= 5AP tức là: AA1 =3 A1P nên
5 3 2 3 2
A1 = 1A+ P; tức là A1 là tâm tỷ cự của hệ f1A; Pg.
2 2 2
Tương tự, 5 B1 = 1B+2P; 5 C
1 = 1C +1 P: Ngoài ra do chóp S.ABC là chóp
3 3 4 4
tam giác đều nên 3K = 1A + 1B + 1C. Giả sử Q là tâm tỷ cự của hệ 6
chất điểm đang xét, tức là:
1A + 3P+1B+ 2P+1C+ 1P = 65Q:
2 3 4 12
Áp dụng kỹ thuật giao hoán-kết hợp ta có
65Q = (1A+ 3P)+(1B+ 2P)+(1C+ 1P) = 5A1 +5B1 +1 C1:
12
Tải tài liệu tại sividoc.com
Viết đề tài giá sinh viên – ZALO:0973.287.149-TEAMLUANVAN.COM
2 3 4 2 3 4
Tải tài liệu tại sividoc.com
Viết đề tài giá sinh viên – ZALO:0973.287.149-TEAMLUANVAN.COM
36
Suy ra Q 2 a: Áp dụng kỹ thuật giao hoán-kết hợp theo kiểu khác:
65
Q= (1A + 1B + 1C) + (
3
P +
2
P) +
1
P) + (1C +
1
P) = 3M +
29
p;
12 2 3 4 4 12
ta thu được Q 2 PM: Vậy Q = PM a.
Các bài toán khác
Bài toán 2.22. Trong góc PAQd vẽ nội tiếp một đường tròn tiếp xúc với hai cạnh
của góc tại P, Q. Đường thẳng BC, B 2 (AP), C 2 (AQ) tiếp xúc với đường tròn tại
T. Gọi M = BQ  CP. Chứng minh rằng ba điểm A, T, M thẳng hàng.
Bài toán 2.23. Đáy hình chóp F.ABCD là hình bình hành. Mặt phẳng a
cắt các cạnh bên, BF, CF, DF tương ứng tại A1;B1;C1;D1 sao cho
AA
1 = 2;
BB
1 = 5;
CC
1 = 10:
A1F B1F C1F
DD1
Tính tỷ số x = :
! ! ! !
Bài toán 2.24. Trên cạnh4 ABC lấy các điểm L, M sao cho: CL = aCA;CM = b CB
(0 < a < 1; 0 < b < 1). Giả sử P là giao của AM và BL. Tính tỷ số
AP
:
AM
2.4 Kỹ thuật quán tính.
Kỹ thuật mô men quán tính dựa vào các tính chất 1.5 và 1.6 nói về các
công thức Lagrang và Jacobi. Với kỹ thuật này ta thu được các công thức
về khoảng cách. Đặc biệt cho ta phép chứng minh mới về công thức Euler.
Ví dụ 2.18. Biết các bán kính đường tròn ngoại tiếp, nội tiếp tam giác
ABC là R;r. Tính khoảng cách d giữa các tâm đường tròn này.
Lời giải. Giả sử Z là tâm nội tiếp và O là tâm ngoại tiếp, a, b, c là độ dài các
cạnh tam giác ABC. Dễ thấy điểm Z là tâm tỷ cự của hệ chất điểm aA, bB, cC.
Theo công thức Lagrang, (a + b + c)jOZj
2
= JO JZ. Hơn nữa, theo định nghĩa
mô men quán tính thì JO = ajOAj
2
+ bjOBj
2
+ cjOCj
2
= (a + b + c)
2
R
2
:
1
(abjABj
2
+bcjBCj
2
+cajCAj
2
) =
a + b + c
Tải tài liệu tại sividoc.com
Viết đề tài giá sinh viên – ZALO:0973.287.149-TEAMLUANVAN.COM
Cuối cùng, theo công thức Jacobi, JZ =
Tải tài liệu tại sividoc.com
Viết đề tài giá sinh viên – ZALO:0973.287.149-TEAMLUANVAN.COM
37
abc
2
+ bca
2
+ cab
2
= abc. Bởi vậy, d
2
= jOZj
2
=
JO JZ
= R
2 abc
. Từ
a + b + c a + b + c a + b + c
các công thức quen biết S = abc =1(a + b + c)r ta rút ra: abc = 2Rr và
4R 2 a + b + c
bởi vậy, d
2
= R(R 2r), ta thu được công thức Euler.
Ví dụ 2.19. (Bài toán Apolonius) Trong mặt phẳng cho hai điểm A, B. l
6= 1 là một số cho trước. Tìm quỹ tích các điểm M sao cho
MA
= l .
MB
Lời giải. Điều kiện
MA
= l tương đương với hệ thức l 2
MB2
MA
2
= 0. Ta
MB
nhận thấy vế trái là mô men quán tính của hệ hai chất điểm fl
2
B;( 1)Ag đối
với điểm M. Nghĩa là bài toán chuyển về tìm quỹ tích điểm M sao cho JM = 0.
Ký hiệu Z là tâm tỷ cự của hệ chất điểm fl
2
B;( 1)Ag, ta có theo công thức
Lagrang và Jacobi:
JM = JZ + (l
2
1)ZM
2
= l2
AB
2
+ (l
2
1)ZM
2
:
l
2
1
Bởi vậy điều kiện JM = 0 được thực hiện khi và chỉ khi
l
ZM =
jl 2
l
Từ đó rõ ràng quỹ tích cần tìm là đường tròn tâm Z, bán kính jl 2
1jAB:
Nhận xét 2.1. Có thể tổng quát hóa bài toán như sau: Ký hiệu s là hệ chất điểm
m1A1;m2A2; :::;mnAn: Mô men quán tính của hệ này đối với điểm M
được ký hiệu là JM(s): Ta giải thích dạng tập hợp F tất cả các điểm M
trên mặt phẳng sao cho JM(s) = h với h là hằng số cho trước.
Lời giải.
Trường hợp 1. Nếu m = m1 + m2 + ::: + mn 6= 0 thì gọi Z là tâm tỷ cự của hệ
s ta có theo công thức Lagrang,
ZM
2
=
1
(k JZ(s)):
m
1jAB:
Tải tài liệu tại sividoc.com
Viết đề tài giá sinh viên – ZALO:0973.287.149-TEAMLUANVAN.COM
Từ đó ta thấy F là đường tròn (với JZ(s) < h), hoặc một điểm (với
JZ(s) = h) hoặc rỗng (với JZ(s) > h).
Trường hợp 2. Nếu m = m1 + m2 + ::: + mn = 0 nhưng ít nhất có một mi 6=
Tải tài liệu tại sividoc.com
Viết đề tài giá sinh viên – ZALO:0973.287.149-TEAMLUANVAN.COM
38
0, chẳng hạn mn 6= 0. Ký hiệu C là tâm tỷ cự của hệ s
0
gồm các chất
điểm m1A1;m2A2; :::;mn 1An 1: Với mọi M được chọn ta có:
J (s0
) = J (s0
) + ( m )MC2
; J (s) = J (s0
) + ( m )MA2
.
M C n M M n n
mn:MA
2
n + ( mn)MC
2
= h JC(s
0
) = const:
Khi C 6= An hình F là một đường thẳng vuông góc với CAn. Nếu C An
hình F là toàn mặt phẳng hoặc rỗng.
Trường hợp 3. Nếu tất cả các mi đều bằng không, hình F sẽ là toàn
mặt phẳng (khi h = 0) hoặc tập hợp rỗng.
Ví dụ 2.20. Gọi các đường tròn bàng tiếp của DA1A2A3 là g1;g2;g3:
Giả sử g1 tiếp xúc với cạnh A2A3 tại B1, g2 tiếp xúc với cạnh A3A1 tại
B2, g3 tiếp xúc với cạnh A2A1 tại B3. Qua B1;B2;B3 kẻ các đường
thẳng d1;d2;d3 tương ứng vuông góc với các cạnh A2A3;A1A3;A2A1.
Chứng minh rằng các đường thẳng này đồng quy tại một điểm.
Lời giải. Xét ba hệ chất điểm
s1 : 0:A1 1:A2 ( 1):A3
s2 : ( 1):A1 0:A2 1:A3
s3 : 1:A1 ( 1):A2 0:A3
Đường thẳng d1 được đặc trưng bởi điều kiện (xem nhận xét sau bài toán trên)
JM(s1) = JB1 (s1) = B1A
2
2 B1A
2
3(= h1)
Để tính h1 ta có thể ký hiệu T2;T3 là tiếp điểm của g1 với các đường
thẳng A1A2 và A1A3 và đặt
A2A3 = a;A3A1 = b;A1A2 = c;B1A2 = x;B1A3 = y;A1T2 = z. Khi đó, x +
y = a, z - x = c, z - y = b
Nhận được x - y = b - c và bởi vậy,
h1 = x
2
y
2
= (x + y)(x y) = a(b c)
Tải tài liệu tại sividoc.com
Viết đề tài giá sinh viên – ZALO:0973.287.149-TEAMLUANVAN.COM
39
Như vậy điều kiện JM(s1) = h1; đặc trưng cho đường thẳng d1 có dạng
1:MA2
2
+ ( 1)MA3
2
= a(b c) (2.1)
Tương tự các đường thẳng d1;d2 được đặc trưng bởi
1:MA3
2
+( 1)MA1
2
= b(c a) (2.2)
1:MA1
2
+( 1)MA2
2
= c(a b) (2.3)
Bây giờ gọi M = d1  d2 thì M thoa mãn các điều kiện (2.1, 2.2). Từ
đây cộng các hệ thức này và nhận kết quả với (-1) thì nhận được
(2.3). Điều đó nghĩa là M thỏa mãn cả (2.3), tức M thuộc d3.
Ví dụ 2.21. (Công thức tính khoảng cách giữa hai điểm)
Giả sử P [p1A1; p2A2; :::; pnAn]; Q [q1A1;q2A2; :::;qnAn]: Khi đó,
khoảng cách của hai điểm P, Q được tính theo công thức
PQ
2
=å (pi qi)(pj qj) j AiAj j2
: (2.4)
1 i< j n
Chứng minh. Ta xét với n = 3 (trường hợp tổng quát suy luận tương
tự). Theo điều kiện,
1:P = p1A1 + p2A2 + p3A3;1:Q = q1A1 + q2A2 + q3A3
p1 + p2 + p3 = q1 + q2 + q3 = 1 Xét 3 hệ chất điểm
s1 :
s2 :
s3 :
(p1 q1)A1; (p2
q1A1;
p1A1;
q2)A2; (p3
q2A2;
p2A2;
q3)A3;
q3A3;
p3A3;
1Q:
1Q:
Các mô men quán tính của các hệ này đối với điểm P được ký hiệu là
J
(
P
1)
;J
(
P
2)
;J
(
P
3)
còn đối với Q được ký hiệu là J
(
Q
1)
;J
(
Q
2)
;J
(
Q
3)
. Tâm tỷ cự của các
hệ s1;s2;s3 tương ứng là các điểm P, Q và trung điểm Z của đoạn PQ. Bởi vậy ta có:
J
(
P
3)
= J
(
Z
3)
+ 2 j PZ j
2
;J
(
Q
3)
= J
(
Z
3)
+ 2 j QZ j
2
:
Tải tài liệu tại sividoc.com
Viết đề tài giá sinh viên – ZALO:0973.287.149-TEAMLUANVAN.COM
40
và do đó, J
(
P
3)
= J
(
Q
3)
= p1 j QA1 j
2
+p2 j QA2 j
2
+p3 j QA3 j
2
. Ngoài
ra, theo định lý Lagrang,
J
(2)
= J
(2)
+ 1: j PQ j
2
= q j QA j
2
+q j QA j
2
+q j QA j
2
+ j PQ j
2
:
P Q 1 1 2 2 3 3
JP
(1)
= å (pi
3
qi)(pj qj) j AiAj j
2
+ å (pi qi) j AiQ j
2
:
1 i<j 3 i=1
Từ hê thức hiển nhiên: JP
(3)
= JP
(1)
+ JP
(2)
ta có JQ
(3)
= JZ
(3)
+ 2 j QZ j
2
=
= å (pi qi)(pj
3
qj) j AiAj j
2
+ å (pi qi) j AiQ j
2
+JQ
(2)
+ 1: j PQ j
2
1 i<j 3 i=1
= q1 j QA1 j
2
+q2 j QA2 j
2
vế ta có
PQ
2
=
+q3 j QA3 j
2
+ j PQ j
2
. Sau khi giản ước và chuyển
å (pi qi)(pj qj) j AiAj j2
:
1 i<j n
Các bài toán khác
Bài toán 2.25. Cho a, b, c là độ dài ba cạnh của tam giác ABC. Tính
độ dài đường phân giác.
Bài toán 2.26. Cạnh của tứ diện đều bằng a. Tính bán kính mặt cầu ngoại tiếp.
Bài toán 2.27. Cho O là tâm đường tròn nội tiếp tam giác ABC, a, b, c
là độ dài các cạnh của tam giác. Tính tổng: s = aOA
2
+ bOB
2
+ cOC
2
Chương hai trình bày các kỹ thuật biến đổi tâm tỷ cự với các ví dụ minh họa
phong phú cùng các bài toán bổ sung nhằm làm rõ ý tưởng của phương pháp áp
dụng tâm tỷ cự vào các bài toán chúng minh, tính tỷ số, tìm quỹ tích. Rõ ràng áp
dụng khái niệm tâm tỷ cự theo các kỹ thuật trên đã thu được các kết quả tốt. Tiếp
sang chương ba các kỹ thuật đó được phối hợp khá thành công trong các ví dụ
khó hơn: chứng minh các định lý cổ điển và giải một số đề thi Olympic.
Tải tài liệu tại sividoc.com
Viết đề tài giá sinh viên – ZALO:0973.287.149-TEAMLUANVAN.COM
41
Chương 3
Các vấn đề liên quan
Với khuôn khổ của một luận văn, trong chương này tác giả chỉ trình
bày hai phần có áp dụng các kỹ thuật biến đổi tâm tỷ cự. Đó là chứng
minh lại một số định lý và giải một số bài toán thi Olympic quốc tế
bằng cách áp dụng tâm tỷ cự. Nội dung chương này chủ yếu được
tham khảo trong [6] với cách trình bày chi tiết và có chọn lọc.
3.1 Chứng minh một số định lý nổi tiếng
Ta sẽ áp dụng khái niệm tâm tỷ cự vào việc chứng minh các kết quả sau:
định lý Routh, định lý Mene
0
laus, định lý Ce
0
va, công thức Euler, đường
thẳng Euler, đường thẳng Nagel, công thức Sterwartz, một số hệ thức liên
quan đến phương tích, định lý Ptolemy, bất đẳng thức Gerretsen I, II,...
Ví dụ 3.1. (Định lý Routh) Cho tam giác ABC, các điểm M 2 BC, N 2
CA, P 2 AB và chia các đoạn BC, CA, AB theo các tỷ số m : 1; n : 1; p
: 1. Gọi S1 là diện tích tam giác tạo bởi giao điểm của AM, BN, CP; S2
= S(MNP): Khi đó hãy chứng minh:
(mnp 1)2
a.S1 =
(mn + m + 1)(np + n + 1)(pm + p + 1)
:S(ABC);
mnp + 1
b.S
2
=
(m + 1)(n + 1)(p + 1)
:S
(ABC)
:
Chứng minh. Ta có D=(0 : 1 : m), E=(n : 0 : 1), F=(1 : p : 0). Gọi I = BN CP,
các điểm I, K được xác định tương tự. Ta tìm được I=(n : np : 1), J=(1 : p : pm),
Tải tài liệu tại sividoc.com
Viết đề tài giá sinh viên – ZALO:0973.287.149-TEAMLUANVAN.COM
42
K=(mn : 1 : m). Suy ra
n np 1
1 p pm
mn 1 m
S
1 =
:S
(ABC)
;
(mn + m + 1)(np + n + 1)(pm + p + 1)
= (mnp 1)
2 :S
(ABC)
:
(mn + m + 1)(np + n + 1)(pm + p + 1)
0 1 m
n 0 1
1 p 0
S2 =
:S
(ABC)
;
(m + 1)(n + 1)(p + 1)
=
(mnp + 1):S(ABC)
:
(m + 1)(n + 1)(p + 1)
Các đẳng thức được chứng minh.
Định lý Routh chính là kết quả tổng quát của hai định lý nổi tiếng:
Định lý Ce’va: Nếu mnp = 1 thì S1 = 0; AM; BN; CP đồng quy;
Định lý Mene’laus: Nếu mnp = 1 thì S2 = 0; M; N; P thẳng hàng.
Nhận xét 3.1. Hai định lý này có thể chứng minh trực tiếp bằng
phương pháp chọn tâm tỉ cự. Rõ ràng cách chứng ba định lý trên
mang đặc trưng của tâm tỷ cự.
Tọa độ tỷ cự có thể dùng để xem xét tính thẳng hàng của các điểm. Giả sử ta
[PBC]
có hai điểm chuẩn hóa P = (x1 : y1 : z1);Q = (x2 : y2 : z2) theo nghĩa x1 = [ABC],
tương tự cho các y1;z1;x2;y2;z2. Gọi P1;P2;P3 là hình chiếu của P lên các đường
thẳng BC, CA, AB tương ứng và tương tự đối với Q. Vì [PBC] = 1:PP1:a nên ta
2
có: PP1 =2[PBC] =2[ABC]:x1. Tương tự, QQ1 =2[ABC] :x2.
a a a
Xét điểm R(x3 : y3 : z3) trên đường thẳng BC sao cho
PR
= k với k 2 R nào đó
PQ
Tải tài liệu tại sividoc.com
Viết đề tài giá sinh viên – ZALO:0973.287.149-TEAMLUANVAN.COM
và R1 là hình chiếu của nó trên BC. Theo tính chất hình thang PP1Q1Q có thể
Tải tài liệu tại sividoc.com
Viết đề tài giá sinh viên – ZALO:0973.287.149-TEAMLUANVAN.COM
43
Hình 3.1: P;Q;R thẳng hàng
xem RR1 = (1 k):PP1 + k:QQ1, tức là x3 = (1 k):x1 + k:x2. Tương tự đối với
y3 và z3 ta tìm được
R=(1 k)P + k:Q: (3.1)
Đây là phương trình ba điểm thẳng hàng sau khi các điểm đã được chuẩn hóa.
Hơn nữa, giá trị của k mà R = (1 k):P +k:Q tương ứng với tỷ số k =
PR
. Nhắc
PQ
lại là P = (x1 : y1 : z1);Q = (x2 : y2 : z2);R = (x3 : y3 : z3) thẳng hàng khi và chỉ
khi
x
1
x
2
x
3 = 0.
y1
y2
y3
z1
z2
z3
+ Đường thẳng Euler:
Các điểm H, G, O nằm trên một đường thẳng theo thứ tự đó và OH = 3OG.
Diện tích tam giác OBC là 1R
2
sin 2a = R
2
sin a:cos a =1Ra cos a, ta thu được
2
2
tọa độ chuẩn hóa của O là
O = R (a cos a : : ) = 1 (a
2
(b
2
+ c
2
a
2
) : : )
2rp 4p
2
r
2
G và H đã có chuẩn hóa như sau:
G = 1(1:1:1);H= R (a cos b cos g : b cos g cos a : c cos a cos b ). Ngoài ra với
3 pr
chủ ý lập quan hệ giữa các điểm G, O, H ta viết:
1 = Ra:cos b :cos g +Ra :cos a
r p r p
Tải tài liệu tại sividoc.com
Viết đề tài giá sinh viên – ZALO:0973.287.149-TEAMLUANVAN.COM
Đẳng thức này tương đương đẳng thức hiển nhiên
abc
= 1 với chú ý
rằng 4prR
Tải tài liệu tại sividoc.com
Viết đề tài giá sinh viên – ZALO:0973.287.149-TEAMLUANVAN.COM
44
bc
cos a + cos b cos g = cos a + cos(b + g) + sin b sin g = 4R2 . Cuối
cùng, ta thu được phương trình đường thẳng Euler là 3G = H + 2O.
+ Đường thẳng Nagel:
Ký hiệu các tiếp điểm là D 2 BC;E 2 AC;F 2 AB. Sử dụng định lý Ce
0
va chứng
minh được các đường thẳng AD;BE;CF đồng quy tại một điểm. Đó là điểm Nagel
N (giao của các đường thẳng nối các đỉnh tam giác với các tiếp điểm của đường
tròn bàng tiếp). Tâm Spieker có tọa độ chuẩn hóa S =
1
(b+c : c+a :
4p
a + b).
Điểm Nagel N, điểm Spieker S, trọng tâm G và tâm nội tiếp I thẳng hàng,
đường thẳng đó có tên là đường thẳng Nagel, S và G tương ứng là điểm chia đôi
và điểm chia ba của đoạn NI. Vì BD = (p c)(p b), tương tự với E và F nên
1
DC
ta được N = (p a : p b : p c). Dễ kiểm tra được:
p
p a + a
2(N+I) = 2 2p : : = 4p : : = S
2
1 b c
và 3I= 3p + 3p : : = 3 : : = G
3N +
1 2 p a a 1
Tức là ta có phương trình đường Nagel:
(
N + I = 2S
.
N+2I = 3G
Định lý 3.1. (Hệ thức Sterwartz) Trong tam giác ABC với mọi M thuộc
cạnh BC ta có
MB:AC
2
+ MC:AB
2
= BC:MA
2
+ MB:MC:MA:
Chứng minh. Khi M thuộc cạnh BC thì dễ thấy M [MB:B; MC:C], áp dụng
hệ thức Jacobi với hệ 2 điểm ta có:
: :BC
2
MB MC
MB:AC
2
MC:AB
2
= (MB MC)AM
2
MB MC
() MBAC
2
MC:AB
2
= CB:AM
2
+ MB:MC:BC:
!
Chọn véc tơ đơn vị chỉ phương cùng phương với BC ta suy ra:
Tải tài liệu tại sividoc.com
Viết đề tài giá sinh viên – ZALO:0973.287.149-TEAMLUANVAN.COM
45
MB:AC
2
+ MC:AB
2
= BC:MA
2
+ MB:MC:BC
Từ đồng nhất thức Jacobi đối với hệ 3 điểm với I [a:A;b :B;g:C], P
là điểm tùy ý:
a:PA2 + b :PB2 + g:PC2 = (a + b + g)PI2 +
b g:a2 + ga:b2 + ab :c2
:
a + b + g
Trừ vế với vế cho (a + b + g)R
2
, theo tính chất của phương tích ta có:
a:PA=(O) +b :PB=(O) +g:PC=(O) = (a +b +g)PI=(O) +
b g:a2 + ga:b2 + ab :c2
a
+ b + g
() P
I=(O)
=
aP
A=(O)
+ b P
B=(O)
+ gP
C=(O) b g:a
2
+ ga:b
2
+ ab :c
2
:
(a + b + g) (a + b + g)
2
(3.2)
Đây là công thức tính phương tích của tâm tỷ cự I [a:A;b :B;g:C] đối với
đường tròn tùy ý tâm O. Các trường hợp đặc biệt cho ta các hệ quả sau:
Hệ quả 3.1. Cho tam giác ABC và đường tròn ngoại tiếp (O), phương
tích của điểm I [a:A;b :B;g:C] đối với (O) được tính theo công thức:
b g:a
2
+ ga:b
2
+ ab :c
2
(3.3)
P I=(O) = :
(a + b + g)
2
Hệ quả 3.2. Điểm I [a:A;b :B;g:C] thuộc đường tròn ngoại tiếp (O) khi
và chỉ khi
b g:a
2
+ ga:b
2
+ ab :c
2
= 0:
Hệ quả 3.3. (Hệ thức Euler) Tam giác ABC nội tiếp đường tròn tâm O,
bán kính R, ngoại tiếp đường tròn tâm I, bán kính r. Khi đó OI
2
= R
2
2Rr.
Chứng minh. Vì I [aA;bB;cC] nên áp dụng (3.3)
P
I=(O)
= bc:a
2
+ ca:b
2
+ ab:c
2
= abc :
(a + b + c)
2
(a + b + c)
Vế trái bằng OI
2
R
2
. Vế phải bằng 2Rr với lưu ý: SABC =abc = p:r. Vậy
4R
đẳng thức trên tương đương với OI
2
= R
2
2Rr:
Tải tài liệu tại sividoc.com
Viết đề tài giá sinh viên – ZALO:0973.287.149-TEAMLUANVAN.COM
Tương tự đối với các đường tròn bàng tiếp (Ia;ra);(Ib;rb);(Ic;rc)
Tải tài liệu tại sividoc.com
Viết đề tài giá sinh viên – ZALO:0973.287.149-TEAMLUANVAN.COM
46
Hệ quả 3.4.
OI
2
a = R
2
+ 2Rra; OI
2
b = R
2
+ 2Rrb; OI
2
c = R
2
+ 2Rrc:
Bài toán 3.1. Cho tam giác ABC. Các điểm O, G và H lần lượt là tâm
ngoại tiếp, trọng tâm và trực tâm tam giác. Khi đó ta có:
a. OG
2
= R
2 a
2
+ b
2
+ c
2
;
9
b. PH=(O) = 8R
2
cos A cos B cos C:
c. cos
2
A + cos
2
B + cos
2
C + 2 cos A cos B cos C = 1:
Hệ quả 3.5. (Định lý Ptolemy) Cho tam giác ABC nội tiếp đường tròn
(O) và D là điểm nằm trên đường tròn. Khi đó trong ba số sau có một
số bằng tổng hai số kia aDA;bDB;cDC . Cụ thể:
Nếu D thuộc cung nhỏ BC thì aDA = bDB + cDC
Nếu D thuộc cung nhỏ CA thì bDB = cDC + aDA
Nếu D thuộc cung nhỏ AB thì cDC = aDA + bDB:
Chứng minh. Khi D thuộc cung nhỏ BC dễ thấy
D [
a
:A;
b
:B;
c
:C]:
DA DB DC
Áp dụng (3.3) ta có điều phải chứng minh. Khi D ở các vị trí còn lại
làm tương tự.
Hệ quả 3.6. Cho tam giác ABC với đường tròn ngoại tiếp (O) và
đường tròn nội tiếp (I). Tâm tỷ cự P [a:A;b :B;g:C]. Khi đó
a(p a)
2
+ b (p b)
2
+ g(p c)
2
(3.4)
P P
P=(O)
= :
P=(I)
a + b + g
Chứng minh. Gọi tiếp điểm của đường tròn (I) với các cạnh BC, CA, AB là
D,E,F tương ứng. Theo công thức phương tích và tiếp tuyến ta có: PA=(O) =
AE
2
= AF
2
= (p a)
2
; P = BD
2
= BF
2
= (p b)
2
; P = CE
2
= CD
2
=
B=(O) C=(O)
Tải tài liệu tại sividoc.com
Viết đề tài giá sinh viên – ZALO:0973.287.149-TEAMLUANVAN.COM
(p c)2
: Sau khi áp dụng công thức 3.2 ta thu được điều phải chứng minh.
Tải tài liệu tại sividoc.com
Viết đề tài giá sinh viên – ZALO:0973.287.149-TEAMLUANVAN.COM
47
Bài toán 3.2. Tam giác ABC ngoại tiếp đường tròn (I) có G là trọng
tâm tam giác. Khi đó:
p2
4r(4R + r)
(3.5)
P G=(I) = :
9
Chứng minh. Áp dụng công thức tính phương tích của G [1A;1B;1C]
đối với đường tròn (I) ta có
(p a)
2
+ (p b)
2
+ (p c)
2
a
2
+ b
2
+ c
2
P
G=(I)
=
+ :
3 9
Bằng phép biến đổi thích hợp ta suy ra điều phải chứng minh.
Từ (3.5), GI
2
r
2
p
2
4r(4R + r)
() GI
2
=
p
2
16Rr + 5r
2
= . Suy ra
9 9
Hệ quả 3.7. (Bất đẳng thức Gerretsen thứ
nhất) p
2
16Rr 5r
2
:
Bài toán 3.3. Cho tam giác ABC nội tiếp đường tròn tâm (O) bán kính R,
ngoai tiếp đường tròn tâm I, bán kính r. Gọi H là trực tâm tam giác. Khi đó:
P = 4R
2
+ 4Rr + 2r
2
p
2
(3.6)
H=(O)
p2 16Rr + 5r
2
Chứng minh. Ta đã có các công thức GI
2
= ; OI
2
= R
2
2Rr
9
9R2
(a2
+ b2
+ c2
)
và OG 2 . Từ hệ thức Euler ! ! ~ , áp dụng hệ
= = 0
3HG 2HO
9
2IO
2
= IH
2
6OG
2
, bằng các tính toán thích hợp
thức Jacobi ta suy ra: 3IG
2
ta suy ra điều phải chứng minh.
Từ (3.6) dễ chỉ ra 0 IH
2
= 4R
2
+ 4Rr + 3r
2
p
2
: Từ đó thu được
Hệ quả 3.8. (Bất đẳng thức Gerretsen thứ hai)
p
2
4R
2
+ 4Rr + 3r
2
:
Ví dụ 3.2. Cho tam giác ABC và (O9) là đường tròn Euler hay đường tròn chín
Tải tài liệu tại sividoc.com
Viết đề tài giá sinh viên – ZALO:0973.287.149-TEAMLUANVAN.COM
48
điểm của tam giác. Tâm tỷ cự P [a:A;b :B;g:C]. Khi đó
abc cos A + b ca cos B + gab cos C b g:a
2
+ ga:b
2
+ ab :c
2
P
P=(O9)
=
2(a + b + g) (a + b + g)2
(3.7)
Chứng minh. Ký hiệu A
0
;B
0
;C
0
là trung điểm các cạnh BC, CA, AB và D, E, F
là chân các đường cao hạ từ A, B, C tương ứng. Ta chú ý rằng đường tròn
Euler (O9) đi qua các điểm A
0
;B
0
;C
0
và D,E,F. Theo công thức phương tích:
P
A=(O9)
=
: =
bc cos A. Tương tự, PB=(O9) = ca cos B;P
C=(O9)
= ab cos C
.
AO9 AB
0
2 2 2
Áp dụng 3.2 ta được
P
P=(O9)
= abc cos A + b ca cos B + gab cos C b g:a
2
+ ga:b
2
+ ab :c
2
:
(a + b + g)
2
2(a + b + g)
Đó là điều phải chứng minh.
Ta đã có kết quả sau: Cho tam giác ABC. Khi đó nếu PA=(GBC) =
PB=(GCA) = PC=(GAB) thì G là trọng tâm tam giác . Phần đảo lại cũng đúng
nhưng chứng minh khó hơn (T8/399/THTT): Nếu điểm G không nằm trên
cạnh, không nằm trên đường tròn ngoại tiếp và thỏa mãn hệ thức trên thì G
là trọng tâm tam giác. Ở đây ta sẽ chứng minh một kết quả tổng quát hơn:
Ví dụ 3.3. Cho tam giác ABC và điểm P = (a : b : g) bất kỳ không nằm trên
cạnh, không nằm trên đường tròn nội tiếp tam giác. Chứng minh rằng
aP
A=(PBC)
= aP
B=(PCA)
= aP
C=(PAB)
= (a + b + g)P
A=(PBC)
: (3.8)
Chứng minh. Vì A,B,C,P là các điểm tùy ý trong mặt phẳng nên ta có thể tìm
PA + ~ PB + PC = 0, với
tọa độ tỷ cự của A đối với tam giac cơ sở PBC. Từ a
!
b! !g
mọi M ta có:
)MP
+ + = ( +
b
+
!
aMA
!
bMB
!
gMC a g !
AB + AC=(
a
+
b
+ )AP hay ( +
b
+ )AP + AB +
Cho M A được b
!
g
!
g
!
a g
!
b
!
~
. Như vậy,
A [ ( +
b
+
g
)P :
b
B :
g
C]
AC=0
g
!
a . Theo hệ thức (3.3) áp dụng
vào điểm A và (PBC):
b ga
2
b (a + b + g)PB
2
(a + b + g)PC
2
P
A=(PBC)
= =
Tải tài liệu tại sividoc.com
Viết đề tài giá sinh viên – ZALO:0973.287.149-TEAMLUANVAN.COM
[ (a + b + g) + b + g]
2
Tải tài liệu tại sividoc.com
Viết đề tài giá sinh viên – ZALO:0973.287.149-TEAMLUANVAN.COM
49
=
b ga
2
(a + b + g)(b PB
2
+ gPC
2
)
:
a
2
Từ hệ thức Leibniz cho moi M ta có: aMA
2
+ b MB
2
+ gMC
2
= (a + b +
g)MP
2
+aPA
2
+b PB
2
+gPC
2
hay aMA
2
+b MB
2
+gMC
2
= (a +b +g)MP
2
=
(a + b + g)b ga
2
+ gab
2
+ ab c
2
.
a + b + g
Nếu cho M A ta được:
b c
2
+ gb
2
= (a + b + g)AP
2
b ga
2
+ gab
2
+ ab c
2
+ :
a + b + g
Do đó PA
2
=
(b + g)(b c
2
+ gb
2
) b ga
2
. Tương tự,
(a + b + g)
2
PB
2
=
(g + a)(gb
2
+ ac
2
) gab
2
; PC
2
=
(a + b )(ab
2
+ b c
2
) ab c
2
. Lại
(a + b + g)
2
(a + b + g)
2
theo công thức
b ga
2
+ gab
2
+ ab c
2
b PB
2
+ gPC
2
= aPA
2
+ b PB
2
+ gPC
2
aPA
2
= a + b + g
a (b + g)(b c
2
+ gb
2
) b ga
2
= (a + b + g)b ga
2
+ a(b ga
2
+ gab
2
+ ab c
2
):
(a + b + g)
2
(a + b + g)
2
Thay vào đẳng thức
trên ta được
b ga
2 (a + b + g)b ga
2
+ a(b ga
2
+ gab
2
+ ab c
2
)
a + b + g
P
A=(PBC)
=
=
a
2
b ga
2
+ gab
2
+ ab c
2
: Mặt khác cũng từ bổ đề ta có:
a(a + b + g)
P P=(ABC) =
b ga
2
+ gab
2
+ ab c
2
. Từ hai công thức trên, PA=(PBC) = (a +
(a + b + g)2
b+ g)P
P=(ABC)
.
Làm tương tự với các đỉnh B;C ta có điều phải chứng minh.
Các trường hợp đặc biệt:
Bài toán 3.4. Cho DABC và điểm P bất kỳ không thuộc cạnh, không
thuộc đường tròn ngoại tiếp tam giác. Nếu PA=(PBC) = PB=(PCA) =
PC=(PAB) thì P là trọng tâm DABC.
Tải tài liệu tại sividoc.com
Viết đề tài giá sinh viên – ZALO:0973.287.149-TEAMLUANVAN.COM
50
Bài toán 3.5. Cho DABC và điểm P bất kỳ không thuộc cạnh, không
thuộc đường tròn ngoại tiếp tam giác. Nếu a:PA=(PBC) = b :PB=(PCA)
= g:PC=(PAB) thì P [aA;b B;gC] hay P = (a : b : g).
Bài toán 3.6. Cho DABC và điểm P bất kỳ không thuộc cạnh, không
thuộc đường tròn ngoại tiếp tam giác. Nếu a:PA=(PBC) = b:PB=(PCA)
= c:PC=(PAB) thì P [aA : bB : cC] hay P là tâm đường tròn nội tiếp.
Các bài toán khác
Bài toán 3.7. (Công thức Euler) Cho DABC nội tiếp trong đường tròn tâm O,
bán kính R. M là một điểm bất kỳ trong mặt phẳng tam giác. Gọi A1; B1; C1
lần lượt là hình chiếu vuông góc của M lên BC; CA;AB. Chứng minh rằng
A
1
B
1
C
1 =
P
M=(O) :
ABC 4R
2
Bài toán 3.8. Cho DABC trọng tâm G. Gọi A
0
;B
0
;C
0
là giao của
GA;GB;GC với đường tròn ngoại tiếp (O). Hãy tính
1
+
1
+
1
:
GA
02
GB
02
GC
02
Bài toán 3.9. Cho DABC và điểm I [aA;b B;gC]. Gọi A
0
;B
0
;C
0
là giao
của
IA;IB;IC với đường tròn ngoại tiếp (O). Hãy tính
a
+
b
+
g
:
IA
02
IB
02
IC
02
Bài toán 3.10. Cho DABC nội tiếp trong đường tròn (O), hai phân giác trong là
AE;BF, đường thẳng EF cắt (O) tại I: Chứng minh rằng
1
=
1
+
1
:
IA IB IC
Bài toán 3.11. Cho DABC nội tiếp trong đường tròn (O). Giả sử I
[aA : b B :
gC]. Các đường thẳng AI;BI cắt BC;CA tại E;F. Tia EF cắt (O) tại I. Chứng
minh rằng
a
=
b
+
g
:
a:IA b :IB g:IC
3.2 Một số bài toán thi học sinh giỏi và thi Olympic
Tải tài liệu tại sividoc.com
Viết đề tài giá sinh viên – ZALO:0973.287.149-TEAMLUANVAN.COM
Ta sẽ bổ sung thêm một số kỹ thuật cơ bản trước khi áp dụng các
phương pháp nói trên vào giải các bài toán ở mức độ khó: Các bài
toán thi học sinh giỏi và thi Olympic.
Tải tài liệu tại sividoc.com
Viết đề tài giá sinh viên – ZALO:0973.287.149-TEAMLUANVAN.COM
51
3.2.1 Véc tơ chuyển chỗ
Véc tơ chuyển chỗ của hai điểm (tọa độ chuẩn hóa) P = (p1 : p2 : p3); Q =
(q
1:
q
2:
q
3
) PQ và bằng (p
1
q
1
; p
2
q
2
; p
3
q
3
): Chú ý rằng
được ký hiệu là!
tổng các tọa độ của véc tơ chuyển chỗ p1 q1 + p2 q2 + p3 q3 = 0:
Trong các phép chứng minh tiếp theo ta sẽ dẫn ra một số tính chất sau với
quy ước DABC là tam giác cơ sở và O là tâm đường tròn ngoại tiếp tam giác
ABC, bán kính đường tròn ngoại tiếp là R:
! ! ! !
O = 0 ;tức OO = 0
! ! ! !
A : A = R2
;tức OA:OA = R2
c
2
c
2
! ! ! !
A:B =R
2
;tức OA:OB= R
2
:
2 2
3.2.2 Đường thẳng vuông góc
Bổ đề sau được chứng minh nhờ định lý Pythagorean
Bổ đề 3.1. !
MN 1 1 1 ! 2 2 2
): Khi
Xét các véc tơ chuyển chỗ = (x ;y ;z ) và PQ = (x ;y ;z
đó
MN ? PQ , 0 = a
2
(z1y2 + y1z2) + b
2
(x1z2 + z1x2) + c
2
(y1x2 + x1y2):
Lấy
~
=
~
khi đó:
Chứng minh. O 0
! ! ! ! ! !
MN ? PQ , (x1 A + y1 B + z1 C )(x2 A + y2 B + z2 C ) = 0:
Khai triển vế trái ta có
c
2
2
å(x1x2R
2
) + å(x1y2 + x2y1) R
2
= 0:
,R
2
å(x1x2) + å(x1y2 + x2y1)
!
=
1
å(x1y2 + x2y1):(c2
))
2
1
,R
2
(x1 + x2 + x3)(y1 + y2 + y3) = å(x1y2 + x2y1):(c2
))
2
,R
2
0:0 =
1
å(x1y2 + x2y1):(c2
)) , 0 =
1
å(x1y2 + x2y1):(c2
))
2 2
Tải tài liệu tại sividoc.com
Viết đề tài giá sinh viên – ZALO:0973.287.149-TEAMLUANVAN.COM
Trong các biểu thức trên là ký hiệu hoán vị vòng tròn.
Tải tài liệu tại sividoc.com
Viết đề tài giá sinh viên – ZALO:0973.287.149-TEAMLUANVAN.COM
52
Bổ đề trên có hai hệ quả sau:
Hệ quả 3.9.
!
1 1 1
) là véc tơ chuyển chỗ. Khi đó PQ
?
BC khi và chỉ
PQ = (x ;y ;z
khi 0 = a
2
(z1 y1) + x1(c
2
b
2
):
Hệ quả 3.10. Trung trực của cạnh BC trong tam giác ABC có phương
trình 0 = a
2
(z y) + x(c
2
b
2
):
3.2.3 Phương trình đường tròn
Giả sử DABC là tam giác cơ sở, độ dài các cạnh là a; b; c. Khi đó
phương trình tổng quát của đường tròn là
a
2
yz b
2
zx c
2
xy + (ux + vy + wz)(x + y + z) = 0:
Chứng minh. Giả sử đường tròn có tâm ( j : k : l) và bán kính r. Điều
đó có nghĩa là
a
2
(y k)(z l) b
2
(z l)(x j) c
2
(x j)(y k) = r
2
:
Khai triển vế trái
a
2
yz b
2
zx c
2
xy + D1x + D2y + D3z = D
với các hằng số Di; D nào đó. Vì x + y + z = 1 nên ta có thể viết lại
đẳng thức đó là
a2yz
a2yz
b2zx
b2zx
c2xy + ux + vy + wz = 0 ,
c2xy + (ux + vy + wz)(x + y + z) = 0:
Trường hợp đặc biệt: Đường tròn ngoại tiếp tam giác ABC có phương
trình a
2
yz + b
2
zx + c
2
xy = 0:
Tất cả các bài toán sau đây đều đã có lời giải theo phương pháp truyền
thống. Khi áp dụng tọa độ tâm tỷ cự ta nhận được cách giải độc đáo, thể
hiện ưu thế của phương pháp tâm tỷ cự. Chúng ta bắt đầu với các bài toán
USAMO, được giải bằng cách áp dụng tâm tỷ cự với các kỹ thuật đã nói
trong chương 2. Phần này có tham khảo cách giải của Evan Chen trong[6].
Tải tài liệu tại sividoc.com
Viết đề tài giá sinh viên – ZALO:0973.287.149-TEAMLUANVAN.COM
53
Ví dụ 3.4. (MOP 2006) Tam giác ABC nội tiếp trong đường tròn w. Điểm
P nằm trên đường thẳng BC sao cho PA tiếp xúc với w: Phân giác góc
APBd cắt các cạnh AB; AC tương ứng tại D; E, các cạnh BE; CD cắt
nhau tại Q. Cho đường thẳng PQ đi qua tâm của w; hãy tính góc BACd:
Lời giải.
Hình 3.2: MOP 2006
Chứng minh.
Từ sự đồng dạng của các tam giác PBA;PAC ta có
PB
=
PA
=
c
nên
BD
=
PA PC b DA
c
và
AE
=
c
. Điều đó cho D(c : b : 0);E(b : 0 : c);DQ(bc : b2
: c2
): Điểm
b EC b
P 2 BC có dạng P(0 : x : y) với x;y nào đó và vì P, D và E thẳng hàng nên ta có
0 x y = 0, tức làx = b
2
. Do đó, P = (0 : b
2
: c
2
).
c b 0 2
y c
b 0 c
Cuối cùng, vì P;Q và O = (a cos a : : ) thẳng hàng ta tìm được
0 b 2
bc b 2
a cos a b cos b
2 = bc; 0 b c = 0:
c2
bc b c
c
c cos g a cos a cos b cos g
Điều đó kéo theo 2abc cos a = bc2
cos b + b2
cos g = bc(c cos b + b cos g) =
abc; tức là cos a =
1
;a = 600
:
2
Tải tài liệu tại sividoc.com
Viết đề tài giá sinh viên – ZALO:0973.287.149-TEAMLUANVAN.COM
54
Ví dụ 3.5. (USAMO 2001 #2) Giả sử ABC là một tam giác có w là tâm
đường tròn nội tiếp. Ký hiệu D1;E1 là các tiếp điểm của (w) với các
cạnh BC;AC (tương ứng). D2;E2 là các điểm trên cạnh BC;AC (tương
ứng) sao cho CD2 = BD1;CE2 = AE1. Ngoài ra P = AD2  BE2. Đường
tròn (w) giao với AD2 tại hai điểm, điểm gần với A được ký hiệu là Q.
Chứng minh rằng AQ = D2P.
Hình 3.3: USAMO 2001 #2
Chứng minh.
Ta có thể xác định tọa độ tỷ cự của các điểm w = 1(a : b : c); D1 =1 (0 :
1 1 2p a
p c : p b); D2 = (0 : p b : p c); E2 = (p a : 0 : p c); P Na =
a b
1(p a : p p : p c) (Na là điểm Nagel). Để tìm tọa độ điểm Q ta chú ý rằng
a
phép vị tự tâm A biến đường tròn bàng tiếp góc A thành đường tròn nội tiếp sẽ
biến D2 thành Q. Nghĩa là bán kính wQ song song với waD2 k wD1 hay Q là
điểm xuyên tâm đối của D1: Vì vậy Q = 2w D1 = ( a : b p c :c p b ).
a p a
p p
Đó là tất cả những gì cần để chứng minh AQ = PD2.
AQ=PD , tức là:
Ta sẽ chứng minh
!
! 2
p 1 : pa : p a = p : a p : a p :
a b p c c p b p a p b p b p c p c
Có thể kiểm tra đẳng thức đó trực tiếp: Các tọa độ thứ nhất bằng nhau, các tọa
độ thứ hai cũng vậy do 2p = a + b + c và tương tự đối với các tọa độ thứ ba.
Tải tài liệu tại sividoc.com
Viết đề tài giá sinh viên – ZALO:0973.287.149-TEAMLUANVAN.COM
55
Phương pháp khác là đi chứng minh AQB = PD2B. Điều đó chính là
a b
1
p p
0
tức là: p b c =
a p
đã được kiểm tra.
p c c p b
p
p
p
p
b
p
c
= a p ;
p b p c
0 0
a p a 0
a a
0 1 0
1 0
p a p c 2
p a ()
a + b + c =
2
p
. Đẳng thức
! !
: AQ=PD
Ví dụ 3.6. (USAMO, 2008, #2) Giả sử ABC là một tam giác nhọn,
không cân. M, N, P là trung điểm các cạnh BC;CA;AB tương ứng.
Trung trực của AB và AC cắt tia AM tương ứng ở D và E và giả sử các
đường thẳng BD và CE giao nhau tại F ở trong tam giác ABC. Chứng
minh rằng A;N;F;P nằm trên một đường tròn.
Hình 3.4: USAMO 2008
Chứng minh.
1 1
Giả sử A = (1 : 0 : 0);B = (0 : 1 : 0);C = (0 : 0 : 1). Hiển nhiên, F = (: :
0),...Kiểm tra được phương trình của AM là y = z. Bây giờ ta sẽ tính
tọa độ của D. Vì D 2 AM nên có thể viết D = (1 2t;t;t). Áp dụng điều
kiện vuông góc DP ? AB:
Kỹ thuật biến đổi tâm tỷ cự Và ứng dụng vào giải toán.doc
Kỹ thuật biến đổi tâm tỷ cự Và ứng dụng vào giải toán.doc
Kỹ thuật biến đổi tâm tỷ cự Và ứng dụng vào giải toán.doc
Kỹ thuật biến đổi tâm tỷ cự Và ứng dụng vào giải toán.doc
Kỹ thuật biến đổi tâm tỷ cự Và ứng dụng vào giải toán.doc

More Related Content

Similar to Kỹ thuật biến đổi tâm tỷ cự Và ứng dụng vào giải toán.doc

Ứng Dụng Hình Học Giải Tích Vào Giải Phương Trình, Bất Phương Trình Và Hệ Phư...
Ứng Dụng Hình Học Giải Tích Vào Giải Phương Trình, Bất Phương Trình Và Hệ Phư...Ứng Dụng Hình Học Giải Tích Vào Giải Phương Trình, Bất Phương Trình Và Hệ Phư...
Ứng Dụng Hình Học Giải Tích Vào Giải Phương Trình, Bất Phương Trình Và Hệ Phư...Dịch vụ viết đề tài trọn gói 0934.573.149
 
Luận Văn Phương Pháp Phần Tử Hữu Hạn Tính Khung Một Nhịp Có Xét Đến Biến Dạng...
Luận Văn Phương Pháp Phần Tử Hữu Hạn Tính Khung Một Nhịp Có Xét Đến Biến Dạng...Luận Văn Phương Pháp Phần Tử Hữu Hạn Tính Khung Một Nhịp Có Xét Đến Biến Dạng...
Luận Văn Phương Pháp Phần Tử Hữu Hạn Tính Khung Một Nhịp Có Xét Đến Biến Dạng...sividocz
 
Xây Dựng Bộ Điều Khiển Mờ Thích Nghi Để Cân Bằng Tải Cho Hệ Hai Động Cơ Một C...
Xây Dựng Bộ Điều Khiển Mờ Thích Nghi Để Cân Bằng Tải Cho Hệ Hai Động Cơ Một C...Xây Dựng Bộ Điều Khiển Mờ Thích Nghi Để Cân Bằng Tải Cho Hệ Hai Động Cơ Một C...
Xây Dựng Bộ Điều Khiển Mờ Thích Nghi Để Cân Bằng Tải Cho Hệ Hai Động Cơ Một C...DV Viết Luận văn luanvanmaster.com ZALO 0973287149
 
Nghiên Cứu Thiết Kế Hệ Thống Điều Khiển Tốc Độ Động Cơ Đồng Bộ Nam Châm Vĩnh ...
Nghiên Cứu Thiết Kế Hệ Thống Điều Khiển Tốc Độ Động Cơ Đồng Bộ Nam Châm Vĩnh ...Nghiên Cứu Thiết Kế Hệ Thống Điều Khiển Tốc Độ Động Cơ Đồng Bộ Nam Châm Vĩnh ...
Nghiên Cứu Thiết Kế Hệ Thống Điều Khiển Tốc Độ Động Cơ Đồng Bộ Nam Châm Vĩnh ...DV Viết Luận văn luanvanmaster.com ZALO 0973287149
 
3 câu thi thử;
3 câu thi thử;3 câu thi thử;
3 câu thi thử;nam nam
 
bộ 3 câu hỏi khó phân loại trong đề thi thử THPT Quốc gia môn Toán​ năm 2015
bộ 3 câu hỏi khó phân loại trong đề thi thử THPT Quốc gia môn Toán​ năm 2015bộ 3 câu hỏi khó phân loại trong đề thi thử THPT Quốc gia môn Toán​ năm 2015
bộ 3 câu hỏi khó phân loại trong đề thi thử THPT Quốc gia môn Toán​ năm 2015Linh Nguyễn
 
Một số lớp bài toán tối ưu không lồi, Thuật toán và ứng dụng.pdf
Một số lớp bài toán tối ưu không lồi, Thuật toán và ứng dụng.pdfMột số lớp bài toán tối ưu không lồi, Thuật toán và ứng dụng.pdf
Một số lớp bài toán tối ưu không lồi, Thuật toán và ứng dụng.pdfMan_Ebook
 

Similar to Kỹ thuật biến đổi tâm tỷ cự Và ứng dụng vào giải toán.doc (20)

Luận văn: Phân tích bài toán tuyến tính kết cấu dàn, HOT
Luận văn: Phân tích bài toán tuyến tính kết cấu dàn, HOTLuận văn: Phân tích bài toán tuyến tính kết cấu dàn, HOT
Luận văn: Phân tích bài toán tuyến tính kết cấu dàn, HOT
 
Nghiên Cứu Cấu Trúc Đa Thù Hình Và Các Tính Chất Động Học Của Co Bằng Phương ...
Nghiên Cứu Cấu Trúc Đa Thù Hình Và Các Tính Chất Động Học Của Co Bằng Phương ...Nghiên Cứu Cấu Trúc Đa Thù Hình Và Các Tính Chất Động Học Của Co Bằng Phương ...
Nghiên Cứu Cấu Trúc Đa Thù Hình Và Các Tính Chất Động Học Của Co Bằng Phương ...
 
Ứng Dụng Hình Học Giải Tích Vào Giải Phương Trình, Bất Phương Trình Và Hệ Phư...
Ứng Dụng Hình Học Giải Tích Vào Giải Phương Trình, Bất Phương Trình Và Hệ Phư...Ứng Dụng Hình Học Giải Tích Vào Giải Phương Trình, Bất Phương Trình Và Hệ Phư...
Ứng Dụng Hình Học Giải Tích Vào Giải Phương Trình, Bất Phương Trình Và Hệ Phư...
 
Luận Văn Phương Pháp Phần Tử Hữu Hạn Tính Khung Một Nhịp Có Xét Đến Biến Dạng...
Luận Văn Phương Pháp Phần Tử Hữu Hạn Tính Khung Một Nhịp Có Xét Đến Biến Dạng...Luận Văn Phương Pháp Phần Tử Hữu Hạn Tính Khung Một Nhịp Có Xét Đến Biến Dạng...
Luận Văn Phương Pháp Phần Tử Hữu Hạn Tính Khung Một Nhịp Có Xét Đến Biến Dạng...
 
Xây Dựng Bộ Điều Khiển Mờ Thích Nghi Để Cân Bằng Tải Cho Hệ Hai Động Cơ Một C...
Xây Dựng Bộ Điều Khiển Mờ Thích Nghi Để Cân Bằng Tải Cho Hệ Hai Động Cơ Một C...Xây Dựng Bộ Điều Khiển Mờ Thích Nghi Để Cân Bằng Tải Cho Hệ Hai Động Cơ Một C...
Xây Dựng Bộ Điều Khiển Mờ Thích Nghi Để Cân Bằng Tải Cho Hệ Hai Động Cơ Một C...
 
Ứng Dụng Mạng Nơron Nhân Tạo Dự Báo Số Học Sinh Tuyển Vào Trung Tâm Gdnn – Gd...
Ứng Dụng Mạng Nơron Nhân Tạo Dự Báo Số Học Sinh Tuyển Vào Trung Tâm Gdnn – Gd...Ứng Dụng Mạng Nơron Nhân Tạo Dự Báo Số Học Sinh Tuyển Vào Trung Tâm Gdnn – Gd...
Ứng Dụng Mạng Nơron Nhân Tạo Dự Báo Số Học Sinh Tuyển Vào Trung Tâm Gdnn – Gd...
 
Các Mô Hình Học Sâu Tiên Tiến Và Ứng Dụng Trong Phân Tích Chuỗi Thời Gian Lâm...
Các Mô Hình Học Sâu Tiên Tiến Và Ứng Dụng Trong Phân Tích Chuỗi Thời Gian Lâm...Các Mô Hình Học Sâu Tiên Tiến Và Ứng Dụng Trong Phân Tích Chuỗi Thời Gian Lâm...
Các Mô Hình Học Sâu Tiên Tiến Và Ứng Dụng Trong Phân Tích Chuỗi Thời Gian Lâm...
 
Luận văn: Phương pháp giải phương trình chứa ẩn dưới dấu căn
Luận văn: Phương pháp giải phương trình chứa ẩn dưới dấu cănLuận văn: Phương pháp giải phương trình chứa ẩn dưới dấu căn
Luận văn: Phương pháp giải phương trình chứa ẩn dưới dấu căn
 
Một số phương pháp giải các đề thi olympic Về phương trình diophant.docx
Một số phương pháp giải các đề thi olympic Về phương trình diophant.docxMột số phương pháp giải các đề thi olympic Về phương trình diophant.docx
Một số phương pháp giải các đề thi olympic Về phương trình diophant.docx
 
Luận văn: Tính toán kết cấu bằng phương pháp so sánh, HAY
Luận văn: Tính toán kết cấu bằng phương pháp so sánh, HAYLuận văn: Tính toán kết cấu bằng phương pháp so sánh, HAY
Luận văn: Tính toán kết cấu bằng phương pháp so sánh, HAY
 
Nghiên Cứu Thiết Kế, Mô Phỏng Và Thử Nghiệm Cảm Biến Góc Nghiêng Hai Chiều Cấ...
Nghiên Cứu Thiết Kế, Mô Phỏng Và Thử Nghiệm Cảm Biến Góc Nghiêng Hai Chiều Cấ...Nghiên Cứu Thiết Kế, Mô Phỏng Và Thử Nghiệm Cảm Biến Góc Nghiêng Hai Chiều Cấ...
Nghiên Cứu Thiết Kế, Mô Phỏng Và Thử Nghiệm Cảm Biến Góc Nghiêng Hai Chiều Cấ...
 
Đề tài: Phương pháp phần tử hữu hạn tính khung một nhịp, HAY
Đề tài: Phương pháp phần tử hữu hạn tính khung một nhịp, HAYĐề tài: Phương pháp phần tử hữu hạn tính khung một nhịp, HAY
Đề tài: Phương pháp phần tử hữu hạn tính khung một nhịp, HAY
 
Nghiên Cứu Thiết Kế Hệ Thống Điều Khiển Tốc Độ Động Cơ Đồng Bộ Nam Châm Vĩnh ...
Nghiên Cứu Thiết Kế Hệ Thống Điều Khiển Tốc Độ Động Cơ Đồng Bộ Nam Châm Vĩnh ...Nghiên Cứu Thiết Kế Hệ Thống Điều Khiển Tốc Độ Động Cơ Đồng Bộ Nam Châm Vĩnh ...
Nghiên Cứu Thiết Kế Hệ Thống Điều Khiển Tốc Độ Động Cơ Đồng Bộ Nam Châm Vĩnh ...
 
Bat Phương Trình Hàm Sinh B I Các Đại Lư Ng Trung Bình B C Tùy Ý Và Các Dạng ...
Bat Phương Trình Hàm Sinh B I Các Đại Lư Ng Trung Bình B C Tùy Ý Và Các Dạng ...Bat Phương Trình Hàm Sinh B I Các Đại Lư Ng Trung Bình B C Tùy Ý Và Các Dạng ...
Bat Phương Trình Hàm Sinh B I Các Đại Lư Ng Trung Bình B C Tùy Ý Và Các Dạng ...
 
Phương Trình Bậc Bốn Và Các Hệ Thức Hình Học Trong Tứ Giác Hai Tâm.doc
Phương Trình Bậc Bốn Và Các Hệ Thức Hình Học Trong Tứ Giác Hai Tâm.docPhương Trình Bậc Bốn Và Các Hệ Thức Hình Học Trong Tứ Giác Hai Tâm.doc
Phương Trình Bậc Bốn Và Các Hệ Thức Hình Học Trong Tứ Giác Hai Tâm.doc
 
3 câu thi thử;
3 câu thi thử;3 câu thi thử;
3 câu thi thử;
 
bộ 3 câu hỏi khó phân loại trong đề thi thử THPT Quốc gia môn Toán​ năm 2015
bộ 3 câu hỏi khó phân loại trong đề thi thử THPT Quốc gia môn Toán​ năm 2015bộ 3 câu hỏi khó phân loại trong đề thi thử THPT Quốc gia môn Toán​ năm 2015
bộ 3 câu hỏi khó phân loại trong đề thi thử THPT Quốc gia môn Toán​ năm 2015
 
Nghiên Cứu Tính Chất Và Động Học Phát Quang Của Các Hạt Nano Bán Dẫn Cdse Tro...
Nghiên Cứu Tính Chất Và Động Học Phát Quang Của Các Hạt Nano Bán Dẫn Cdse Tro...Nghiên Cứu Tính Chất Và Động Học Phát Quang Của Các Hạt Nano Bán Dẫn Cdse Tro...
Nghiên Cứu Tính Chất Và Động Học Phát Quang Của Các Hạt Nano Bán Dẫn Cdse Tro...
 
Phương Pháp Phương Trình Đại Số Chứng Minh Các Hệ Thức Lượng Giác.docx
Phương Pháp Phương Trình Đại Số Chứng Minh Các Hệ Thức Lượng Giác.docxPhương Pháp Phương Trình Đại Số Chứng Minh Các Hệ Thức Lượng Giác.docx
Phương Pháp Phương Trình Đại Số Chứng Minh Các Hệ Thức Lượng Giác.docx
 
Một số lớp bài toán tối ưu không lồi, Thuật toán và ứng dụng.pdf
Một số lớp bài toán tối ưu không lồi, Thuật toán và ứng dụng.pdfMột số lớp bài toán tối ưu không lồi, Thuật toán và ứng dụng.pdf
Một số lớp bài toán tối ưu không lồi, Thuật toán và ứng dụng.pdf
 

More from DV Viết Luận văn luanvanmaster.com ZALO 0973287149

More from DV Viết Luận văn luanvanmaster.com ZALO 0973287149 (20)

Ảnh Hưởng Của Marketing Quan Hệ Đến Lòng Trung Thành Của Khách Hàng.Tình Huốn...
Ảnh Hưởng Của Marketing Quan Hệ Đến Lòng Trung Thành Của Khách Hàng.Tình Huốn...Ảnh Hưởng Của Marketing Quan Hệ Đến Lòng Trung Thành Của Khách Hàng.Tình Huốn...
Ảnh Hưởng Của Marketing Quan Hệ Đến Lòng Trung Thành Của Khách Hàng.Tình Huốn...
 
Phát triển nguồn nhân lực tại Uỷ ban nhân dân huyện Trà Bồng, tỉnh Quảng Ngãi...
Phát triển nguồn nhân lực tại Uỷ ban nhân dân huyện Trà Bồng, tỉnh Quảng Ngãi...Phát triển nguồn nhân lực tại Uỷ ban nhân dân huyện Trà Bồng, tỉnh Quảng Ngãi...
Phát triển nguồn nhân lực tại Uỷ ban nhân dân huyện Trà Bồng, tỉnh Quảng Ngãi...
 
Báo cáo tốt Nghiệp tài chính hợp nhất tại tổng công ty Indochina gol...
Báo cáo tốt Nghiệp  tài chính hợp nhất tại tổng công ty Indochina gol...Báo cáo tốt Nghiệp  tài chính hợp nhất tại tổng công ty Indochina gol...
Báo cáo tốt Nghiệp tài chính hợp nhất tại tổng công ty Indochina gol...
 
Tạo động lực thúc đẩy nhân viên làm việc tại ngân hàng TMCP Ngoại Thương Việt...
Tạo động lực thúc đẩy nhân viên làm việc tại ngân hàng TMCP Ngoại Thương Việt...Tạo động lực thúc đẩy nhân viên làm việc tại ngân hàng TMCP Ngoại Thương Việt...
Tạo động lực thúc đẩy nhân viên làm việc tại ngân hàng TMCP Ngoại Thương Việt...
 
Phát triển công nghiệp trên địa bàn Thành phố Tam Kỳ, Tỉnh Quảng Na...
Phát triển công nghiệp trên địa bàn Thành phố Tam Kỳ, Tỉnh Quảng Na...Phát triển công nghiệp trên địa bàn Thành phố Tam Kỳ, Tỉnh Quảng Na...
Phát triển công nghiệp trên địa bàn Thành phố Tam Kỳ, Tỉnh Quảng Na...
 
Giải pháp phát triển cho vay xuất nhập khẩu tại ngân hàng NN&PTNN ch...
Giải pháp phát triển cho vay xuất nhập khẩu tại ngân hàng NN&PTNN ch...Giải pháp phát triển cho vay xuất nhập khẩu tại ngân hàng NN&PTNN ch...
Giải pháp phát triển cho vay xuất nhập khẩu tại ngân hàng NN&PTNN ch...
 
Hoàn thiện công tác lập báo cáo tài chính hợp nhất tại tổng công ...
Hoàn thiện công tác lập báo cáo tài chính hợp nhất tại tổng công ...Hoàn thiện công tác lập báo cáo tài chính hợp nhất tại tổng công ...
Hoàn thiện công tác lập báo cáo tài chính hợp nhất tại tổng công ...
 
Luận Văn Thạc Sĩ Quản trị thành tích nhân viên tại Cục Hải quan TP Đà Nẵng.doc
Luận Văn Thạc Sĩ  Quản trị thành tích nhân viên tại Cục Hải quan TP Đà Nẵng.docLuận Văn Thạc Sĩ  Quản trị thành tích nhân viên tại Cục Hải quan TP Đà Nẵng.doc
Luận Văn Thạc Sĩ Quản trị thành tích nhân viên tại Cục Hải quan TP Đà Nẵng.doc
 
Hoàn thiện công tác quản lý thuế thu nhập cá nhân tại cục thuế Tỉ...
Hoàn thiện công tác quản lý thuế thu nhập cá nhân tại cục thuế Tỉ...Hoàn thiện công tác quản lý thuế thu nhập cá nhân tại cục thuế Tỉ...
Hoàn thiện công tác quản lý thuế thu nhập cá nhân tại cục thuế Tỉ...
 
Đề Tài Phát triển bền vững nông nghiệp Huyện Ba Tơ, Tỉnh Quảng Ngãi....
Đề Tài Phát triển bền vững nông nghiệp Huyện Ba Tơ, Tỉnh Quảng Ngãi....Đề Tài Phát triển bền vững nông nghiệp Huyện Ba Tơ, Tỉnh Quảng Ngãi....
Đề Tài Phát triển bền vững nông nghiệp Huyện Ba Tơ, Tỉnh Quảng Ngãi....
 
Hoàn thiện công tác bảo trợ xã hội trên địa bàn huyện Phong Điền, tỉnh Thừa T...
Hoàn thiện công tác bảo trợ xã hội trên địa bàn huyện Phong Điền, tỉnh Thừa T...Hoàn thiện công tác bảo trợ xã hội trên địa bàn huyện Phong Điền, tỉnh Thừa T...
Hoàn thiện công tác bảo trợ xã hội trên địa bàn huyện Phong Điền, tỉnh Thừa T...
 
Đề Tài Luận VănPhát triển sản phẩm du lịch tại thành phố Đà Nẵng.doc
Đề Tài Luận VănPhát triển sản phẩm du lịch tại thành phố Đà Nẵng.docĐề Tài Luận VănPhát triển sản phẩm du lịch tại thành phố Đà Nẵng.doc
Đề Tài Luận VănPhát triển sản phẩm du lịch tại thành phố Đà Nẵng.doc
 
Đào tạo nghề cho lao động thuộc diện thu hồi đất trên địa bàn Thàn...
Đào tạo nghề cho lao động thuộc diện thu hồi đất trên địa bàn Thàn...Đào tạo nghề cho lao động thuộc diện thu hồi đất trên địa bàn Thàn...
Đào tạo nghề cho lao động thuộc diện thu hồi đất trên địa bàn Thàn...
 
Tóm Tắt Luận Văn Thạc Sĩ Quản Trị Kinh Doanh Xây dựng chính sách Marketing tạ...
Tóm Tắt Luận Văn Thạc Sĩ Quản Trị Kinh Doanh Xây dựng chính sách Marketing tạ...Tóm Tắt Luận Văn Thạc Sĩ Quản Trị Kinh Doanh Xây dựng chính sách Marketing tạ...
Tóm Tắt Luận Văn Thạc Sĩ Quản Trị Kinh Doanh Xây dựng chính sách Marketing tạ...
 
Đề Tài Nghiên cứu rủi ro cảm nhận đối với mua hàng thời trang trực tuyến.docx
Đề Tài Nghiên cứu rủi ro cảm nhận đối với mua hàng thời trang trực tuyến.docxĐề Tài Nghiên cứu rủi ro cảm nhận đối với mua hàng thời trang trực tuyến.docx
Đề Tài Nghiên cứu rủi ro cảm nhận đối với mua hàng thời trang trực tuyến.docx
 
Giải pháp nâng cao động lực thúc đẩy người lao động tại công ty khai...
Giải pháp nâng cao động lực thúc đẩy người lao động tại công ty khai...Giải pháp nâng cao động lực thúc đẩy người lao động tại công ty khai...
Giải pháp nâng cao động lực thúc đẩy người lao động tại công ty khai...
 
Giải pháp phát triển dịch vụ ngân hàng điện tử tại ngân hàng đầu ...
Giải pháp phát triển dịch vụ ngân hàng điện tử tại ngân hàng đầu ...Giải pháp phát triển dịch vụ ngân hàng điện tử tại ngân hàng đầu ...
Giải pháp phát triển dịch vụ ngân hàng điện tử tại ngân hàng đầu ...
 
Giải pháp phát triển dịch vụ ngân hàng điện tử tại ngân hàng đầu ...
Giải pháp phát triển dịch vụ ngân hàng điện tử tại ngân hàng đầu ...Giải pháp phát triển dịch vụ ngân hàng điện tử tại ngân hàng đầu ...
Giải pháp phát triển dịch vụ ngân hàng điện tử tại ngân hàng đầu ...
 
Quản trị quan hệ khách hàng tại Chi nhánh Viettel Đà Nẵng – Tập đoàn Viễn thô...
Quản trị quan hệ khách hàng tại Chi nhánh Viettel Đà Nẵng – Tập đoàn Viễn thô...Quản trị quan hệ khách hàng tại Chi nhánh Viettel Đà Nẵng – Tập đoàn Viễn thô...
Quản trị quan hệ khách hàng tại Chi nhánh Viettel Đà Nẵng – Tập đoàn Viễn thô...
 
Đề Tài Đánh giá thành tích đội ngũ giảng viên trường Đại Học Phạm ...
Đề Tài Đánh giá thành tích đội ngũ giảng viên trường Đại Học Phạm ...Đề Tài Đánh giá thành tích đội ngũ giảng viên trường Đại Học Phạm ...
Đề Tài Đánh giá thành tích đội ngũ giảng viên trường Đại Học Phạm ...
 

Recently uploaded

TỔNG HỢP HƠN 100 ĐỀ THI THỬ TỐT NGHIỆP THPT TOÁN 2024 - TỪ CÁC TRƯỜNG, TRƯỜNG...
TỔNG HỢP HƠN 100 ĐỀ THI THỬ TỐT NGHIỆP THPT TOÁN 2024 - TỪ CÁC TRƯỜNG, TRƯỜNG...TỔNG HỢP HƠN 100 ĐỀ THI THỬ TỐT NGHIỆP THPT TOÁN 2024 - TỪ CÁC TRƯỜNG, TRƯỜNG...
TỔNG HỢP HƠN 100 ĐỀ THI THỬ TỐT NGHIỆP THPT TOÁN 2024 - TỪ CÁC TRƯỜNG, TRƯỜNG...Nguyen Thanh Tu Collection
 
NHKTS SLIDE B2 KHAI NIEM FINTECH VA YEU TO CUNG CAU DOI MOI TRONG CN_GV HANG ...
NHKTS SLIDE B2 KHAI NIEM FINTECH VA YEU TO CUNG CAU DOI MOI TRONG CN_GV HANG ...NHKTS SLIDE B2 KHAI NIEM FINTECH VA YEU TO CUNG CAU DOI MOI TRONG CN_GV HANG ...
NHKTS SLIDE B2 KHAI NIEM FINTECH VA YEU TO CUNG CAU DOI MOI TRONG CN_GV HANG ...songtoan982017
 
Báo cáo bài tập nhóm môn Văn hóa kinh doanh và tinh thần khởi nghiệp Trình bà...
Báo cáo bài tập nhóm môn Văn hóa kinh doanh và tinh thần khởi nghiệp Trình bà...Báo cáo bài tập nhóm môn Văn hóa kinh doanh và tinh thần khởi nghiệp Trình bà...
Báo cáo bài tập nhóm môn Văn hóa kinh doanh và tinh thần khởi nghiệp Trình bà...lamluanvan.net Viết thuê luận văn
 
Báo cáo tốt nghiệp Đánh giá thực trạng an toàn vệ sinh lao động và rủi ro lao...
Báo cáo tốt nghiệp Đánh giá thực trạng an toàn vệ sinh lao động và rủi ro lao...Báo cáo tốt nghiệp Đánh giá thực trạng an toàn vệ sinh lao động và rủi ro lao...
Báo cáo tốt nghiệp Đánh giá thực trạng an toàn vệ sinh lao động và rủi ro lao...lamluanvan.net Viết thuê luận văn
 
Phân tích báo cáo tài chính tại công ty TNHH xây dựng và thương mại Thịnh An
Phân tích báo cáo tài chính tại công ty TNHH xây dựng và thương mại Thịnh AnPhân tích báo cáo tài chính tại công ty TNHH xây dựng và thương mại Thịnh An
Phân tích báo cáo tài chính tại công ty TNHH xây dựng và thương mại Thịnh Anlamluanvan.net Viết thuê luận văn
 
40 ĐỀ LUYỆN THI ĐÁNH GIÁ NĂNG LỰC ĐẠI HỌC QUỐC GIA HÀ NỘI NĂM 2024 (ĐỀ 21-30)...
40 ĐỀ LUYỆN THI ĐÁNH GIÁ NĂNG LỰC ĐẠI HỌC QUỐC GIA HÀ NỘI NĂM 2024 (ĐỀ 21-30)...40 ĐỀ LUYỆN THI ĐÁNH GIÁ NĂNG LỰC ĐẠI HỌC QUỐC GIA HÀ NỘI NĂM 2024 (ĐỀ 21-30)...
40 ĐỀ LUYỆN THI ĐÁNH GIÁ NĂNG LỰC ĐẠI HỌC QUỐC GIA HÀ NỘI NĂM 2024 (ĐỀ 21-30)...Nguyen Thanh Tu Collection
 
4.NGÂN HÀNG KĨ THUẬT SỐ-slide CHƯƠNG 3.pptx
4.NGÂN HÀNG KĨ THUẬT SỐ-slide CHƯƠNG 3.pptx4.NGÂN HÀNG KĨ THUẬT SỐ-slide CHƯƠNG 3.pptx
4.NGÂN HÀNG KĨ THUẬT SỐ-slide CHƯƠNG 3.pptxsongtoan982017
 
Bài tập lớn môn Văn hóa kinh doanh và tinh thần khởi nghiệp Trình bày về triế...
Bài tập lớn môn Văn hóa kinh doanh và tinh thần khởi nghiệp Trình bày về triế...Bài tập lớn môn Văn hóa kinh doanh và tinh thần khởi nghiệp Trình bày về triế...
Bài tập lớn môn Văn hóa kinh doanh và tinh thần khởi nghiệp Trình bày về triế...lamluanvan.net Viết thuê luận văn
 
Mở rộng hoạt động cho vay tiêu dùng tại Ngân hàng TMCP Hàng Hải Việt Nam (Mar...
Mở rộng hoạt động cho vay tiêu dùng tại Ngân hàng TMCP Hàng Hải Việt Nam (Mar...Mở rộng hoạt động cho vay tiêu dùng tại Ngân hàng TMCP Hàng Hải Việt Nam (Mar...
Mở rộng hoạt động cho vay tiêu dùng tại Ngân hàng TMCP Hàng Hải Việt Nam (Mar...lamluanvan.net Viết thuê luận văn
 
Báo cáo thực tập tốt nghiệp Phân tích thực trạng hoạt động bán hàng tại Công ...
Báo cáo thực tập tốt nghiệp Phân tích thực trạng hoạt động bán hàng tại Công ...Báo cáo thực tập tốt nghiệp Phân tích thực trạng hoạt động bán hàng tại Công ...
Báo cáo thực tập tốt nghiệp Phân tích thực trạng hoạt động bán hàng tại Công ...lamluanvan.net Viết thuê luận văn
 
Vận dụng thi pháp học vào phân tích truyện ngắn Chiếc thuyền ...
Vận dụng thi pháp học vào phân tích truyện ngắn Chiếc thuyền ...Vận dụng thi pháp học vào phân tích truyện ngắn Chiếc thuyền ...
Vận dụng thi pháp học vào phân tích truyện ngắn Chiếc thuyền ...lamluanvan.net Viết thuê luận văn
 
Hoàn thiện công tác kiểm soát chi NSNN qua Kho bạc Nhà nước huyện Tri Tôn – t...
Hoàn thiện công tác kiểm soát chi NSNN qua Kho bạc Nhà nước huyện Tri Tôn – t...Hoàn thiện công tác kiểm soát chi NSNN qua Kho bạc Nhà nước huyện Tri Tôn – t...
Hoàn thiện công tác kiểm soát chi NSNN qua Kho bạc Nhà nước huyện Tri Tôn – t...lamluanvan.net Viết thuê luận văn
 
TỔNG HỢP HƠN 100 ĐỀ THI THỬ TỐT NGHIỆP THPT VẬT LÝ 2024 - TỪ CÁC TRƯỜNG, TRƯ...
TỔNG HỢP HƠN 100 ĐỀ THI THỬ TỐT NGHIỆP THPT VẬT LÝ 2024 - TỪ CÁC TRƯỜNG, TRƯ...TỔNG HỢP HƠN 100 ĐỀ THI THỬ TỐT NGHIỆP THPT VẬT LÝ 2024 - TỪ CÁC TRƯỜNG, TRƯ...
TỔNG HỢP HƠN 100 ĐỀ THI THỬ TỐT NGHIỆP THPT VẬT LÝ 2024 - TỪ CÁC TRƯỜNG, TRƯ...Nguyen Thanh Tu Collection
 
Báo cáo tốt nghiệp Đánh giá rủi ro môi trường từ ô nhiễm hữu cơ nước thải các...
Báo cáo tốt nghiệp Đánh giá rủi ro môi trường từ ô nhiễm hữu cơ nước thải các...Báo cáo tốt nghiệp Đánh giá rủi ro môi trường từ ô nhiễm hữu cơ nước thải các...
Báo cáo tốt nghiệp Đánh giá rủi ro môi trường từ ô nhiễm hữu cơ nước thải các...lamluanvan.net Viết thuê luận văn
 
Báo cáo tốt nghiệp Đánh giá công tác đào tạo và phát triển nguồn nhân lực tại...
Báo cáo tốt nghiệp Đánh giá công tác đào tạo và phát triển nguồn nhân lực tại...Báo cáo tốt nghiệp Đánh giá công tác đào tạo và phát triển nguồn nhân lực tại...
Báo cáo tốt nghiệp Đánh giá công tác đào tạo và phát triển nguồn nhân lực tại...lamluanvan.net Viết thuê luận văn
 
VẤN ĐỀ 12 VI PHẠM HÀNH CHÍNH VÀ.pptx
VẤN ĐỀ 12 VI PHẠM HÀNH CHÍNH VÀ.pptxVẤN ĐỀ 12 VI PHẠM HÀNH CHÍNH VÀ.pptx
VẤN ĐỀ 12 VI PHẠM HÀNH CHÍNH VÀ.pptxGingvin36HC
 
Báo cáo tốt nghiệp Hoàn thiện công tác đào tạo và phát triển nguồn nhân lực c...
Báo cáo tốt nghiệp Hoàn thiện công tác đào tạo và phát triển nguồn nhân lực c...Báo cáo tốt nghiệp Hoàn thiện công tác đào tạo và phát triển nguồn nhân lực c...
Báo cáo tốt nghiệp Hoàn thiện công tác đào tạo và phát triển nguồn nhân lực c...lamluanvan.net Viết thuê luận văn
 
30 ĐỀ PHÁT TRIỂN THEO CẤU TRÚC ĐỀ MINH HỌA BGD NGÀY 22-3-2024 KỲ THI TỐT NGHI...
30 ĐỀ PHÁT TRIỂN THEO CẤU TRÚC ĐỀ MINH HỌA BGD NGÀY 22-3-2024 KỲ THI TỐT NGHI...30 ĐỀ PHÁT TRIỂN THEO CẤU TRÚC ĐỀ MINH HỌA BGD NGÀY 22-3-2024 KỲ THI TỐT NGHI...
30 ĐỀ PHÁT TRIỂN THEO CẤU TRÚC ĐỀ MINH HỌA BGD NGÀY 22-3-2024 KỲ THI TỐT NGHI...Nguyen Thanh Tu Collection
 
TỔNG HỢP HƠN 100 ĐỀ THI THỬ TỐT NGHIỆP THPT HÓA HỌC 2024 - TỪ CÁC TRƯỜNG, TRƯ...
TỔNG HỢP HƠN 100 ĐỀ THI THỬ TỐT NGHIỆP THPT HÓA HỌC 2024 - TỪ CÁC TRƯỜNG, TRƯ...TỔNG HỢP HƠN 100 ĐỀ THI THỬ TỐT NGHIỆP THPT HÓA HỌC 2024 - TỪ CÁC TRƯỜNG, TRƯ...
TỔNG HỢP HƠN 100 ĐỀ THI THỬ TỐT NGHIỆP THPT HÓA HỌC 2024 - TỪ CÁC TRƯỜNG, TRƯ...Nguyen Thanh Tu Collection
 
Everybody Up 1 - Unit 5 - worksheet grade 1
Everybody Up 1 - Unit 5 - worksheet grade 1Everybody Up 1 - Unit 5 - worksheet grade 1
Everybody Up 1 - Unit 5 - worksheet grade 1mskellyworkmail
 

Recently uploaded (20)

TỔNG HỢP HƠN 100 ĐỀ THI THỬ TỐT NGHIỆP THPT TOÁN 2024 - TỪ CÁC TRƯỜNG, TRƯỜNG...
TỔNG HỢP HƠN 100 ĐỀ THI THỬ TỐT NGHIỆP THPT TOÁN 2024 - TỪ CÁC TRƯỜNG, TRƯỜNG...TỔNG HỢP HƠN 100 ĐỀ THI THỬ TỐT NGHIỆP THPT TOÁN 2024 - TỪ CÁC TRƯỜNG, TRƯỜNG...
TỔNG HỢP HƠN 100 ĐỀ THI THỬ TỐT NGHIỆP THPT TOÁN 2024 - TỪ CÁC TRƯỜNG, TRƯỜNG...
 
NHKTS SLIDE B2 KHAI NIEM FINTECH VA YEU TO CUNG CAU DOI MOI TRONG CN_GV HANG ...
NHKTS SLIDE B2 KHAI NIEM FINTECH VA YEU TO CUNG CAU DOI MOI TRONG CN_GV HANG ...NHKTS SLIDE B2 KHAI NIEM FINTECH VA YEU TO CUNG CAU DOI MOI TRONG CN_GV HANG ...
NHKTS SLIDE B2 KHAI NIEM FINTECH VA YEU TO CUNG CAU DOI MOI TRONG CN_GV HANG ...
 
Báo cáo bài tập nhóm môn Văn hóa kinh doanh và tinh thần khởi nghiệp Trình bà...
Báo cáo bài tập nhóm môn Văn hóa kinh doanh và tinh thần khởi nghiệp Trình bà...Báo cáo bài tập nhóm môn Văn hóa kinh doanh và tinh thần khởi nghiệp Trình bà...
Báo cáo bài tập nhóm môn Văn hóa kinh doanh và tinh thần khởi nghiệp Trình bà...
 
Báo cáo tốt nghiệp Đánh giá thực trạng an toàn vệ sinh lao động và rủi ro lao...
Báo cáo tốt nghiệp Đánh giá thực trạng an toàn vệ sinh lao động và rủi ro lao...Báo cáo tốt nghiệp Đánh giá thực trạng an toàn vệ sinh lao động và rủi ro lao...
Báo cáo tốt nghiệp Đánh giá thực trạng an toàn vệ sinh lao động và rủi ro lao...
 
Phân tích báo cáo tài chính tại công ty TNHH xây dựng và thương mại Thịnh An
Phân tích báo cáo tài chính tại công ty TNHH xây dựng và thương mại Thịnh AnPhân tích báo cáo tài chính tại công ty TNHH xây dựng và thương mại Thịnh An
Phân tích báo cáo tài chính tại công ty TNHH xây dựng và thương mại Thịnh An
 
40 ĐỀ LUYỆN THI ĐÁNH GIÁ NĂNG LỰC ĐẠI HỌC QUỐC GIA HÀ NỘI NĂM 2024 (ĐỀ 21-30)...
40 ĐỀ LUYỆN THI ĐÁNH GIÁ NĂNG LỰC ĐẠI HỌC QUỐC GIA HÀ NỘI NĂM 2024 (ĐỀ 21-30)...40 ĐỀ LUYỆN THI ĐÁNH GIÁ NĂNG LỰC ĐẠI HỌC QUỐC GIA HÀ NỘI NĂM 2024 (ĐỀ 21-30)...
40 ĐỀ LUYỆN THI ĐÁNH GIÁ NĂNG LỰC ĐẠI HỌC QUỐC GIA HÀ NỘI NĂM 2024 (ĐỀ 21-30)...
 
4.NGÂN HÀNG KĨ THUẬT SỐ-slide CHƯƠNG 3.pptx
4.NGÂN HÀNG KĨ THUẬT SỐ-slide CHƯƠNG 3.pptx4.NGÂN HÀNG KĨ THUẬT SỐ-slide CHƯƠNG 3.pptx
4.NGÂN HÀNG KĨ THUẬT SỐ-slide CHƯƠNG 3.pptx
 
Bài tập lớn môn Văn hóa kinh doanh và tinh thần khởi nghiệp Trình bày về triế...
Bài tập lớn môn Văn hóa kinh doanh và tinh thần khởi nghiệp Trình bày về triế...Bài tập lớn môn Văn hóa kinh doanh và tinh thần khởi nghiệp Trình bày về triế...
Bài tập lớn môn Văn hóa kinh doanh và tinh thần khởi nghiệp Trình bày về triế...
 
Mở rộng hoạt động cho vay tiêu dùng tại Ngân hàng TMCP Hàng Hải Việt Nam (Mar...
Mở rộng hoạt động cho vay tiêu dùng tại Ngân hàng TMCP Hàng Hải Việt Nam (Mar...Mở rộng hoạt động cho vay tiêu dùng tại Ngân hàng TMCP Hàng Hải Việt Nam (Mar...
Mở rộng hoạt động cho vay tiêu dùng tại Ngân hàng TMCP Hàng Hải Việt Nam (Mar...
 
Báo cáo thực tập tốt nghiệp Phân tích thực trạng hoạt động bán hàng tại Công ...
Báo cáo thực tập tốt nghiệp Phân tích thực trạng hoạt động bán hàng tại Công ...Báo cáo thực tập tốt nghiệp Phân tích thực trạng hoạt động bán hàng tại Công ...
Báo cáo thực tập tốt nghiệp Phân tích thực trạng hoạt động bán hàng tại Công ...
 
Vận dụng thi pháp học vào phân tích truyện ngắn Chiếc thuyền ...
Vận dụng thi pháp học vào phân tích truyện ngắn Chiếc thuyền ...Vận dụng thi pháp học vào phân tích truyện ngắn Chiếc thuyền ...
Vận dụng thi pháp học vào phân tích truyện ngắn Chiếc thuyền ...
 
Hoàn thiện công tác kiểm soát chi NSNN qua Kho bạc Nhà nước huyện Tri Tôn – t...
Hoàn thiện công tác kiểm soát chi NSNN qua Kho bạc Nhà nước huyện Tri Tôn – t...Hoàn thiện công tác kiểm soát chi NSNN qua Kho bạc Nhà nước huyện Tri Tôn – t...
Hoàn thiện công tác kiểm soát chi NSNN qua Kho bạc Nhà nước huyện Tri Tôn – t...
 
TỔNG HỢP HƠN 100 ĐỀ THI THỬ TỐT NGHIỆP THPT VẬT LÝ 2024 - TỪ CÁC TRƯỜNG, TRƯ...
TỔNG HỢP HƠN 100 ĐỀ THI THỬ TỐT NGHIỆP THPT VẬT LÝ 2024 - TỪ CÁC TRƯỜNG, TRƯ...TỔNG HỢP HƠN 100 ĐỀ THI THỬ TỐT NGHIỆP THPT VẬT LÝ 2024 - TỪ CÁC TRƯỜNG, TRƯ...
TỔNG HỢP HƠN 100 ĐỀ THI THỬ TỐT NGHIỆP THPT VẬT LÝ 2024 - TỪ CÁC TRƯỜNG, TRƯ...
 
Báo cáo tốt nghiệp Đánh giá rủi ro môi trường từ ô nhiễm hữu cơ nước thải các...
Báo cáo tốt nghiệp Đánh giá rủi ro môi trường từ ô nhiễm hữu cơ nước thải các...Báo cáo tốt nghiệp Đánh giá rủi ro môi trường từ ô nhiễm hữu cơ nước thải các...
Báo cáo tốt nghiệp Đánh giá rủi ro môi trường từ ô nhiễm hữu cơ nước thải các...
 
Báo cáo tốt nghiệp Đánh giá công tác đào tạo và phát triển nguồn nhân lực tại...
Báo cáo tốt nghiệp Đánh giá công tác đào tạo và phát triển nguồn nhân lực tại...Báo cáo tốt nghiệp Đánh giá công tác đào tạo và phát triển nguồn nhân lực tại...
Báo cáo tốt nghiệp Đánh giá công tác đào tạo và phát triển nguồn nhân lực tại...
 
VẤN ĐỀ 12 VI PHẠM HÀNH CHÍNH VÀ.pptx
VẤN ĐỀ 12 VI PHẠM HÀNH CHÍNH VÀ.pptxVẤN ĐỀ 12 VI PHẠM HÀNH CHÍNH VÀ.pptx
VẤN ĐỀ 12 VI PHẠM HÀNH CHÍNH VÀ.pptx
 
Báo cáo tốt nghiệp Hoàn thiện công tác đào tạo và phát triển nguồn nhân lực c...
Báo cáo tốt nghiệp Hoàn thiện công tác đào tạo và phát triển nguồn nhân lực c...Báo cáo tốt nghiệp Hoàn thiện công tác đào tạo và phát triển nguồn nhân lực c...
Báo cáo tốt nghiệp Hoàn thiện công tác đào tạo và phát triển nguồn nhân lực c...
 
30 ĐỀ PHÁT TRIỂN THEO CẤU TRÚC ĐỀ MINH HỌA BGD NGÀY 22-3-2024 KỲ THI TỐT NGHI...
30 ĐỀ PHÁT TRIỂN THEO CẤU TRÚC ĐỀ MINH HỌA BGD NGÀY 22-3-2024 KỲ THI TỐT NGHI...30 ĐỀ PHÁT TRIỂN THEO CẤU TRÚC ĐỀ MINH HỌA BGD NGÀY 22-3-2024 KỲ THI TỐT NGHI...
30 ĐỀ PHÁT TRIỂN THEO CẤU TRÚC ĐỀ MINH HỌA BGD NGÀY 22-3-2024 KỲ THI TỐT NGHI...
 
TỔNG HỢP HƠN 100 ĐỀ THI THỬ TỐT NGHIỆP THPT HÓA HỌC 2024 - TỪ CÁC TRƯỜNG, TRƯ...
TỔNG HỢP HƠN 100 ĐỀ THI THỬ TỐT NGHIỆP THPT HÓA HỌC 2024 - TỪ CÁC TRƯỜNG, TRƯ...TỔNG HỢP HƠN 100 ĐỀ THI THỬ TỐT NGHIỆP THPT HÓA HỌC 2024 - TỪ CÁC TRƯỜNG, TRƯ...
TỔNG HỢP HƠN 100 ĐỀ THI THỬ TỐT NGHIỆP THPT HÓA HỌC 2024 - TỪ CÁC TRƯỜNG, TRƯ...
 
Everybody Up 1 - Unit 5 - worksheet grade 1
Everybody Up 1 - Unit 5 - worksheet grade 1Everybody Up 1 - Unit 5 - worksheet grade 1
Everybody Up 1 - Unit 5 - worksheet grade 1
 

Kỹ thuật biến đổi tâm tỷ cự Và ứng dụng vào giải toán.doc

  • 1. Tải tài liệu tại sividoc.com Viết đề tài giá sinh viên – ZALO:0973.287.149-TEAMLUANVAN.COM ĐẠI HỌC THÁI NGUYÊN TRƯỜNG ĐẠI HỌC KHOA HỌC --------------  ------------- NGUYỄN VĂN NGHĨA KỸ THUẬT BIẾN ĐỔI TÂM TỶ CỰ VÀ ỨNG DỤNG VÀO GIẢI TOÁN LUẬN VĂN THẠC SĨ TOÁN HỌC THÁI NGUYÊN - 2017
  • 2. Tải tài liệu tại sividoc.com Viết đề tài giá sinh viên – ZALO:0973.287.149-TEAMLUANVAN.COM ĐẠI HỌC THÁI NGUYÊN TRƯỜNG ĐẠI HỌC KHOA HỌC --------------  ------------- NGUYỄN VĂN NGHĨA KỸ THUẬT BIẾN ĐỔI TÂM TỶ CỰ VÀ ỨNG DỤNG VÀO GIẢI TOÁN LUẬN VĂN THẠC SĨ TOÁN HỌC Chuyên ngành: Phương pháp Toán sơ cấp Mã số: 60 46 01 13 NGƯỜI HƯỚNG DẪN KHOA HỌC PGS.TS. NGUYỄN VIỆT HẢI THÁI NGUYÊN - 2017
  • 3. Tải tài liệu tại sividoc.com Viết đề tài giá sinh viên – ZALO:0973.287.149-TEAMLUANVAN.COM i Lời cảm ơn Tôi xin chân thành cảm ơn phòng Đào tạo bộ phận sau đại học, quý thầy cô giảng dạy lớp Cao học K9B (2015 - 2017) Trường Đại Học Khoa Học - Đại Học Thái Nguyên đã tận tình truyền đạt những kiến thức quý báu cũng như tạo điều kiện cho tôi hoàn thành khóa học. Để hoàn thành được luận văn một cách hoàn chỉnh, tôi luôn nhận được sự hướng dẫn và giúp đỡ nhiệt tình của PGS.TS. Nguyễn Việt Hải, Giảng viên cao cấp Trường Đại Học Hải Phòng. Tôi xin chân thành bày tỏ lòng biết ơn sâu sắc đến thầy và xin gửi lời tri ân nhất của tôi đối với những điều thầy đã dành cho tôi. Tôi xin gửi lời cảm ơn chân thành nhất tới gia đình, bạn bè, những người đã luôn động viên, hỗ trợ và tạo mọi điều kiện cho tôi trong suốt quá trình học tập và thực hiện luận văn. Xin trân trọng cảm ơn! Hải Phòng, tháng 6 năm 2017 Người viết Luận văn Nguyễn Văn Nghĩa
  • 4. Tải tài liệu tại sividoc.com Viết đề tài giá sinh viên – ZALO:0973.287.149-TEAMLUANVAN.COM i Danh mục hình 1.1 Quy tắc Archimedes . . . . . . . . . . . . . . . . . . . . . . . 7 1.2 Tọa độ diện tích . . . . . . . . . . . . . . . . . . . . . . . . . . 11 2.1 Chọn tâm tỷ cự . . . . . . . . . . . . . . . . . . . . . . . . . . 19 2.2 Quĩ tích là đường tròn . . . . . . . . . . . . . . . . . . . . . . . 21 2.3 I là đỉnh thứ tư hình bình hành . . . . . . . . . . . . . . . . . . 23 2.4 Trực tâm H . . . . . . . . . . . . . . . . . . . . . . . . . . . . 27 2.5 Tọa độ tỷ cự điểm đồng quy . . . . . . . . . . . . . . . . . . . . 29 2.6 Tính tỷ số . . . . . . . . . . . . . . . . . . . . . . . . . . . . . 32 2.7 Tính diện tích . . . . . . . . . . . . . . . . . . . . . . . . . . . 34 2.8 Hình chóp tam giác đều . . . . . . . . . . . . . . . . . . . . . . 35 3.1 P;Q;R thẳng hàng . . . . . . . . . . . . . . . . . . . . . . . . . 43 3.2 MOP2006............................. 53 3.3 USAMO2001#2......................... 54 3.4 USAMO2008........................... 55
  • 5. Tải tài liệu tại sividoc.com Viết đề tài giá sinh viên – ZALO:0973.287.149-TEAMLUANVAN.COM i Mục lục Lời cảm ơn i Mở đầu 1 1 Tâm tỷ cự của hệ chất điểm 4 1.1 Hệ chất điểm và tâm tỷ cự . . . . . . . . . . . . . . . . . . . . 4 1.2 Các tính chất cơ bản của tâm tỷ cự . . . . . . . . . . . . . . . . 7 1.3 Tâm tỷ cự và diện tích đại số . . . . . . . . . . . . . . . . . . . 9 1.3.1 Diện tích đại số . . . . . . . . . . . . . . . . . . . . . . 9 1.3.2 Tọa độ tỷ cự trong mặt phẳng . . . . . . . . . . . . . . 12 1.4 Công thức Lagrang và công thức Jacobi . . . . . . . . . . . . . 15 2 Các kỹ thuật biến đổi tâm tỷ cự và ứng dụng 18 2.1 Kỹ thuật chọn tâm tỷ cự . . . . . . . . . . . . . . . . . . . . . . 18 2.2 Kỹ thuật diện tích hóa và tọa độ hóa. . . . . . . . . . . . . . . . 26 2.3 Kỹ thuật giao hoán-kết hợp. . . . . . . . . . . . . . . . . . . . . 31 2.4 Kỹ thuật quán tính. . . . . . . . . . . . . . . . . . . . . . . . . 36 3 Các vấn đề liên quan 41 3.1 Chứng minh một số định lý nổi tiếng . . . . . . . . . . . . . . . 41 3.2 Một số bài toán thi học sinh giỏi và thi Olympic . . . . . . . . . 50 3.2.1 Véc tơ chuyển chỗ . . . . . . . . . . . . . . . . . . . . 51 3.2.2 Đường thẳng vuông góc . . . . . . . . . . . . . . . . . 51 3.2.3 Phương trình đường tròn . . . . . . . . . . . . . . . . . 52 Tài liệu tham khảo 59
  • 6. Tải tài liệu tại sividoc.com Viết đề tài giá sinh viên – ZALO:0973.287.149-TEAMLUANVAN.COM 1 Mở đầu 1. Mục đích của đề tài luận văn Khái niệm tâm tỷ cự đã được các nhà toán học đề cập đến từ lâu, chẳng hạn xem ([1], [5], [6]). Tuy nhiên việc ứng dụng khái niệm này còn rất hạn chế vì ngoài định nghĩa thông qua véc tơ, các tính chất và các biểu diễn khác của tâm tỷ cự chưa được nêu trong các tài liệu truyền thống. Mục đích của đề tài này là nghiên cứu đề xuất các tính chất đăc trưng của tâm tỷ cự từ đó đề ra các kỹ thuật biến đổi tâm tỷ cự để giải các loại toán hình học phẳng. Cụ thể là: - Nghiên cứu các tính chất đặc trưng của tâm tỷ cự của hệ chất điểm. Đưa ra các kỹ thuật biến đổi tâm tỷ cự nhằm ứng dụng có hiệu quả vào việc giải toán Hình học. - Ứng dụng các kỹ thuật biến đổi tâm tỷ cự vào giải các bài toán tính toán, chứng minh, tìm tập hợp điểm và các vấn đề khác nhằm khắc sâu phương pháp giải các bài toán liên quan đến tâm tỷ cự. - Các kiến thức được nâng cao: Xây dựng một lý thuyết chặt chẽ và có hệ thống về tâm tỷ cự, các kỹ thuật biến đổi tâm tỷ cự, tính chất mô men quán tính,. . . dựa vào khái niệm véc tơ. Bổ sung thêm một phương pháp hiệu quả khi giải các bài toán hình học sơ cấp. Đặc biệt áp dụng được vào việc giải các bài toán thi olympic Quốc gia và Quốc tế. Có thể nói đây một sáng tạo mới để giải các bài toán hình học, một phương pháp giải toán có hiệu quả. 2. Nội dung của đề tài, những vấn đề cần giải quyết Đề tài sẽ giải quyết các vấn đề sau: Hệ thống, chứng minh các tính chất của tâm tỷ cự, trình bày các kỹ thuật biến đổi tâm tỷ cự để ứng dụng vào giải
  • 7. Tải tài liệu tại sividoc.com Viết đề tài giá sinh viên – ZALO:0973.287.149-TEAMLUANVAN.COM 2 các bài toán hình học có liên quan. Nêu ra được các bài toán mẫu, điển hình minh họa cho các kỹ thuật biến đổi, giải được các bài toán khó, thể hiện được tính hơn hẳn so với cách giải thông thường. Nội dung chia làm 3 chương: Chương 1. Tâm tỷ cự của hệ chất điểm Định nghĩa và nêu các tính chất của tâm tỷ cự chủ yếu là trên mặt phẳng, các kiến thức cần thiết để xây dựng một số kỹ thuật biến đổi tâm tỷ cự, chuẩn bị cho chương hai. Các tính chất được xây dựng và chứng minh chặt chẽ, đầy đủ. Chương 1 gồm 4 mục sau. 1.1. Định nghĩa và ký hiệu. 1.2. Các tính chất cơ bản của tâm tỷ cự. 1.3. Các ví dụ mở đầu. 1.4. Công thức Lagrang và công thức Jacobi. Chương 2. Các kỹ thuật biến đổi tâm tỷ cự và ứng dụng Lần lượt trình bày các kỹ thuật biến đổi dựa vào các tính chất của tâm tỷ cự trên mặt phẳng. Mỗi kỹ thuật được nêu thành các bước vận dụng, các ví dụ và các bài toán mẫu. Hình thành các kỹ năng " chọn tâm tỷ cự, biến đổi tâm tỷ cự, coi diện tích là tọa độ tâm tỷ cự,. . . " để giải các loại toán hình học phẳng: chứng minh, tính toán, tìm quỹ tích,. . . Chương 2 trình bày 4 mục sau: 2.1. Kỹ thuật chọn tâm tỷ cự. 2.2. Kỹ thuật diện tích hóa. 2.3. Kỹ thuật giao hoán và kết hợp. 2.4. Kỹ thuật quán tính. Chương 3. Các vấn đề liên quan
  • 8. Tải tài liệu tại sividoc.com Viết đề tài giá sinh viên – ZALO:0973.287.149-TEAMLUANVAN.COM Trình bày các bài toán liên quan đến tâm tỷ cự ở mức độ khó hơn, gồm hai nội dung:
  • 9. Tải tài liệu tại sividoc.com Viết đề tài giá sinh viên – ZALO:0973.287.149-TEAMLUANVAN.COM 3 3.1. Chứng minh một số định lý nổi tiếng của hình học sơ cấp. 3.2. Một số bài toán thi học sinh giỏi và thi Olympic. - Mặc dù đã rất cố gắng nhưng luận văn không tránh khỏi những hạn chế, khiếm khuyết. Tác giả rất mong sự góp ý, bổ sung của các đồng nghiệp và các thầy cô giáo nhằm làm cho kết quả nghiên cứu hoàn chỉnh và có ích hơn. Xin chân thành cảm ơn. Tác giả.
  • 10. Tải tài liệu tại sividoc.com Viết đề tài giá sinh viên – ZALO:0973.287.149-TEAMLUANVAN.COM 4 Chương 1 Tâm tỷ cự của hệ chất điểm Các khái niệm ở đây được xét trong mặt phẳng hoặc trong không gian. Thuật ngữ "barycentric" được nhiều tác giả dịch là "tâm tỷ cự" hoặc "khối tâm",. . . Thực ra sử dụng các từ này chỉ đúng nghĩa một phần bởi "barycentric" chỉ liên quan đến đoạn thẳng và các khái niệm quen thuộc trong cơ học. Đến nay "barycentric" đã được toán học hóa dựa vào khái niệm không gian véc tơ thì các cách Việt hóa như trên có những hạn chế nhất định. Trong luận văn này chúng tôi vẫn sử dụng chữ "tâm tỷ cự" do tính chất lịch sử của khái niệm và phù hợp với các tài liệu hiện hành (xem [1]). Các ký hiệu cũng được tham khảo và vận dụng vào việc trình bày cho thuận tiện nhất. 1.1 Hệ chất điểm và tâm tỷ cự Mệnh đề 1.1. Cho hai điểm A;B và hai số thực m1;m2 không đồng thời bằng 0. Khi đó i. Nếum + m = 0 thì không có Z sao cho ! ! ~ 1 2 m ZA + m ZB=0: 1 2 ii. Nếu m1 + m26= 0 thì tồn tại duy nhất điểm Z sao cho ! ! m1ZA + m2ZB =~0: Khi Z thỏa mãn đẳng thức trên thì với mọi điểm O ta luôn có: ! !! m OA + m OB OZ = 1 2 : m1 + m2
  • 11. Tải tài liệu tại sividoc.com Viết đề tài giá sinh viên – ZALO:0973.287.149-TEAMLUANVAN.COM 5 Chứng minh. ! ! ! ! ! i. Ta có ! ~ () ~ () 1 2 1 2 1 2 ZB = 0 ZA (ZA+AB) = 0 )ZA+ ! m ZA+m m +m (m +m ~ Nếu thì không có . = 0: m + m = 0 Z m AB 2 2 1 m2 m + m = AZ = AB; chứng tỏ Z xác ii. Nếu 1 2 6 0 thì đẳng thức trên là ! ! (m1 + m2) định và duy nhất. m OA + m OB ta có: Với O tùy ý, xen điểm Z vào ! ! ! ! ! ! 1 ! 2 ! m OA + m OB = m (OZ + ZA) + m (OZ + ZB) 1 2 1 ! ! 2 ! = (m ! + m )OZ + (m ZA + m ZB) = (m + m )OZ. 1 2 1 2 1 2 Mệnh đề 1.2. Cho ba điểm A;B;C và ba số thực m1;m2;m3 không đồng thời bằng O. Khi đó, i. Nếu m1 + m2 + m3 = 0 thì không có Z sao cho ! + ! + ! =~: m1ZA m2ZB m3ZC 0 ii. Nếu m1 + m2 + m3 6= 0 thì tồn tại duy nhất điểm Z sao cho ! + ! + ! =~ m1ZA m2ZB m3ZC 0 Khi Z thỏa mãn đẳng thức trên thì với mọi điểm O ta luôn có: ! ! ! ! m OA + m OB + m ZC OZ = 1 2 3 : (1.1) m1 + m2 + m3 Chứng minh. Chứng minh tương tự mệnh đề 1.1. Nhận xét 1.1. ! ! Trong trường hợp m1 = m2 = m3 6= 0 thì đẵng thức (1.1) trở thành ZA + ZB + ! ~ () ZC=0 Z G trọng tâm tam giác ABC. Mệnh đề 1.3. Cho n điểm A1;A2; ::;An và n số thực m1;m2; :::;mn không đồng thời bằng O . Khi đó i. Nếu m1 + m2 + ::: + mn = 0 thì không có Z sao cho ! + ! +:::+ ! =~:
  • 12. Tải tài liệu tại sividoc.com Viết đề tài giá sinh viên – ZALO:0973.287.149-TEAMLUANVAN.COM m1ZA1 m2ZA2mnZAn 0
  • 13. Tải tài liệu tại sividoc.com Viết đề tài giá sinh viên – ZALO:0973.287.149-TEAMLUANVAN.COM 6 ii. Nếu m1 + m2 + ::: + mn 6= 0 thì tồn tại duy nhất điểm Z sao cho ! + ! +:::+ ! =~: m1ZA1 m2ZA2mnZAn 0 Khi có Z thỏa mãn (1.2) thì với mọi điểm O ta luôn có : ! m ! 1OA1 + m ! 2OA2 + ::: + m ! nOAn OZ = : m1 + m2 + ::: + mn (1.2) (1.3) m + m + ::: + m = ~ Chứng minh. i. Ta có ! 1ZA1 ! 2ZA2 ! nZAn 0 () m + m ( + ) + ::: + m ( + ) = ~ ! 1ZA1 ! 2ZA1 ! A1A2 ! nZA1 ! A1An 0 () ( 1 +:::+ n ) 1 + m 2 1 2 + ::: + m n 1 n = ~ : Từ đây ta có: m m!ZA !A A !A A 0 Nếu m1 + m2 + ::: + mn = 0 thì không có Z. với A Z = ii. Nếu m + m + ::: + m = 0 thì đẳng thức trên tương đương 12 n 6 ! 1 m + m + ::: + m ! 2A1A2 ! 3A1A3 ! nA1An ; chứng tỏ Z xác định và duy nhất. Với mọi (m1 + m2 + ::: + mn) điểm O, từ m ! 1OA1 + m ! 2OA2 + ::: + m ! nOAn ta có: m ! 1OA1 + m ! 2OA2 + ::: + m ! nOAn = ! ! = m ! ! ! ! ) + ::: + m ) (OZ + ZA ) + m (OZ + ZA (OZ + ZA 1 1 2 2 n n = (m + m ! ! ! + ::: + m )OZ + (m ZA + m ZA 2 1 2 n 1 1 2 = (m + m ! + ::: + m )OZ. Từ đó suy ra (1.3). 1 2 n ! + ::: + mnZAn) Định nghĩa 1.1. Giả sử P là tập hợp điểm trên mặt phẳng, tích Decasterte R P được gọi là "một hệ chất điểm" trong mặt phẳng. Mỗi chất điểm có hai thành phần, được viết là mA hoặc m:A hoặc (m;A) 2 R P, thành phần thứ nhất là số, thành phần thứ hai là điểm. Định nghĩa 1.2. Điểm Z xác định duy nhất từ hệ thức (1.3) với các số thực m1;m2; :::;mn thoả mãn điều kiện m1 + m2 + ::: + m3 6= 0 được gọi là tâm tỷ cự của hệ chất điểm fmiAig n i=1, với S n i=1mi 6= 0 và viết Z [m1A1;m2A2; :::;mnAn] hay ký hiệu tắt là Z [miAi]1 i n: Ký hiệu I [1A;1B], tức I là tâm tỷ cự của hệ chất điểm f1A;1Bg, đó là trung điểm của đoạn AB. Khi G là trọng tâm tam giác ABC, ta viết G [1A;1B;1C] (trọng tâm là tâm tỷ cự của 3 đỉnh tam giác).
  • 14. Tải tài liệu tại sividoc.com Viết đề tài giá sinh viên – ZALO:0973.287.149-TEAMLUANVAN.COM 7 1.2 Các tính chất cơ bản của tâm tỷ cự Trước hết ta xét 3 tính chất sau của tâm tỷ cự. Tính chất 1.1. Mỗi hệ hữu hạn các chất điểm fm1A1; :::;mkAng với m1 + :: + mn 6= 0 đều xác định duy nhất tâm tỷ cự của hệ sai khác một hằng số khác không, tức là tồn tại duy nhất Z sao cho Z [miAi]1 i n [kmiAi]1 i n;k 6= 0: Chứng minh. Thật vậy, chọn O tùy ý khi đó Z xác định duy nhất theo đẳng thức véc tơ (1.3). Ta có thể viết (1.3) ở dạng sau với k 6= 0: ! km ! 1OA1 + km ! 2OA2 + ::: + km ! nOAn OZ = : km1 + km2 + ::: + kmn Tính chất 1.2. (Quy tắc Archimedes) Tâm tỷ cự của hệ hai chất điểm fm1A1;m2A2g nằm trên đoạn thẳng (hoặc đường thẳng) nối hai điểm A1;A2. Vị trí tâm tỷ cự xác định theo "quy tắc cân bằng của đòn bẩy" của Archimedes (gọi là quy tắc Archimedes): jm1jd1 = jm2jd2. Hình 1.1: Quy tắc Archimedes Chứng minh. Giả sử Z là tâm tỷ cự của hệ hai chất điểm fm1A1;m2A2g. Khi đó: ! 1ZA1 ! 2ZA2 0 () ! 1ZA1 = ! 2ZA2 : Nếu 1 ;m 2 cùng dấu (có m + m = ~ m m m thể coi m ;m > 0) ta thấy các véc tơ ZA ;ZA ngược hướng nên điểm Z nằm 1 2 ! ! 2 1 trên đoạn thẳng A A, hơn nữa, m ! ZA 1 j= m ! ZA 2 j; tức là m d 1 = m d . Nếu m ;m 1 2 1 j 2 j 1 2 2 tơ ZA ;ZA cùng hướng nên điểm Z nằm ngoài 1 2 trái dấu ta thấy các véc ! ! 2 j 1jj 1 j 2jj 2 j j 1 j j 2 j 2 đoạn thẳng 1 2 1 j 1 . A A, ngoài ra,m ! ZA = m ! ZA ; tức làm d = m d
  • 15. Tải tài liệu tại sividoc.com Viết đề tài giá sinh viên – ZALO:0973.287.149-TEAMLUANVAN.COM Từ đây ta cũng thấy tâm tỷ cự của hệ hai điểm ở gần điểm có "trọng lượng" lớn hơn trong hai "trọng lượng" của hai chất điểm.
  • 16. Tải tài liệu tại sividoc.com Viết đề tài giá sinh viên – ZALO:0973.287.149-TEAMLUANVAN.COM 8 Hệ quả 1.1. Điểm M chia đoạn thẳng AB theo tỷ số b khi và chỉ khi M a [aA;b B]. Tính chất 1.3. (Tính chất kết hợp) Giả sử ta lấy ra k chất điểm fm1A1;m2A2; :::;mkAkg trong hệ n chất điểm fm1A1;m2A2; :::;mnAng và gọi C là tâm tỷ cự của hệ k chất điểm đó. Khi đó hệ chất điểm ban đầu có cùng tâm tỷ cự với hệ chất điểm là: f(m1 + m2 + ::: + mk)C;mk+1Ak+1; :::;mnAng: Chứng minh. Gọi Z là tâm tỷ cự của hệ n chất điểm, ta có: ! +:::+ ! + ! +:::+ ! =~: m 1 ZA 1 m k ZA k m k+1 ZA k+1 m n ZA n 0 Vì C là tâm tỷ cự của hệ fm1A1;m2A2; :::;mkAkg nên ta có: ! m + ::: + m ZC = ! 1ZA1 ! kZAk : m1+ ::: + mk Từ hai đẳng thức trên ta có ! ! ! ~ (m + ::: + m )ZC + m ZA + ::: + m ZA k+1 k+1 n = 0: 1 k n Đó là điều cần chứng minh. Từ tính chất kết hợp ta có các hệ quả hiển nhiên sau. Hệ quả 1.2. Nếu Z là tâm tỷ cự của hệ 3 điểm là đỉnh tam giác ABC. Khi đó đường thẳng AZ cắt cạnh BC ở điểm A 0 là tâm tỷ cự của hệ hai chất điểm đặt tại B và C. Hệ quả 1.3. Giả sử tại các đỉnh A,B,C của tam giác ABC theo thứ tự đặt các trọng lượng m1;m2;m3. Nếu B 0 là tâm tỷ cự của hệ fm1A;m3Cg; C 0 là tâm tỷ cự của hệ fm1A;m2Bg thì Z = BB 0 CC 0 là tâm tỷ cự của hệ ba điểm fm1A;m2B;m3Cg. Ký hiệu thu gon Trong công thức (1.3), O là điểm tùy ý trong không gian nên có thể quy ước bỏ điểm O và không dùng ký hiệu véc tơ. Như thế (1.3) được ký hiệu thu gọn là Z = m1A1 + m2A2 + ::: + mnAn (1.4) m1 + m2 + ::: + mn
  • 17. Tải tài liệu tại sividoc.com Viết đề tài giá sinh viên – ZALO:0973.287.149-TEAMLUANVAN.COM
  • 18. Tải tài liệu tại sividoc.com Viết đề tài giá sinh viên – ZALO:0973.287.149-TEAMLUANVAN.COM 9 hay (m1 + m2 + ::: + mn)Z = m1A1 + m2A2 + ::: + mnAn (1.5) Mỗi ký hiệu thu gọn nói trên khẳng định điểm Z là tâm tỷ cự của hệ chất điểm m1A1;m2A2; :::;mnAn. Khi viết P = 2A + 3B + 8C nghĩa là điểm P là tâm tỷ cự 13 của hệ điểm f2A;3B;8Cg. Từ tính chất kết hợp ta có thể sử dụng ký hiệu thu gọn linh hoạt hơn. Chẳng hạn, ký hiệu P = (2A+3B)+8C = 5D+8C =) P 2 CD 13 13 diễn tả bằng lời:" Giả sử P là tâm tỷ cự của hệ chất điểm f2A;3B;8Cg, nếu tâm tỷ cự của hệ f2A;3Bg là D thì điểm P là tâm tỷ cự của hệ f5D;8Cg". 1.3 Tâm tỷ cự và diện tích đại số 1.3.1 Diện tích đại số Để xét tính chất quan trọng của tâm tỷ cự liên quan đến diện tích, ta giới thiệu diện tích đại số thông qua khái niệm tích ngoài như sau. Định nghĩa 1.3. Trong mặt phẳng định hướng diện tích đại số của tam giác định hướng ABC, ký hiệu là ABC, là một số thực mà trị tuyệt đối của nó là diện tích (hình học) của tam giác đó với dấu + hay tùy theo tam giác ABC có hướng thuận hay nghịch: ABC = S(ABC): Trường hợp DABC suy biến thì ABC = 0 , C 2 AB: Định nghĩa 1.4. Tích ngoài (hay tích phản vô hướng) của hai véc tơ ~u;~v, ký ~ ~ hiệu là ~u ^~v là một số thực bằng 0 khi ~u = 0 hoặc~v = 0, bằng j~ujj~vjsin(~u;~v) khi ~ ~u =~v 6= 0;. Ta có nhận xét ngay: ! ^ ! ABC = 1 AB AC(từ đó tam giác ABC định hướng thuận khi và chỉ khi ! ! 2 AB^AC > 0). ~v ^~u = ~u ^~v; ~u và ~v cùng phương ()~u ^~v = 0: Dạng tọa độ. Xét mặt phẳng tọa độ Oxy, gọi ~ i; ~ j là hai véc tơ chỉ phương đơn
  • 19. Tải tài liệu tại sividoc.com Viết đề tài giá sinh viên – ZALO:0973.287.149-TEAMLUANVAN.COM vị của hai trục Ox, Oy, góc định hướng ( ~ i; ~ j) = p [mod2p]. Khi đó nếu ~u = 2
  • 20. Tải tài liệu tại sividoc.com Viết đề tài giá sinh viên – ZALO:0973.287.149-TEAMLUANVAN.COM 10 (u1;u2);~v = (v1;v2) thì u1 v1 ~u ^~v = u1v2 u2v1 = : u 2 v 2 ~ Thật vậy, khi ~u hay ~v bằng 0 thì rõ ràng u1v2 u2v1 = 0. ~ Khi ~u và ~v khác 0 thì ta có: sin(~u;~v) = sin(( ~ i;~v) ( ~ i;~u)) = sin( ~ i;~v)cos( ~ i;~u) cos( ~ i;~v)sin( ~ i;~u) = v2 u1 v1 u2 nên ~u ^~v = u1v2 u2v1. ~v j ~u ~v j ~u j j Từ đó dễ kiểm tra được Hệ quả 1.4. ( ~) ^~ = ~ ^ ~ = (~ ^~); (~ + ~0 ) ^~ = ~ ^~ + ~0 ^~: ku v u kv k u v u u v u v u v Diện tích đại số ABC có các tính chất sau: a. Với mọi 4 điểm A, B, C, M ta có hệ thức Chasles: ABM + BCM + CAM = ABC: b. Cho DABC. Với 2 điểm B 0 ;C 0 2 BC ta có: BC ABC B0C0 = AB0C0 c. Cho tam giác ABC và điểm O. Giả sử các đường thẳng AO;BC cắt nhau tại M (khác B,C). Ta có: MB = OBA MC OCA d. Cho tam giác ABC và M thuộc đường thẳng BC. Ta có: MB = MBA MC MCA Chứng minh. ! !!!!! a. Thật vậy, AB ^AM + BC ^BM + CA ^CM ! !!!!!!! = AB^(AC+CM)+BC^(BC+CM)+CA^(CM ! !!!!! !! = AB^(AC+(AB+BC+CA)^CM = AB^(AC: Các đẳng thức b., c., d. là hiển nhiên.
  • 21. Tải tài liệu tại sividoc.com Viết đề tài giá sinh viên – ZALO:0973.287.149-TEAMLUANVAN.COM
  • 22. Tải tài liệu tại sividoc.com Viết đề tài giá sinh viên – ZALO:0973.287.149-TEAMLUANVAN.COM 11 Tính chất sau đặc biệt quan trọng khi ta chuyển sang xét tọa độ tâm tỷ cự của một điểm đối với tam giác ABC (không suy biến). Chính vì tính chất này mà tọa độ tỷ cự còn được gọi là "tọa độ diện tích". Tính chất 1.4. DABC và điểm M tùy ý trong mặt phẳng. Khi đó (1.6) M [MBC:A;MCA:B;MAB:C]: Chứng minh. Hình 1.2: Tọa độ diện tích Trong các đường thẳng AM,BM,CM phải có ít nhất một đường thẳng không song song với BC, CA, AB theo thứ tự đó vì chẳng hạn có BM k AC;CM k AB thì tứ giác ABMC là hình bình hành nên AM phải cắt BC. Bởi vậy, ta có thể coi AM cắt BC ở điểm A 0 . ! ! ! Bước 1. Ta có BCMA0 = (A0 C A0 B) = A0 CMA0 A0 BMA0 !MB BA0 !MC CA0 A0C A0B = ( ! + ) ( !+ ) ! ! ! = A0 ! 0 0 0 ! 0 0 0 ! 0 ! 0 0 C C:MB+BA:A C A B:MC CA :A B. = A C:MB A B:MC+BA :A ! ! A0 C A0 B CA0:A0B. Từ đây ta suy ra: MA0 = ! MB ! MC. BC BC Bước 2. Theo tính chất về độ dài đại số và diện tích đại số: A 0 C = AA 0 C = MA 0 C = MCA . Suy raA 0 C = A 0 C A 0 B ABA 0 MBA 0 MAB BC A 0 C A 0 B = MCA . Tương tự, A0 B = MAB . BC MCA MAB ! MCA + MAB Thay vào đẳng thức tính MA 0 ở trên ta được: ! MCA ! :MB + MAB ! :MC: MA0 = MCA + MAB MCA + MAB Bước 3. Vì MA0 = MCA0 =MA0 B = MCA0 + MA0 B nên
  • 23. Tải tài liệu tại sividoc.com Viết đề tài giá sinh viên – ZALO:0973.287.149-TEAMLUANVAN.COM MA MCA MAB MCA + MAB
  • 24. Tải tài liệu tại sividoc.com Viết đề tài giá sinh viên – ZALO:0973.287.149-TEAMLUANVAN.COM 12 ! MBC ! MCA ! MAB ! MA0 = MA = MB + MC. Suy ra MCA + MAB MCA + MAB MCA + MAB ~ tương đương (1.6). MBC:MA + MCA:MB + MAB:MC = 0 ! ! ! Từ tính chất này ta suy ra các hệ quả. Hệ quả 1.5. Nếu M nằm trong miền tam giác ABC thì M [S (MBC) A;S (MCA) B;S (MAB) C]: (1.7) (M là tâm tỷ cự của hệ chất điểm fS(MBC)A;S(MCA)B;S(MAB)Cg) Hệ quả 1.6. Nếu M ở ngoài miền tam giác ABC và ở trong BACd thì M [ S (MBC) A;S (MCA) B;S (MAB) C]: (1.8) (M là tâm tỷ cự của hệ chất điểm f S(MBC)A;S(MCA)B;S(MAB)Cg). Tương tự như thế nếu M ở ngoài tam giác nhưng ở trong [ hay ở trong [ . ACB CBA 1.3.2 Tọa độ tỷ cự trong mặt phẳng Ta cố định tam giác ABC, gọi nó là tam giác cơ sở (không suy biến). Khi đó theo 1.6, mọi điểm M trong mặt phẳng đều có M [MBC:A;MCA:B;MAB:C]. Từ đó ta định nghĩa Định nghĩa 1.5. Giả sử ABC là tam giác cơ sở. Tọa độ tỷ cự của điểm M đối với tam giác ABC là bộ ba số (x : y : z) sao cho x : y : z = MBC : MCA : MAB Ta có tọa độ tỷ cự của một số điểm đặc biệt trong tam giác ABC: Cho 4 ABC, Gọi G, I, O, H, Oa lần lượt là trọng tâm, tâm đường tròn nội tiếp, tâm đường tròn ngoại tiếp, trực tâm, tâm đường tròn bàng tiếp trong góc A. Khi đó ta có: a. G(1A: 1B: 1C), b. I(aA: bB: cC), c. O [sin2A.A, sin2B.B, sin2C.C] hay dưới dạng: O [a 2 (b 2 + c 2 a 2 )A,b 2 (c 2 + a 2 b 2 )B,c 2 (a 2 + b 2 c 2 )C].
  • 25. Tải tài liệu tại sividoc.com Viết đề tài giá sinh viên – ZALO:0973.287.149-TEAMLUANVAN.COM 13 d.O a [-S (OaBC) A,S (OaCA) B,S (OaAB) C] [-aA, bB, cC] e.H [tan A.A, tan B.B, tanC.C] hay dưới dạng: H [ 1 A, 1 B, 1 C]. b2 + c2 a2 c2 + a2 b2 a2 + b2 c2 (Xem chứng minh trong ví dụ 2.9) Khi M có tọa độ tỷ cự là (x : y : z) mà x + y + z 6= 0 thì ta nói (x : y : z) là tọa độ tỷ cự tuyệt đối của M, nếu x + y + z = 1 thì (x : y : z) được gọi là tọa độ tỷ cự chuẩn của M. Từ tính chất của tâm tỷ cự ta rút ra: nếu M và N có tọa độ tỷ cự tuyệt đối, điểm X chia đoạn MN theo tỷ số MX : XN = m : n sẽ có tọa độ tỷ cự tuyệt đối là n:M + m:N . Tuy nhiên để thuận tiện trong tính toán ta sẽ tính tọa độ m + n của X theo cách tương đương như sau: Nếu M = (u : v : w);N = (u0 : v0 : w0 ) mà u + v + w = u0 + v0 + w0 thì điểm X sẽ có tọa độ tỷ cự là (nu + mu0 : nv + mv0 : nw + mw 0 ). Các sự kiện cơ bản trong tọa độ tỷ cự (a) Phương trình đường thẳng đi qua 2 điểm P, Q: Khi P(a1 : b1 : c1); Q(a2 : b2 : c2) thì PQ được xác định bởi Dax + Dby + Dcz = 0; trong đó, Da = c 1 c 2; Db b b 1 2 = a1 a2; Dc = b 1 b 2; hoặc định thức cấp 3: c c a 1 a 2 1 2 x y z = 0: a1 b1 c1 a2 b2 c2 Từ đó suy ra rằng điểm R(x 0 : y : z 0 ) nằm trên PQ khi và chỉ khi 0 x0 y0 z0 = 0: a1 b1 c1 a2 b2 c2 (b) Giao điểm của 2 đường thẳng a 1 x + by + c 1 z = 0 và a 2 x + b 2 y + c 2 z = 0 1
  • 26. Tải tài liệu tại sividoc.com Viết đề tài giá sinh viên – ZALO:0973.287.149-TEAMLUANVAN.COM xác định bởi công thức M(b1c2 b2c1 : c1a2 c2a1 : a1b2 a2b1): Từ đó suy ra
  • 27. Tải tài liệu tại sividoc.com Viết đề tài giá sinh viên – ZALO:0973.287.149-TEAMLUANVAN.COM 14 điều kiện để ba đường thẳng có phương trình aix + biy + ciz = 0;i = 1;2;3 đồng quy là a 1 b 1 c 1 = 0: a2 b2 c2 a 3 b 3 c 3 (c) Điều kiện để hai đường thẳng aix + biy + ciz = 0;i = 1;2 song song: (a1 b1 : b1 c1 : c1 a1) = (a2 b2 : b2 c2 : c2 a2) (d) Trung điểm của đoạn thẳng BC là điểm M(0 : 1 : 1); Tiếp điểm với cạnh BC của đường tròn nội tiếp DABC là D(0 : 1 : 1 ) và tương tự với các p b p c cạnh còn lại. (d) Phương trình đường tròn ngoại tiếp tam giác cơ sở ABC: a 2 yz + b 2 zx + c 2 xy = 0: (e)Hai điểm P(x1 : y1 : z1);Q(x2 : y2 : z2) liên hợp đẳng giác với nhau khi và chỉ khi tồn tại số k 2 R để có x1x2 = ka 2 ;y1y2 = kb 2 :z1z2 = kc 2 : Diện tích tam giác Lấy ABC là tam giác cơ sở. Giả sử P(p1 : p2 : p3);Q(q1 : q2 : q3);R(r1 : r2 : r3) có tọa độ tỷ cự chuẩn hóa theo ABC. Khi đó = p 1 q 1 r 1 : : (1.9) PQR p2 q2 r2 ABC p3 q3 r3 Chứng minh. Với mọi điểm O, ! ! ! ! ! ! + OP = p OA + p OB + pOC; OQ = q OA ! ! 1 2 3 1 q OB + q OC. Từ đó, 2 3 ! = (q 1 1 ! 2 2! + (q 3 3! PQ p )OA + (q p )OB p)OC: Lấy O PQ = (q p )CA + (q p )CB. Tương tự, PR = (r C ta có:! 1 1 ! 2 ! ! 1 ! ! ! 2 ! p )CA + (r 2 p )CB. Ta nhận được: PQR = 1 PQ ^ PR = 1 1 2 ! ! 1 2 ! ! = (q 1 p 1 )(r 2 q )CA ^ CB + (q 2 p )(r 1 p)CB ^ CA:
  • 28. Tải tài liệu tại sividoc.com Viết đề tài giá sinh viên – ZALO:0973.287.149-TEAMLUANVAN.COM 2 2 2 2 1
  • 29. Tải tài liệu tại sividoc.com Viết đề tài giá sinh viên – ZALO:0973.287.149-TEAMLUANVAN.COM 15 Vì ! ! ! ! ABC 2 ^ CB = 2 ^ CA nên ta tìm được: = 1 CA 1 CB PQR = ((q1 p1)(r2 p2) (q2 p2)(r1 p1))ABC = = [(p1q2 p2q1) + (q1r2 q2r1) + (r1 p2 r2 p1)]ABC: Cuối cùng sau khi nhân biểu thức (...)thứ nhất trong ngoặc vuông với r1 + r2 + r3 = 1, biểu thức (...)thứ hai trong ngoặc vuông với p1 + p2 + p3 = 1, biểu thức (...)thứ ba trong ngoặc vuông với q1 + q2 + q3 = 1 ta nhận được (1.9). 1.4 Công thức Lagrang và công thức Jacobi Định nghĩa 1.6. Giả sử có hệ chất điểm m1A1;m2A2; :::;mnAn. Mô men quán tính của hệ đối với điểm S được Euler định nghĩa là đại lượng n JS = m1jSA1j 2 + m2jSA2j 2 + ::: + mnjSAnj 2 = å mkjSAkj 2 (1.10) k=1 Chẳng hạn, ABCD là hình vuông với cạnh 2a, tại mỗi đỉnh đặt trọng lượng 1. Gọi S là trung điểm của AB, Z là tâm hình vuông. Ta hãy tính các mô men quán tính: JS và JZ. Ta có p JS = 1:jSAj 2 + 1:jSBj 2 + 1:jSCj 2 + 1:jSDj 2 = 2a 2 + 2(a 5) 2 = 12a 2 . p JZ = 1:jZAj 2 + 1:jZBj 2 + 1:jZCj 2 + 1:jZDj 2 = 4(a 2) 2 = 8a 2 . Công thức sau đây được nhà toán học Pháp Lagrang tìm ra thể hiện mối liên hệ giữa mô men quán tính JS của hệ chất điểm đối với điểm S tùy ý và mô men quán tính JZ cũng trong hệ chất điểm đó đối với tâm tỷ cự Z của nó. Tính chất 1.5. Giả sử Z[m1A1;m2A2; :::;mnAn]. Khi đó đối với điểm S tùy ý ta có: JS = JZ + mjSZj 2 trong đó,m = m1 + m2 + ::: + mn: (1.11) n ! n ! ! Chứng minh. Ta có, J = å å ) 2 = S m SA 2 = m (SZ + ZA k k k k k=1 k=1 n ! n ! ! n ! å å å ): m ZA 2 +( m )SZ2 + 2SZ: m (ZA k k k k k k=1 k=1 k=1 n ( ) = ~ ; ta thu được (1.11). Vì Z là tâm tỷ cự nên å m ! kZAk 0
  • 30. Tải tài liệu tại sividoc.com Viết đề tài giá sinh viên – ZALO:0973.287.149-TEAMLUANVAN.COM k=1
  • 31. Tải tài liệu tại sividoc.com Viết đề tài giá sinh viên – ZALO:0973.287.149-TEAMLUANVAN.COM 16 Một công thức khác được nhà toán học Đức K.G. Jacobi tìm ra biểu diễn mô men quán tính JZ của hệ chất điểm (với tổng trọng lương khác không) đối với tâm tỷ cự Z của nó qua trọng lượng của các chất điểm và khoảng cách AiAj giữa chúng. Tính chất 1.6. Giả sử Z[m1A1;m2A2; :::;mnAn]. Khi đó, JZ = 1 å mim jjAiAjj2 : (1.12) m 1 i< j n Chứng minh. Xét n = 3 (trường hợp tổng quát làm tương tự). Ta có công thức Lagrang: JA1 = JZ + mjA1Zj 2 . Từ đó, sau khi nhân với m1 nhận được m1m2jA1A2j 2 + m1m3jA1A3j 2 = m1JZ + mm1jA1Zj 2 Tương tự ta có (mô men quán tính đối với các điểm A2;A3): m2m1jA2A1j2 + m2m3jA2A3j2 = m2JZ + mm2jA2Zj2 m3m1jA3A1j2 + m3m2jA3A2j2 = m3JZ + mm3jA3Zj2 : Cộng vế với vế các đẳng thức trên ta nhận được tổng các tích mim jjAiAjj 2 với i 6= j. Mỗi tích như thế ta gặp 2 lần. Nếu ta chỉ viết các số hạng với i < j thì ta nhận được 2 å mim jjAiAjj2 = mJZ + mJZ 1 i< j 3 tương đương với công thức (1.12) với n = 3: Kết hợp các công thức 1.11 và 1.12 ta có công thức tính khoảng cách từ một điểm bất kỳ đến tâm tỷ cự của hệ điểm. n Hệ quả 1.7. Giả sử Z [m1A1;m2A2; :::;mnAn] với å mi = 1. Khi đó đối với điểm S tùy ý ta có công thức khoảng cách i=1 n å mimjAiAj 2 SZ 2 = JS JZ = å miSAi 2 i=1 1 i<j n
  • 32. Tải tài liệu tại sividoc.com Viết đề tài giá sinh viên – ZALO:0973.287.149-TEAMLUANVAN.COM 17 3 Chẳng hạn với Z [m1A1;m2A2;m3A3]; å mi = 1 thì SZ 2 = i=1 m1SA1 2 + m2SA2 2 + m3SA3 2 (m1m2A1A2 2 + m1m3A1A3 2 + m2m3A2A3 2 ): Ví dụ 1.1. Cho tứ giác ABCD, cạnh và đường chéo có các độ dài a;b;c;d;e; f . Tại mỗi đỉnh của tứ giác đặt trọng lượng 1. Tính mô men quán tính của hệ chất điểm này đối với trung điểm Z của đoạn thẳng PQ nối trung điểm các đường chéo. Lời giải. Điểm Z là tâm tỷ cự của hệ chất điểm đang xét. Theo công thức Jacobi ta có: JZ = 1 (a2 + b2 + c2 + d2 + e2 + f2 ): 4 Đó chính là mô men quán tính của hệ chất điểm này đối với trung điểm Z của đoạn thẳng PQ. Nội dung của chương một là nhắc lại khái niêm hệ chất điểm, tâm tỷ cự của hệ chất điểm và hệ thống các tính chất cần thiết của tâm tỷ cự. Các phép chứng minh đều dựa vào công cụ véc tơ. Trong chương tiếp theo của luận văn ta sẽ xây dựng các kỹ thuật dựa vào các tính chất đó và áp dụng khái niệm mô men quán tính vào giải các bài toán hình học. Đây cũng là những kỹ thuật khá điển hình khi khai thác khái niệm tâm tỷ cự.
  • 33. Tải tài liệu tại sividoc.com Viết đề tài giá sinh viên – ZALO:0973.287.149-TEAMLUANVAN.COM 18 Chương 2 Các kỹ thuật biến đổi tâm tỷ cự và ứng dụng Từ các tính chất cơ bản ta nêu ra các kỹ thuật được sử dụng thường xuyên và có hiệu quả khi giải các bài toán hình học. Ta lần lượt trình bày các kỹ thuật biến đổi dựa vào các tính chất của tâm tỷ cự trên mặt phẳng. Mỗi kỹ thuật được nêu thành các bước vận dụng, các ví dụ và các bài toán mẫu. 2.1 Kỹ thuật chọn tâm tỷ cự Kỹ thuật chọn tâm tỷ cự dựa vào tính chất 1.1 và 1.2. Chọn tâm tỷ cự thích hợp đi liền với đặt trọng lượng tương ứng làm cho lời giải bài toán đơn giản hơn. Ở đây lưu ý đến quy tắc Archimedes (tính chất 1.2). Ví dụ 2.1. Trên cạnh AC của tam giác ABC lấy điểm M sao cho AM = 1 AC. 3 Trên phần kéo dài của tia CB lấy điểm N sao cho BN = BC. Đường thẳng MN cắt cạnh AB tại điểm P. Điểm P chia các đoạn thẳng AB và NM theo tỷ số nào? Lời giải. Rõ ràng tại N và C cần đặt các trọng lượng bằng nhau vì khi đó tâm tỷ cự của hai điểm này là B, bởi vậy, ta sẽ đặt tại mỗi điểm N và C các chất điểm 1N, 1C. Tiếp theo vì CM=2AM nên theo quy tắc Archimedes, M [1C, 2A]. Do đó Z [1N, 1C, 2A] [1N, 3M] (theo tính chất kết hợp), tức là Z 2 MN. Vậy Z = AB MN hay Z P. Vì Z [1N, 3M] [2A, 2B] nên theo quy tắc Archimedes ta có NP : PM = 3:1; AP:PM = 1:1. Chú ý. Ta có thể đưa ra cách giải bằng phương pháp véc tơ thông qua cách giải trên.
  • 34. Tải tài liệu tại sividoc.com Viết đề tài giá sinh viên – ZALO:0973.287.149-TEAMLUANVAN.COM 19 Tức là cách giải thuần túy véc tơ không nói gì đến ”chất điểm” hay ”tâm tỷ cự”: Cố định một điểm O nào đó, từ giả thiết ta có: ! ! !! ! ! 2MA + MC = 0 ; BC + BN = 0 và tương đương với các đẳng thức véc tơ sau: ! OM= ! ! ! ;OM= ! ! : 2OA + OC 2OA + OC 3 3 Hình 2.1: Chọn tâm tỷ cự ! ! ! ! 2OA+OC+ON Xét điểm Z cho bởi điều kiện OZ = ! ! 4 Ta có: ! 3OM + ON ) ! ! ! ) 2 4 OZ = ! 3ZM +ZN= 0 Z MN: ! ! ! ! ! ) ! ! ! 2 2OA 4 +ON) 4 OZ = + (OC 2OA + 2OB ZA+ZB= 0;Z AB. Do đó, Z = là giao điểm của các đoạn thẳng AB và MN, tức là Z trùng với P và bởi vậy, PN:PM = 3; PA = PB. Rõ ràng cách giải bằng véc tơ là hay và chặt chẽ (ý tưởng của nó xuất phát từ ! cách sử dụng tâm tỷ cự). Tuy nhiên xét đẳng thức OZ là không tự nhiên và khó giải thích vì sao lại chọn như vậy. Kỹ thuật chọn tâm tỷ cự đặc biệt có lợi khi gặp các bài toán tìm điểm hoặc tìm quỹ tích điểm thoa mãn đẳng thức véc tơ. Ta thường dùng kỹ thuật này để biến đổi một vế hoặc cả hai vế của một đẳng thức, tâm tỷ cự nếu tồn tại sẽ đóng vai trò điểm cố định. Ví dụ 2.2. Cho hình vuông ABCD. Tìm điểm M thỏa mãn ! ! ! ! ! MA +4MB +MC +4MD = 5AD:
  • 35. Tải tài liệu tại sividoc.com Viết đề tài giá sinh viên – ZALO:0973.287.149-TEAMLUANVAN.COM 20 Lời giải. Cách 1. Ký hiệu G là tâm hình vuông ABCD. Ta có: !MA!!!!!!!!! () +4MB + MC+4MD = 5AD () MA+MC+ 4MB + 4MD = 5AD 2 ! ! ! ! ! :4MG + 84MG = 5AD () MG = 2 1 AD: Vậy M là trung điểm của AB. ~ Cách 2. Chọn 1A ; 4B ; 1C ; 4D ]; GA + 4GB + GC + 4GD = 0: Khi đó [ G tức !! ! ! điều kiện của M tương đương với: ! ! ! () ! ! ! !!!! (GA GM) + 4(GB GM) + (GC GM) + 4(GD GM) = 5:AD 10GM = ! ! :OA + 4:OB + 1:OC + 4:OD 5AD: Cần xác định vị trí của G: Với mọi O, OG = 1 ! ! ! ! : ! IA 10 ! ! ! Từ đẳng thức đó chọn O I (tâm hình vuông) thì ta có: IG = + 4:IB IA 4:IB 10 ! ! = 0 : Tức G I là tâm hình vuông và M là trung điểm của AB. Cách 3. Chọn G [1A;4B;1C;4D] [[1A;1C];[4B;4D]] [2I;8I] [10I] I với I là tâm hình vuông. Cách 1 ngắn vì rất may ta chọn G đúng là tâm hình vuông. Cách 2 chính là áp dụng kỹ thuật chọn tâm tỷ cự của hệ chất điểm 1A, 4B, 1C, 4D. Cách 3 ngắn nhất do sử dụng kỹ thuật kết hợp một cách hợp lý. Các bài toán tiếp theo sẽ khẳng định ưu thế của kỹ thuật này. Ví dụ 2.3. Cho hình vuông ABCD. Tìm điểm M thỏa mãn ! ! ! ! ! MA +2MB +3MC +4MD = 5AD: Lời giải. Chọn G là tâm tỷ cự của hệ chất điểm f1A;2B;3C;4Dg. Khi đó: !!!!! GA + 2GB +3GC+4GD = 0 . Điều kiện đề bài MA + 2MB + 3MC + 4MD = 5AD ! ! ! ! ! ! () ! () ! ! ! ! ! ! ! ! (GA GM)+2(GB GM)+3(GC GM)+4(GD GM) = 5AD 10GM = ! ! ! 5AD () GM = 2 1 AD: ! ! ! ! Xác định vị trí của G: ! ! ! GA + 2GB + 3GC+ 4GD = O ! ! () ! ! ! ! ! OG = 1(OA + 2OB + 3OC + 4OD): OG)= O (OA OG)+2(OB OG)+3(OC OG)+4(OD () ! ! ! ! ! 10 Lấy O I ( tâm hình vuông) thì: 2 ! ! ! ! ! ! ! ! IG = 1 (OA + 2OB + 3OC + 4OD) = (OA + OB) = 2 OE với E là trung 5 5 10 điểm của AB.
  • 36. Tải tài liệu tại sividoc.com Viết đề tài giá sinh viên – ZALO:0973.287.149-TEAMLUANVAN.COM 21 Hình 2.2: Quĩ tích là đường tròn mãn IG = 2 IG; I là tâm hình Quỹ tích các điểm M là đường tròn tâm G (thỏa ! ! 1 5 vuông), bán kính GM = IE = AD. 2 Ví dụ 2.4. Cho tam giác ABC. Tìm quỹ tích điểm M sao cho ! ! ! ! ; a. MA + 2MB + 3MC = 0 ! ! ! ! ; b. MA + 2MB 3MC= 0 ! !!!!! c. 2 j MA + MB + MC j=j MA + 2MB + 3MC j : Lời giải. a. Vì 1 + 2 + 3 = 6 6= 0 nên tồn tại duy nhất điểm M là tâm tỷ cự của hệ chất ! ! ! f g OA +2OB + 3OC 6 điểm 1A ; 2B ; 3C : có: OM = : Ta Suy ra với mọi điểm O ta ! xác định vị trí của M bằng cách cho O vào một trong các vị trí đặc biệt A, B, ! 2OA +3AC = ! ! 1 1 C,... . Chẳng hạn O A, ta có AM = ! ! AB + AC: Như vậy, quỹ 6 3 2 ! ! tích điểm của M là một điểm, đỉnh của hình bình hành với hai cạnh = a 1AB; = ! 3 ! b 1 AB. 2 b. Vì 1 + 2 - 3 = 0 nên không có M thỏa mãn đẳng thức. Quỹ tích rỗng. c. Gọi G [1A;1B;1C] và I [1A;2B;3C]. Theo định nghĩa với mọi điểm M ta ! ! ! ! ! ! ! ! ! có: MG =1 (MA+MB+MC);MI =1 (MA + 2MB +3MC). Từ đó, 2 MA+ 6 ! MB! MC 3 ! ! ! ! j j ! MA j () j j () M cách đều hai + = + 2MB + 3MC 2:3 MG =6MI
  • 37. Tải tài liệu tại sividoc.com Viết đề tài giá sinh viên – ZALO:0973.287.149-TEAMLUANVAN.COM 22 điểm G và I. Ở đây G là trọng tâm 4ABC còn I xác định như điểm M trong phần a. Vậy quỹ tích các điểm M cần tìm là đường trung trực của đoạn IG. Ví dụ 2.5. Cho tam giác ABC và đường thẳng d. Tìm điểm M trên d sao cho ! jMA ! MB ! j đạt giá trị nhỏ nhất. + + 3MC Lời giải. Chọn G [1A;1B;3C] khi đó với mọi điểm M ta có: !MA!MB ! + + 3MC thiết MA + MB + 3MC =5MG = 5MG. : ! MG = 5 Theo giả j ! ! j !j ! ! ! j j Suy !j nhỏ nhất nếu và chỉ nếu MG nhỏ nhất, tức M là hình ra MA+MB + 3MC chiếu của G trên d. ! MA! MB ! Xác định vị trí của G: Từ đẳng thức MG = + 5+ 3MC ; chọn M C ! thì ! ! ! ! CA+CB 2 CG = = CE với E là trung điểm của AB. Từ đó ta dựng được G 5 5 và hình chiếu của G xuống d. Ví dụ 2.6. Cho tứ giác ABCD và số k 2 R. Tìm quỹ tích các điểm M thỏa: ! jMA ! ! j ! j j + 3MB MC = k MD : Lời giải. Chọn G [1A;3B; 1C], nghĩa là với mọi điểm O ta có ! :OA +3:OB 1:OC ! ! ! ! OG = 1!!! = 1:OA + 3:OB 1:OC =3OG; O. 3 ! ) j j ! j j 8 j j j 8 Với O M thì ! jMA ! +3MB MC =3MG, M. Bởi vậy đẳng thức đã cho 3 MG = k MD hay 3MG = kMD. Từ đó ta có tương đương với ! j j !j j Nếu k < 0, quỹ tích là 0/. Nếu k = 3, quỹ tích là đường thẳng trung trực của đoạn GD (G sẽ được xác định ngay sau đây). Nếu 0 < k 6= 3, đẳng thức đã cho tương đương với MG = k , quỹ tích là MD 3 đường tròn Aponolius dựng trên đoạn GD. ! ! 1 Điểm G được xác định: Cho O BG = BA BC CA. 3 = 3 B thì ! ! Ví dụ 2.7. Cho tam giác ABC. Tìm quỹ tích điểm M sao cho !MA ! ! j ! ! ! j j + 3MB 2MC = j MB : 2MA MC
  • 38. Tải tài liệu tại sividoc.com Viết đề tài giá sinh viên – ZALO:0973.287.149-TEAMLUANVAN.COM Lời giải. Ta phải biến đổi cả hai vế: Chọn I [1A, 3B, -2C], nghĩa là với mọi điểm O ta có
  • 39. Tải tài liệu tại sividoc.com Viết đề tài giá sinh viên – ZALO:0973.287.149-TEAMLUANVAN.COM 23 1 ! ! ! ! ! ! ! !OI = :OA +3:OB 2:OC ) j j j j 8 O. Cho O M 2 ! ! = OA + 3:OB 2:OC =2OI; ta có vế ! j ! j + 3MB 2MC j j trái MA =2MI. Hình 2.3: I là đỉnh thứ tư hình bình hành Vế phải ! ! ! ! ! ! ! ! ! ! ! 2MA MB MC = (MA MB)+(MA MC) = BA+CA = (AB+ ! AC) = 2AJ với J là trung điểm BC. Do đó điều kiên của M tương đương với 2! ! j () MI = AJ = const. Quỹ tích điểm M là đường tròn tâm I, j j j MI =2AJ bán kính AJ. Ngay từ khi chọn I là tâm tỷ cự ta đã biết I tồn tại và duy nhất vì 1 + 3 - 2 = 2 6= 0 (vế phải không làm thế này được). Vì I [1A, 3B, -2C] !!!! ()!! !! ! ()! ! = nên IA + 3IB 2IC = 0 (IA + IB) 2(IB IC) = 0 2IE +2CB !0 ! IE ! () = CB (E là trung điểm của AB). Như vậy I là đỉnh thứ tư của hình bình hành BCEI. Ví dụ 2.8. Cho 4 ABC. Tìm quỹ tích những điểm M sao cho ! ! ! ! (2MA 3MB)(MA +2MB) =~0: Lời giải. Gọi M [2A, -3B]; G 0 [1A, 2B] thì G và G 0 tồn tại duy nhất và cố định. Với mọi M trong mặt phẳng: ! ! ! ! ! ! ! ! 0 0 . Do = (1 = 3MG 2MA 3MB = (2 3)MG = MG; MA+ 2MB + 2)MG đó, ! ! ! ! () ! ! 0 () 0 = 0 0 =90 , (2MA 3MB)(MA + 2MB) = 0 3MG:MG GMG quỹ tích của M là đường tròn đường kính GG’.
  • 40. Tải tài liệu tại sividoc.com Viết đề tài giá sinh viên – ZALO:0973.287.149-TEAMLUANVAN.COM 24 ; G0 ! 2 M,GM= Xác định vị trí dựng điểm G : Vì G [2A, -3B] nên với mọi ! ! ! ! ! 0 ! 3 3MB 2MA. Chọn M A thì AG = 3AB. Tương tự, AG = AB. Quỹ tích điểm M là đường tròn đường kính GG0 với G;G0 được dựng theo các đẳng thức véc tơ: ! ! 0 2 ! ; ! . AG = 3AB AG = AB 3 Các bài toán khác Bài toán 2.1. Trên các cạnh tam giác ABC lấy các điểm L, M ! sao cho CL = CA; CM = CB,0< a <1;0< b < 1. Giả sử P = AM BL, tính tỷ số AP . a! ! b! a AM AP 1 HD. = . AM 1 ab Bài toán 2.2. Trên các cạnh AB, AC cua tam giác cân BAC lấy các điểm P, Q sao cho BP = nPA;AQ = nQC. Đường thẳng PQ chia đường cao AM của tam giác theo tỷ số nào? Bài toán 2.3. Trên các cạnh AC, BC cua tam giác ABC lấy các điểm L, M sao ! ! ! ! cho CL = aCA;CM = b CB. Giả sử P = AM BL. Tính các tỷ số AP : AM, BP : BL. Bài toán 2.4. Diện tích hình bình hành ABCD bằng 1. Điểm M chia cạnh BC theo tỷ số 3 : 5. Tính diện tích tứ giác CMPD, trong đó P = AM BD. Bài toán 2.5. Chứng minh định lý Ce 0 va bằng phương pháp chọn tâm tỷ cự. HD. Giả sử các đường thẳng AA1 và CC1 cắt nhau tại O và giả sử AC1 : C1B = p, BA1 : A1C = p. Ta phải chứng minh BB1 đi qua O () CB1 : B1A = 1 : pq. Đặt 1, p, pq tương ứng tại A, B và C. Ta có C1 [1A;pB]; A1 [pB;pqC]. Do đó tâm tỷ cự của 1A, pB, pqC là điểm O = CC1 AA1. Mặt khác O thuộc đoạn thẳng nối B với tâm tỷ cự của A và c. Nếu B1 [1A;pqC] thì AB1 : B1C = pq : 1. Còn lại phải chứng minh có một điểm trên AC chia AC theo tỷ số AB1 : B1C. Bài toán 2.6. Trên các cạnh AB, BC, CD và DA của tứ giác lồi ABCD đặt các điểm K, L, M, N tương ứng và thoa mãn: AK : KB = DM : MC = a;BL : LC = AN : ND = b . Gọi P = KL MN. Chứng minh rằng NP : PL = a; KP : PM = b . Bài toán 2.7. Trong tam giác ABC tìm điểm O sao cho với mọi đường thẳng đi
  • 41. Tải tài liệu tại sividoc.com Viết đề tài giá sinh viên – ZALO:0973.287.149-TEAMLUANVAN.COM 25 qua O giao với AB tại K và giao với BC tại L thỏa mãn p AK + q CL = 1; KB LB trong đó p;q là các số dương cho trước. Bài toán 2.8. Trên các cạnh BC, CA, AB của tam giác ABC đặt các điểm A1;B1;C1 tương ứng và thỏa mãn: các đường thẳng CC1;AA1;BB1 đồng quy tai O. Chứng minh rằng: a. CO = CA 1 + CB 1: OC1 A1B B1A b. AO:BO:CO AO + BO + CO +2>8: OA1 OB1 OC1 OA1 OB1 OC1 Bài toán 2.9. Trên các cạnh BC, CA, AB của tam giác ABC đặt các điểm A1;B1;C1 tương ứng và thỏa mãn: BA1 = CB1 = AC1 . Chứng minh rằng tâm A1C B1A C1B tỷ cự của tam giác ABC và tam giác A1B1C1 trùng nhau. Bài toán 2.10. Trên các cạnh BC, CA, AB của tam giác ABC đặt các điểm A1;B1;C1 tương ứng sao cho các đường thẳng B1C1;BB1;CC1 cắt đường thẳng AA1 tương ứng tại các điểm M, P, Q. Chứng minh rằng: A1M A1P A1Q a. = + . MA PA QA BC1 CB1 b. Nếu P Q thì MC1 : MB1 = : : AB AC Bài toán 2.11. Lấy P;P1 trên đường thẳng BA, Q;Q1 trên đường thẳng AC. Đường thẳng nối A với giao của các đường thẳng PQ và P1Q1 cắt đường thẳng BC tại D. Chứng minh rằng BP BP1 BD = PA P1A CD CQ CQ1 QA Q1A Bài toán 2.12. Cho tam giác ABC, hai điểm M và N thay đổi thỏa ! MN = ! ! ! mãn 4MA + MB 2MC. Chứng minh rằng đường thẳng MN luôn đi qua một điểm cố định.
  • 42. Tải tài liệu tại sividoc.com Viết đề tài giá sinh viên – ZALO:0973.287.149-TEAMLUANVAN.COM 26 Bài toán 2.13. Cho tư giác ABCD. Tìm quỹ tích điểm M sao cho: a.! MA! MB! MC ! MD j ! MA! MB ! j j + + + = j + 2MC ; !!!!! b. MA 2MB + 3MC 4MD= AB; c. ! ! ! = ! 2MA + 3MB MD 2AC. j ! Bài toán 2.14. Cho 4 ABC . Tìm điểm M để ! ! j đạt giá trị 3MA+ 2MB MC bé nhất. Bài toán 2.15. Cho 4 ABC và số k 2 R. Tìm quỹ tích các điểm M thỏa mãn: ! ! ! ! MA + 2MB + kMC = 0 : Bài toán 2.16. Cho hình bình hành ABCD. Tìm quỹ tích những điểm M sao cho: ! ! ! ! a.jMA + MBj = jMC + MDj, ! !!!! b. j2MA MB MCj = jMC + 2MDj. Bài toán 2.17. Cho hình vuông cạnh a. Tìm quỹ tích những điểm M sao cho: ! !!!2 a. MA:MB + MB:MD = a , b. MA 2 +MB 2 +MC 2 = 3MD 2 : 2.2 Kỹ thuật diện tích hóa và tọa độ hóa. Kỹ thuật diện tích hóa là áp dụng trực tiếp các hệ quả của tính chất 1.4 với mục đích chuyển việc tính tâm tỷ cự trong mặt phẳng về tính diện tích đại số các tam giác. Áp dụng tọa độ tỷ cự trong mặt phẳng cho phép giải khá nhiều các bài toán khó bằng tính toán và biến đổi đại số. Vì khuôn khổ luận văn có hạn nên chúng tôi chỉ xét các ví dụ điển hình. Vấn đề này sẽ được đề cập sâu hơn vào một dịp khác. Ví dụ 2.9. Cho DABC. Gọi G, I, O, H, Oa lần lượt là trọng tâm, tâm đường tròn nội tiếp, tâm đường tròn ngoại tiếp, trực tâm, tâm đường tròn bàng tiếp trong góc A. Khi đó ta có tọa độ tỷ cự đối với DABC: a. G(1.A: 1.B:1.C),
  • 43. Tải tài liệu tại sividoc.com Viết đề tài giá sinh viên – ZALO:0973.287.149-TEAMLUANVAN.COM b. I (a.A: b.B: cC),
  • 44. Tải tài liệu tại sividoc.com Viết đề tài giá sinh viên – ZALO:0973.287.149-TEAMLUANVAN.COM 27 c. O(sin2A.A: sin2B.B: sin2C.C) hay dưới dạng: O(a 2 (b 2 + c 2 a 2 ).A: b 2 (c 2 + a 2 b 2 ).B: c 2 (a 2 + b 2 c 2 ).C) d.O a (-S (OaBC) .A:S (OaCA) .B:S (OaAB) .C) (-a.A: b.B: c.C), e. H(tan A.A: tan B.B: tanC.C) hay dưới dạng: H( 1 .A: 1 .B: 1 .C). b 2 + c 2 a 2 c 2 + a 2 b 2 a 2 + b 2 c 2 Chứng minh. a. Vì G ở trong tam giác và S(GBC) =S(GCA) =S(GAB) nên G [1A,1B,1C] b. Ta có I nằm trong tam giác và S(IBC) = 1a.r; S(ICA) = 1b.r; S(IAB) = 1c.r nên 2 1 1 1 2 2 I [ ar.A, br.B, cr.C] [aA, bB, cC]. 2 2 2 c. Nếu Ab 90 0 thì O và A khác phía nhau đối với BC. Khi đó Sa =S(OBC) = 1 OB.OC.sin BOC = 1 R 2 sin 2A. 2 2 Nếu A > 90 0 thì A và O khác phía nhau đối với BC. Khi đó, Sa = S(OBC) = 1OB: b OC:sin(360 0 2A = 1R 2 sin 2A. Tương tự, Sb = 1R 2 sin 2B;Sc =1R 2 sin 2C: 2 2 2 2 Bởi vậy, O [sin 2A:A;sin 2B:B;sin 2C:C]. Vì sin 2A = 2 sin A cos A b 2 + c 2 a 2 c 2 + a 2 b 2 b 2 + a 2 c 2 = 2a: ; sin 2B = 2b: ; sin 2C = 2c: nên ta 2bc 2ca 2ab cũng có O [a 2 (b 2 + c 2 a 2 ).A,b 2 (c 2 + a 2 b 2 ):B;c 2 (a 2 + b 2 c 2 ).C]. d. Áp dụng (1.7). e. Cách 1. Hình 2.4: Trực tâm H
  • 45. Tải tài liệu tại sividoc.com Viết đề tài giá sinh viên – ZALO:0973.287.149-TEAMLUANVAN.COM 28 Khi ABC là tam giác nhọn (trực tâm H ở miền trong tam giác). Ta cần chứng minh: tan A ! ! ! ~ Trước hết ta vẽ hình bình hành = 0: :HA + tan B:HB + tan C:HC HB 0 A1C AB 0 CA 0 . Trong tam giác BB 0 C ta có HA1 k B 0 C, suy ra: = . HB A1B Lại có A1C = AA1 cot C; A1B = AA1 cotB. Do đó, HB 0 = A1C =AA1 cot C = HB tan B HB ! ! A1B AA1 cot B ! 0 tan B ! 0 ! 0 tan C ) tan C = = HB (Vì HB;HB ngược chiều nhau). Tương tự, HA = ! ! ! ! ! ! ! tan A 0 tan A tan B () tan C tan C tan C HA. Từ đó: HA+HB = HA HB=HC tan A.HA + tan B ! . + ! . ~ HB tan C HC = 0: + Khi tam giác ABC là tam giác tù. Chứng minh hoàn toàn tương tự với chú ý rằng ! ! 0 cùng chiều. HB và HB Cách 2. (dùng diện tích) + Trường hợp 4 ABC không có góc tù (trực tâm H ở trong tam giác): S (HBC) =HA 0 =tanHBC =cotACB = 1 S (ABC) AA0 tanABC tanABC tanB:tanC tanB =) Sa = tanA S(ABC). Tương tự, Sb = S (ABC) . S c = tanA:tanB:tanC tanA:tanB:tanC tanC S ~ = Sa ! + ! + ! c = ! + . Từ (1.2.3):0 tanA:tanB:tanC (ABC) HA S HB S HC tanA:HA b tanB ! ! :HB + tanC:HC hay H [tan A.A, tan B.B, tan C.C]. + Khi H ở ngoài tam giác (tam giác ABC tù): Ta vẫn có kết quả trên. Nhờ định lí hàm tan ta còn có cách viết dạng khác : H [ 1 :A; 1 :B; 1 :C]: b 2 + c 2 a 2 c 2 + a 2 b 2 a 2 + b 2 c 2 Đương nhiên với ký hiệu tọa độ tỷ cự trong tam giác ABC ta có thể viết: G = (1:1:1); I = (a: b: c); O = (sin 2A: sin 2B: sin 2C) = (a 2 (b 2 + c 2 a 2 ) : b 2 (c 2 + a 2 b 2 ) : c 2 (a 2 + b 2 c 2 )); Oa = (-a: b: c), Ob = (a: -b: c), Oc = (a: b: -c); 1 1 1 H = (tan A: tan B: tan C) = ( : : ). b 2 + c 2 a 2 c 2 + a 2 b 2 a 2 + b 2 c 2 Ví dụ 2.10. Với các ký hiệu như ví dụ trên, hãy chứng minh S DOIH = 1(a b)(b c)(c a): 8r Lời giải. Theo (1.8) diện tích 4 OIH là
  • 46. Tải tài liệu tại sividoc.com Viết đề tài giá sinh viên – ZALO:0973.287.149-TEAMLUANVAN.COM 29 : R : 1:R. a cos a b cos b c cos g ABC a b c a cos b cos g b cos g cos a c cos a cos b 2rp 2p rp = R2 abc cos a cos b cos g 1 1 1 4r p 2 R2 cos g cos a cos b cos g cos a cos b = (cos b cos a)(cos g cos b )(cos a cosg): Sử dụng công thức cos b p cos a = c 2 + a 2 b 2 b 2 + c 2 a 2 (a b)(a + b + c)(a + b c) (a b)(p c) = = 2ca 2bc 2abc 2rR thu được: 1 1 (p - a)(p - b)(p - c)(a - b)(b - c)(c - a) = (a - b)(b - c)(c - a). 8pr2 8r Chú ý rằng rp = p . p(p a)(p b)(p c) Ví dụ 2.11. Cho tam giác ABC có đường tròn ngoại tiếp (O) và đường tròn nội tiếp (I). Các đường thẳng AO, BO, CO cắt (O) tương ứng tại D, E, F các đường thẳng ID, IE, IF cắt BC, CA, AB tương ứng tại X, Y, Z. Chứng minh rằng AX, BY, CZ đồng quy. Tìm tọa độ tỷ cự của điểm đồng quy. Lời giải. Nếu tam giác ABC vuông chẳng hạn tại A, khi đó BY, CZ BC, bài toán hiển nhiên. Bởi vậy ta chỉ cần xét tam giác ABC không vuông, khi đó SA;SB;SC 6= 0. Hình 2.5: Tọa độ tỷ cự điểm đồng quy
  • 47. Tải tài liệu tại sividoc.com Viết đề tài giá sinh viên – ZALO:0973.287.149-TEAMLUANVAN.COM 30 Phương trình đường thẳng AD: c 2 SCy = b 2 SBz =) z = c2SC y. Suy ra tọa độ b2 SB của D là nghiệm của hệ 8 z = b2SB y > c2 S C 2 yz 2 2xy = 0 < a +b zx + c > Cho x = 1 ta có hệ: : z = c 2 S 8 b2 SB y > C 2yz 2 2 y = 0 < a +bz + c 2 2 c 2 SC c > 2SC 2 c 2 SC = y a : + y: : + yc : = 0 b 2 SB SB b 2 SB ) b 2 c 2 (S + S ) b 2 B () y 2 a 2 c 2 SC + yb 2 c 2 (SB + SC) = 0 () y = C = : Tương a2 c2 S S c2 b2 c2 C C tự ta có: z = =) D 1 : : = ( SBSC : b 2 SB : c 2 SD): Suy ra SB SC SB đường thẳng ID có phương trình bc(cSC bSB)x cSC(SB + ca)y + bSB(SC + ab)z = 0 Kết hợp với phương trình BC: x = 0 cho ta X=BCID= 0: bS cS Bằng cách hoán vị vòng quanh a, b, c B : C SB + ca SC + ab ta thu được tọa độ của Y, Z: ; Z = SA + A bc : SB + B ca : 0 : Y = SC + C ab : 0 : SA + A bc cS aS aS bS Suy ra AX, BY, CZ đồng quy tại H có tọa độ tỷ cự là H= SA +A bc : SB +B ca : SC + C ab : aS bS cS Các bài toán khác Bài toán 2.18. Chứng minh phương trình các đường đặc biệt trong tam giác: a. Phương trình các cạnh BC, CA, AB của tam giác cơ sở ABC tương ứng là: x = 0; y = 0; z = 0. b. Trung trực của cạnh BC: (b 2 c 2 )x + a 2 (y z) =0. Tương tự cho trung
  • 48. Tải tài liệu tại sividoc.com Viết đề tài giá sinh viên – ZALO:0973.287.149-TEAMLUANVAN.COM 31 trực hai cạnh kia. c. Phương trình đường thẳng Euler (SAB SCA)x + (SBC SAB)y + (SCA SBC)z = 0 d. Phương trình đường thẳng OI nối tâm ngoại tiếp O = (a 2 S A : b 2 S B : c 2 S ) C với tâm nội tiếp I = (a:b:c):å(b 2 SBc c 2 SCb)x = åbc(bSB cSC)x = 0 Vì bSB cSC = ... = -2(b - c)p(p - a) nên phương trình này có thể viết lại là åbc(b c)p(p a)x= 0 hay å(b c)(p a) x = 0. a Bài toán 2.19. Cho 4 ABC không vuông tại C. AD, BE là các đường cao còn AP, BQ là các phân giác trong ( D, P 2 BC, E, Q 2 CA). Gọi I, O lần lượt là tâm đường tròn nội tiếp, ngoại tiếp tam giác ABC. chứng minh rằng: D, E, I thẳng hàng () P, Q, O thẳng hàng. Bài toán 2.20. Cho DABC. Gọi D, E, F là ảnh của A, B, C qua phép đối xứng trục SBC;SCA;SAB theo thức tự đó. Gọi (O) và H là đường tròn ngoại tiếp và là trực tâm của tam giác. Chứng minh rằng D, E, F thẳng hàng () OH = 2R. Bài toán 2.21. Cho 4 ABC không cân tại C. Gọi M là trung điểm của AB. AH, CK lần lượt là đường cao, phân giác trong của tam giác ABC. K, L là trung điểm của CH, CD còn P = CD MK. Chứng minh rằng P và L là hai điểm liên hợp đẳng giác trong tam giac ABC. 2.3 Kỹ thuật giao hoán-kết hợp. Kỹ thuật giao hoán-kết hợp dựa vào tính chất 1.3: Nếu s(1);s(2); :::;s(n) là một hoán vị của tập hợp f1;2; :::;ng thì [miAi]1 i n [m s(i) A s(i) ] 1 s(i) n . Nếu hai hệ chất điểm fmiAign i=1;fn jB jgn j=1 thỏa mãn: Sn i=1mi + Sn j=1k j 6= 0 thì
  • 49. Tải tài liệu tại sividoc.com Viết đề tài giá sinh viên – ZALO:0973.287.149-TEAMLUANVAN.COM [m1A1; :::;mnAn;k1B1; :::;knBn] [[m1A1; :::;mnAn];[k1B1; :::;knBn]]:
  • 50. Tải tài liệu tại sividoc.com Viết đề tài giá sinh viên – ZALO:0973.287.149-TEAMLUANVAN.COM 32 Khi áp dụng kỹ thuật này ta hay nhóm lại và kết hợp một biểu thức tỷ cự theo những kiểu khác nhau, giống như cách nhóm kết hợp một biểu thức đại số. Kỹ thuật này thường được sử dụng trong các bài toán tính tỷ số hoặc chứng minh tính đồng quy, thẳng hàng. Ta minh họa kỹ thuật này qua các ví dụ sau: Ví dụ 2.12. Trong tam giác ABC, điểm F nằm trên cạnh BC, chia BC theo tỷ số 3:1, kể từ B. Các điểm M và P tương ứng nằm trên các cạnh AB, AC chia chúng theo tỷ số 1:6 kể từ A và C. Giao điểm của hai đoạn thẳng MP và AF chia các đoạn thẳng này theo tỷ số nào? Lời giải. Hình 2.6: Tính tỷ số Ta sẽ đặt tại B và C các trọng lượng sao cho tâm tỷ cự của hệ là điểm F: Chỉ cần đặt tại B trọng lượng 1 và tại C trọng lượng 3 (theo quy luật Archimedes), tức là xét hệ f1B, 3Cg. Ta thấy 1B và xA thỏa mãn 1.BM = x.MA, suy ra, x = BM : MA = 5. Cuối cùng, xét hệ chất điểm f3C, yAg sao cho P là tâm tỷ cự. Theo quy tắc Archimedes, ta có: 3CP = yPA, suy ra, y = 3CP:PA = 0,6 = 3 . 5 Tới đây xuất hiện sự kiện mới: tại A có hai trọng lượng khác nhau: 5 và 0,6. Áp dụng kỹ thuật giao hoán-kết hợp, gọi Z [1B,5A,3C,0,6A] = [[1B,5A],[3C,0,6A]] = [6M,3,6P]=) Z 2 MP. Lại xét Z [1B,5A,3C,0,6A] = [[1B,3C],[5,6A]] = [4F,5,6A]=) Z2AF. Vì Z [5,6A,4F] nên 5,6 AZ = 4FZ hay AZ : FZ = 5:7. Tương tự, 6MZ = 3,6PZ hay MZ:ZP = 3:5. Ta có kết quả cần tìm. Dùng ký hiệu thu gọn và kỹ thuật giao hoán-kết hợp có thể trình bày lời giải bài toán ở ví dụ 2.3.1 như sau: Từ giả thiết ta có 4F = 1B + 3C; 6M = 1B + 5A;
  • 51. Tải tài liệu tại sividoc.com Viết đề tài giá sinh viên – ZALO:0973.287.149-TEAMLUANVAN.COM 33 1B+5A+3C+0;6A 6P = 3C + 0,6A. Giả sử Z = : Khi đó, Z = (1B + 5A) + (3C + 0;6A) = 6M+3;6P =)Z2MP: 9;6 9;6 Z = (1B + 3C) + (5A + 0;6A) = 4F+5;6A =)Z2FA: 9;6 9;6 Ta thấy Z = MPFA và theo quy luật Archimedes, 6MZ = 3,6ZP; 4FZ = 5,6ZA. Từ đó, MZ:ZP = 3:5; AZ:FZ = 5:7. Ví dụ 2.13. Các điểm M, N, P, Q, R, S theo thứ tự là trung điểm các cạnh AB, BC, CD, DE, EF, FA của lục giác ABCDEF. Chứng minh rằng DMPR;DNQS có cùng trọng tâm. Lời giải. Chọn G [1A, 1B, 1C, 1D, 1E, 1F] thì G duy nhất. Dùng kỹ thuật giao hoán-kết hợp: G [[1A, 1B], [1C, 1D], [1E, 1F]] [2M, 2P, 2R]. Từ đó, G là trọng tâm DMPR: Lại áp dụng kỹ thuật giao hoán-kết hợp kiểu khác thì G [[1B, 1C], [1D, 1E], [1F, 1A]] [2N, 2Q, 2S]. Từ đó, G là trọng tâm DNQS. Vậy hai tam giác MPR và NQS có cùng trọng tâm. Ví dụ 2.14. Qua điểm P ở trong hình bình hành ABCD kẻ các đường thẳng song song với các cạnh, chúng cắt các cạnh AB, BC, CD, DA tại K, L, M, N. Gỉa sử Q là giao các đường trung bình của tứ giác KLMN còn S là tâm hình bình hành ABCD. Chứng minh rằng Q nằm trên đoạn thẳng PS. Điểm này chia đoạn PS theo tỷ số nào? Lời giải. Thoạt tiên ta sẽ đặt các trọng lượng tại 4 đỉnh K, L, M, N sao cho tâm tỷ cự của hệ 4 điểm đó là Q. Để có được điều đó chỉ cần đặt tại mỗi đỉnh các trọng lượng bằng 1:Q [1K, 1L, 1M, 1N]. Bây giờ chú ý rằng KLMN là hình bình hành nên ta có thể thay hệ f1K, 1Lg bằng hệ f1B, 1Pg, tức là Q [1B, 1P, 1D, 1P]. Suy ra Q [1S, 2P] (S là trung điểm của BD). Nhưng khi đó theo quy tắc Archimedes điểm Q nằm đoạn thẳng PS và chia đôi đoạn thẳng này.
  • 52. Tải tài liệu tại sividoc.com Viết đề tài giá sinh viên – ZALO:0973.287.149-TEAMLUANVAN.COM Ví dụ 2.15. Tứ giác ABCD ngoại tiếp đường tròn, các tiếp điểm là M, N, P, Q. Biết độ dài các đoạn thẳng tiếp tuyến xuất phát từ A, B, C, D tương ứng bằng a, b, c, d. Giao điểm của MP và NQ chia mỗi đoạn thẳng đó theo tỷ số nào?
  • 53. Tải tài liệu tại sividoc.com Viết đề tài giá sinh viên – ZALO:0973.287.149-TEAMLUANVAN.COM 34 Lời giải. Đặt các trọng lượng m1;m2 tại các điểm A, B sao cho M [m1A, m2B]. Theo quy tắc Archimedes, am1 = bm2 bởi vậy có thể lấy m1 = 1 ;m2 = 1 . Từ a b cách làm tương tự, bây giờ nếu tại các đỉnh A, B, C, D ta đặt các trọng lượng 1 ; 1 ; 1 ; 1 thì M [ 1 A, 1 B], N [ 1 B, 1 C], P [ 1 C, 1 D], Q [ 1 D, 1 A]. Áp a b c d a b b c c d d a dụng kỹ thuật kết hợp: 1 1 1 1 1 1 1 1 Gọi Z [ A, B, C, D] [( + )M, ( + )P] =) Z 2 MP. Tương tự, ta a b c d a b c d khảng định được Z thuộc đoạn NQ. Do đó, Z là giao của các đoạn thẳng MP và NQ. Theo quy tắc Archimedes, ( 1 + 1 )MZ=( 1 +1 )ZP. Từ đó nhân đượcMZ a c ZP 1 1 1 1 (c + d)ab b d = ( + ): ( + ) = . c d a b (a + b)cd Tương tự: NZ = ( 1 +1): (1 +1 ) = (a + d)bc . ZQ a d b c (b + c)ad Ví dụ 2.16. Trên các cạnh AC, BC của tam giác ABC ta lấy các điểm M và P sao cho: AM : MC = 3 : 1; BP : PC = 1 : 2. Các đoạn thẳng AP và BM cắt nhau ở Q. Biết diện tích tam giác BPQ bằng 1m 2 hãy tính diện tích tam giác ABC. Lời giải. Hình 2.7: Tính diện tích Dễ thấy, SABC = 3SBAP = 3SBPQ:AP. Bởi vậy ta chỉ cần tính tỷ số AP . Ta đặt các QP QP trọng lượng m1;m2;m3 lần lượt vào các đỉnh A, B, C sao cho M [m1A;m3C] còn P [m3C;m2B]: Theo giả thiết bài toán và quy tắc Archimedes ta sẽ đặt trọng lượng m1 = 1 vào A, trọng lượng m3 = 3 vào C và trọng lượng m2 = 6 vào
  • 54. Tải tài liệu tại sividoc.com Viết đề tài giá sinh viên – ZALO:0973.287.149-TEAMLUANVAN.COM B. Giả sử Z [1A, 3C, 6B], Khi đó
  • 55. Tải tài liệu tại sividoc.com Viết đề tài giá sinh viên – ZALO:0973.287.149-TEAMLUANVAN.COM 35 1A +3C+6B (1A+3C)+6B 4M+6B Z = = = =)Z2MB 10 10 10 1A + (3C + 6B) 1A+9P Z = = =)Z2PA. 10 10 Từ đó, Z = MB PA, tức Z Q. Từ đẳng thức trên theo quy tắc Archimedes: 1.AQ = 9.QP; AP = AQ + QP = 10QP. Nghĩa là AQ = 10 =) SABC = 30m2 . AP Ví dụ 2.17. Mặt phẳng a cắt các cạnh bên PA, PB, PC của chóp tam giác đều P.ABC tương ứng theo tỷ số 2 ; 3 ; 4 . Mặt phẳng a cắt đường cao PM của hình 5 5 5 chóp theo tỷ số nào? Lời giải. Hình 2.8: Hình chóp tam giác đều Gọi A1;B1;C1 là giao điểm của các cạnh SA, SB, SC với mặt phẳng a. Đặt các đỉnh A, B, C các trọng lượng bằng 1. VìAP= 5AP tức là: AA1 =3 A1P nên 5 3 2 3 2 A1 = 1A+ P; tức là A1 là tâm tỷ cự của hệ f1A; Pg. 2 2 2 Tương tự, 5 B1 = 1B+2P; 5 C 1 = 1C +1 P: Ngoài ra do chóp S.ABC là chóp 3 3 4 4 tam giác đều nên 3K = 1A + 1B + 1C. Giả sử Q là tâm tỷ cự của hệ 6 chất điểm đang xét, tức là: 1A + 3P+1B+ 2P+1C+ 1P = 65Q: 2 3 4 12 Áp dụng kỹ thuật giao hoán-kết hợp ta có 65Q = (1A+ 3P)+(1B+ 2P)+(1C+ 1P) = 5A1 +5B1 +1 C1: 12
  • 56. Tải tài liệu tại sividoc.com Viết đề tài giá sinh viên – ZALO:0973.287.149-TEAMLUANVAN.COM 2 3 4 2 3 4
  • 57. Tải tài liệu tại sividoc.com Viết đề tài giá sinh viên – ZALO:0973.287.149-TEAMLUANVAN.COM 36 Suy ra Q 2 a: Áp dụng kỹ thuật giao hoán-kết hợp theo kiểu khác: 65 Q= (1A + 1B + 1C) + ( 3 P + 2 P) + 1 P) + (1C + 1 P) = 3M + 29 p; 12 2 3 4 4 12 ta thu được Q 2 PM: Vậy Q = PM a. Các bài toán khác Bài toán 2.22. Trong góc PAQd vẽ nội tiếp một đường tròn tiếp xúc với hai cạnh của góc tại P, Q. Đường thẳng BC, B 2 (AP), C 2 (AQ) tiếp xúc với đường tròn tại T. Gọi M = BQ CP. Chứng minh rằng ba điểm A, T, M thẳng hàng. Bài toán 2.23. Đáy hình chóp F.ABCD là hình bình hành. Mặt phẳng a cắt các cạnh bên, BF, CF, DF tương ứng tại A1;B1;C1;D1 sao cho AA 1 = 2; BB 1 = 5; CC 1 = 10: A1F B1F C1F DD1 Tính tỷ số x = : ! ! ! ! Bài toán 2.24. Trên cạnh4 ABC lấy các điểm L, M sao cho: CL = aCA;CM = b CB (0 < a < 1; 0 < b < 1). Giả sử P là giao của AM và BL. Tính tỷ số AP : AM 2.4 Kỹ thuật quán tính. Kỹ thuật mô men quán tính dựa vào các tính chất 1.5 và 1.6 nói về các công thức Lagrang và Jacobi. Với kỹ thuật này ta thu được các công thức về khoảng cách. Đặc biệt cho ta phép chứng minh mới về công thức Euler. Ví dụ 2.18. Biết các bán kính đường tròn ngoại tiếp, nội tiếp tam giác ABC là R;r. Tính khoảng cách d giữa các tâm đường tròn này. Lời giải. Giả sử Z là tâm nội tiếp và O là tâm ngoại tiếp, a, b, c là độ dài các cạnh tam giác ABC. Dễ thấy điểm Z là tâm tỷ cự của hệ chất điểm aA, bB, cC. Theo công thức Lagrang, (a + b + c)jOZj 2 = JO JZ. Hơn nữa, theo định nghĩa mô men quán tính thì JO = ajOAj 2 + bjOBj 2 + cjOCj 2 = (a + b + c) 2 R 2 : 1 (abjABj 2 +bcjBCj 2 +cajCAj 2 ) = a + b + c
  • 58. Tải tài liệu tại sividoc.com Viết đề tài giá sinh viên – ZALO:0973.287.149-TEAMLUANVAN.COM Cuối cùng, theo công thức Jacobi, JZ =
  • 59. Tải tài liệu tại sividoc.com Viết đề tài giá sinh viên – ZALO:0973.287.149-TEAMLUANVAN.COM 37 abc 2 + bca 2 + cab 2 = abc. Bởi vậy, d 2 = jOZj 2 = JO JZ = R 2 abc . Từ a + b + c a + b + c a + b + c các công thức quen biết S = abc =1(a + b + c)r ta rút ra: abc = 2Rr và 4R 2 a + b + c bởi vậy, d 2 = R(R 2r), ta thu được công thức Euler. Ví dụ 2.19. (Bài toán Apolonius) Trong mặt phẳng cho hai điểm A, B. l 6= 1 là một số cho trước. Tìm quỹ tích các điểm M sao cho MA = l . MB Lời giải. Điều kiện MA = l tương đương với hệ thức l 2 MB2 MA 2 = 0. Ta MB nhận thấy vế trái là mô men quán tính của hệ hai chất điểm fl 2 B;( 1)Ag đối với điểm M. Nghĩa là bài toán chuyển về tìm quỹ tích điểm M sao cho JM = 0. Ký hiệu Z là tâm tỷ cự của hệ chất điểm fl 2 B;( 1)Ag, ta có theo công thức Lagrang và Jacobi: JM = JZ + (l 2 1)ZM 2 = l2 AB 2 + (l 2 1)ZM 2 : l 2 1 Bởi vậy điều kiện JM = 0 được thực hiện khi và chỉ khi l ZM = jl 2 l Từ đó rõ ràng quỹ tích cần tìm là đường tròn tâm Z, bán kính jl 2 1jAB: Nhận xét 2.1. Có thể tổng quát hóa bài toán như sau: Ký hiệu s là hệ chất điểm m1A1;m2A2; :::;mnAn: Mô men quán tính của hệ này đối với điểm M được ký hiệu là JM(s): Ta giải thích dạng tập hợp F tất cả các điểm M trên mặt phẳng sao cho JM(s) = h với h là hằng số cho trước. Lời giải. Trường hợp 1. Nếu m = m1 + m2 + ::: + mn 6= 0 thì gọi Z là tâm tỷ cự của hệ s ta có theo công thức Lagrang, ZM 2 = 1 (k JZ(s)): m 1jAB:
  • 60. Tải tài liệu tại sividoc.com Viết đề tài giá sinh viên – ZALO:0973.287.149-TEAMLUANVAN.COM Từ đó ta thấy F là đường tròn (với JZ(s) < h), hoặc một điểm (với JZ(s) = h) hoặc rỗng (với JZ(s) > h). Trường hợp 2. Nếu m = m1 + m2 + ::: + mn = 0 nhưng ít nhất có một mi 6=
  • 61. Tải tài liệu tại sividoc.com Viết đề tài giá sinh viên – ZALO:0973.287.149-TEAMLUANVAN.COM 38 0, chẳng hạn mn 6= 0. Ký hiệu C là tâm tỷ cự của hệ s 0 gồm các chất điểm m1A1;m2A2; :::;mn 1An 1: Với mọi M được chọn ta có: J (s0 ) = J (s0 ) + ( m )MC2 ; J (s) = J (s0 ) + ( m )MA2 . M C n M M n n mn:MA 2 n + ( mn)MC 2 = h JC(s 0 ) = const: Khi C 6= An hình F là một đường thẳng vuông góc với CAn. Nếu C An hình F là toàn mặt phẳng hoặc rỗng. Trường hợp 3. Nếu tất cả các mi đều bằng không, hình F sẽ là toàn mặt phẳng (khi h = 0) hoặc tập hợp rỗng. Ví dụ 2.20. Gọi các đường tròn bàng tiếp của DA1A2A3 là g1;g2;g3: Giả sử g1 tiếp xúc với cạnh A2A3 tại B1, g2 tiếp xúc với cạnh A3A1 tại B2, g3 tiếp xúc với cạnh A2A1 tại B3. Qua B1;B2;B3 kẻ các đường thẳng d1;d2;d3 tương ứng vuông góc với các cạnh A2A3;A1A3;A2A1. Chứng minh rằng các đường thẳng này đồng quy tại một điểm. Lời giải. Xét ba hệ chất điểm s1 : 0:A1 1:A2 ( 1):A3 s2 : ( 1):A1 0:A2 1:A3 s3 : 1:A1 ( 1):A2 0:A3 Đường thẳng d1 được đặc trưng bởi điều kiện (xem nhận xét sau bài toán trên) JM(s1) = JB1 (s1) = B1A 2 2 B1A 2 3(= h1) Để tính h1 ta có thể ký hiệu T2;T3 là tiếp điểm của g1 với các đường thẳng A1A2 và A1A3 và đặt A2A3 = a;A3A1 = b;A1A2 = c;B1A2 = x;B1A3 = y;A1T2 = z. Khi đó, x + y = a, z - x = c, z - y = b Nhận được x - y = b - c và bởi vậy, h1 = x 2 y 2 = (x + y)(x y) = a(b c)
  • 62. Tải tài liệu tại sividoc.com Viết đề tài giá sinh viên – ZALO:0973.287.149-TEAMLUANVAN.COM 39 Như vậy điều kiện JM(s1) = h1; đặc trưng cho đường thẳng d1 có dạng 1:MA2 2 + ( 1)MA3 2 = a(b c) (2.1) Tương tự các đường thẳng d1;d2 được đặc trưng bởi 1:MA3 2 +( 1)MA1 2 = b(c a) (2.2) 1:MA1 2 +( 1)MA2 2 = c(a b) (2.3) Bây giờ gọi M = d1 d2 thì M thoa mãn các điều kiện (2.1, 2.2). Từ đây cộng các hệ thức này và nhận kết quả với (-1) thì nhận được (2.3). Điều đó nghĩa là M thỏa mãn cả (2.3), tức M thuộc d3. Ví dụ 2.21. (Công thức tính khoảng cách giữa hai điểm) Giả sử P [p1A1; p2A2; :::; pnAn]; Q [q1A1;q2A2; :::;qnAn]: Khi đó, khoảng cách của hai điểm P, Q được tính theo công thức PQ 2 =å (pi qi)(pj qj) j AiAj j2 : (2.4) 1 i< j n Chứng minh. Ta xét với n = 3 (trường hợp tổng quát suy luận tương tự). Theo điều kiện, 1:P = p1A1 + p2A2 + p3A3;1:Q = q1A1 + q2A2 + q3A3 p1 + p2 + p3 = q1 + q2 + q3 = 1 Xét 3 hệ chất điểm s1 : s2 : s3 : (p1 q1)A1; (p2 q1A1; p1A1; q2)A2; (p3 q2A2; p2A2; q3)A3; q3A3; p3A3; 1Q: 1Q: Các mô men quán tính của các hệ này đối với điểm P được ký hiệu là J ( P 1) ;J ( P 2) ;J ( P 3) còn đối với Q được ký hiệu là J ( Q 1) ;J ( Q 2) ;J ( Q 3) . Tâm tỷ cự của các hệ s1;s2;s3 tương ứng là các điểm P, Q và trung điểm Z của đoạn PQ. Bởi vậy ta có: J ( P 3) = J ( Z 3) + 2 j PZ j 2 ;J ( Q 3) = J ( Z 3) + 2 j QZ j 2 :
  • 63. Tải tài liệu tại sividoc.com Viết đề tài giá sinh viên – ZALO:0973.287.149-TEAMLUANVAN.COM 40 và do đó, J ( P 3) = J ( Q 3) = p1 j QA1 j 2 +p2 j QA2 j 2 +p3 j QA3 j 2 . Ngoài ra, theo định lý Lagrang, J (2) = J (2) + 1: j PQ j 2 = q j QA j 2 +q j QA j 2 +q j QA j 2 + j PQ j 2 : P Q 1 1 2 2 3 3 JP (1) = å (pi 3 qi)(pj qj) j AiAj j 2 + å (pi qi) j AiQ j 2 : 1 i<j 3 i=1 Từ hê thức hiển nhiên: JP (3) = JP (1) + JP (2) ta có JQ (3) = JZ (3) + 2 j QZ j 2 = = å (pi qi)(pj 3 qj) j AiAj j 2 + å (pi qi) j AiQ j 2 +JQ (2) + 1: j PQ j 2 1 i<j 3 i=1 = q1 j QA1 j 2 +q2 j QA2 j 2 vế ta có PQ 2 = +q3 j QA3 j 2 + j PQ j 2 . Sau khi giản ước và chuyển å (pi qi)(pj qj) j AiAj j2 : 1 i<j n Các bài toán khác Bài toán 2.25. Cho a, b, c là độ dài ba cạnh của tam giác ABC. Tính độ dài đường phân giác. Bài toán 2.26. Cạnh của tứ diện đều bằng a. Tính bán kính mặt cầu ngoại tiếp. Bài toán 2.27. Cho O là tâm đường tròn nội tiếp tam giác ABC, a, b, c là độ dài các cạnh của tam giác. Tính tổng: s = aOA 2 + bOB 2 + cOC 2 Chương hai trình bày các kỹ thuật biến đổi tâm tỷ cự với các ví dụ minh họa phong phú cùng các bài toán bổ sung nhằm làm rõ ý tưởng của phương pháp áp dụng tâm tỷ cự vào các bài toán chúng minh, tính tỷ số, tìm quỹ tích. Rõ ràng áp dụng khái niệm tâm tỷ cự theo các kỹ thuật trên đã thu được các kết quả tốt. Tiếp sang chương ba các kỹ thuật đó được phối hợp khá thành công trong các ví dụ khó hơn: chứng minh các định lý cổ điển và giải một số đề thi Olympic.
  • 64. Tải tài liệu tại sividoc.com Viết đề tài giá sinh viên – ZALO:0973.287.149-TEAMLUANVAN.COM 41 Chương 3 Các vấn đề liên quan Với khuôn khổ của một luận văn, trong chương này tác giả chỉ trình bày hai phần có áp dụng các kỹ thuật biến đổi tâm tỷ cự. Đó là chứng minh lại một số định lý và giải một số bài toán thi Olympic quốc tế bằng cách áp dụng tâm tỷ cự. Nội dung chương này chủ yếu được tham khảo trong [6] với cách trình bày chi tiết và có chọn lọc. 3.1 Chứng minh một số định lý nổi tiếng Ta sẽ áp dụng khái niệm tâm tỷ cự vào việc chứng minh các kết quả sau: định lý Routh, định lý Mene 0 laus, định lý Ce 0 va, công thức Euler, đường thẳng Euler, đường thẳng Nagel, công thức Sterwartz, một số hệ thức liên quan đến phương tích, định lý Ptolemy, bất đẳng thức Gerretsen I, II,... Ví dụ 3.1. (Định lý Routh) Cho tam giác ABC, các điểm M 2 BC, N 2 CA, P 2 AB và chia các đoạn BC, CA, AB theo các tỷ số m : 1; n : 1; p : 1. Gọi S1 là diện tích tam giác tạo bởi giao điểm của AM, BN, CP; S2 = S(MNP): Khi đó hãy chứng minh: (mnp 1)2 a.S1 = (mn + m + 1)(np + n + 1)(pm + p + 1) :S(ABC); mnp + 1 b.S 2 = (m + 1)(n + 1)(p + 1) :S (ABC) : Chứng minh. Ta có D=(0 : 1 : m), E=(n : 0 : 1), F=(1 : p : 0). Gọi I = BN CP, các điểm I, K được xác định tương tự. Ta tìm được I=(n : np : 1), J=(1 : p : pm),
  • 65. Tải tài liệu tại sividoc.com Viết đề tài giá sinh viên – ZALO:0973.287.149-TEAMLUANVAN.COM 42 K=(mn : 1 : m). Suy ra n np 1 1 p pm mn 1 m S 1 = :S (ABC) ; (mn + m + 1)(np + n + 1)(pm + p + 1) = (mnp 1) 2 :S (ABC) : (mn + m + 1)(np + n + 1)(pm + p + 1) 0 1 m n 0 1 1 p 0 S2 = :S (ABC) ; (m + 1)(n + 1)(p + 1) = (mnp + 1):S(ABC) : (m + 1)(n + 1)(p + 1) Các đẳng thức được chứng minh. Định lý Routh chính là kết quả tổng quát của hai định lý nổi tiếng: Định lý Ce’va: Nếu mnp = 1 thì S1 = 0; AM; BN; CP đồng quy; Định lý Mene’laus: Nếu mnp = 1 thì S2 = 0; M; N; P thẳng hàng. Nhận xét 3.1. Hai định lý này có thể chứng minh trực tiếp bằng phương pháp chọn tâm tỉ cự. Rõ ràng cách chứng ba định lý trên mang đặc trưng của tâm tỷ cự. Tọa độ tỷ cự có thể dùng để xem xét tính thẳng hàng của các điểm. Giả sử ta [PBC] có hai điểm chuẩn hóa P = (x1 : y1 : z1);Q = (x2 : y2 : z2) theo nghĩa x1 = [ABC], tương tự cho các y1;z1;x2;y2;z2. Gọi P1;P2;P3 là hình chiếu của P lên các đường thẳng BC, CA, AB tương ứng và tương tự đối với Q. Vì [PBC] = 1:PP1:a nên ta 2 có: PP1 =2[PBC] =2[ABC]:x1. Tương tự, QQ1 =2[ABC] :x2. a a a Xét điểm R(x3 : y3 : z3) trên đường thẳng BC sao cho PR = k với k 2 R nào đó PQ
  • 66. Tải tài liệu tại sividoc.com Viết đề tài giá sinh viên – ZALO:0973.287.149-TEAMLUANVAN.COM và R1 là hình chiếu của nó trên BC. Theo tính chất hình thang PP1Q1Q có thể
  • 67. Tải tài liệu tại sividoc.com Viết đề tài giá sinh viên – ZALO:0973.287.149-TEAMLUANVAN.COM 43 Hình 3.1: P;Q;R thẳng hàng xem RR1 = (1 k):PP1 + k:QQ1, tức là x3 = (1 k):x1 + k:x2. Tương tự đối với y3 và z3 ta tìm được R=(1 k)P + k:Q: (3.1) Đây là phương trình ba điểm thẳng hàng sau khi các điểm đã được chuẩn hóa. Hơn nữa, giá trị của k mà R = (1 k):P +k:Q tương ứng với tỷ số k = PR . Nhắc PQ lại là P = (x1 : y1 : z1);Q = (x2 : y2 : z2);R = (x3 : y3 : z3) thẳng hàng khi và chỉ khi x 1 x 2 x 3 = 0. y1 y2 y3 z1 z2 z3 + Đường thẳng Euler: Các điểm H, G, O nằm trên một đường thẳng theo thứ tự đó và OH = 3OG. Diện tích tam giác OBC là 1R 2 sin 2a = R 2 sin a:cos a =1Ra cos a, ta thu được 2 2 tọa độ chuẩn hóa của O là O = R (a cos a : : ) = 1 (a 2 (b 2 + c 2 a 2 ) : : ) 2rp 4p 2 r 2 G và H đã có chuẩn hóa như sau: G = 1(1:1:1);H= R (a cos b cos g : b cos g cos a : c cos a cos b ). Ngoài ra với 3 pr chủ ý lập quan hệ giữa các điểm G, O, H ta viết: 1 = Ra:cos b :cos g +Ra :cos a r p r p
  • 68. Tải tài liệu tại sividoc.com Viết đề tài giá sinh viên – ZALO:0973.287.149-TEAMLUANVAN.COM Đẳng thức này tương đương đẳng thức hiển nhiên abc = 1 với chú ý rằng 4prR
  • 69. Tải tài liệu tại sividoc.com Viết đề tài giá sinh viên – ZALO:0973.287.149-TEAMLUANVAN.COM 44 bc cos a + cos b cos g = cos a + cos(b + g) + sin b sin g = 4R2 . Cuối cùng, ta thu được phương trình đường thẳng Euler là 3G = H + 2O. + Đường thẳng Nagel: Ký hiệu các tiếp điểm là D 2 BC;E 2 AC;F 2 AB. Sử dụng định lý Ce 0 va chứng minh được các đường thẳng AD;BE;CF đồng quy tại một điểm. Đó là điểm Nagel N (giao của các đường thẳng nối các đỉnh tam giác với các tiếp điểm của đường tròn bàng tiếp). Tâm Spieker có tọa độ chuẩn hóa S = 1 (b+c : c+a : 4p a + b). Điểm Nagel N, điểm Spieker S, trọng tâm G và tâm nội tiếp I thẳng hàng, đường thẳng đó có tên là đường thẳng Nagel, S và G tương ứng là điểm chia đôi và điểm chia ba của đoạn NI. Vì BD = (p c)(p b), tương tự với E và F nên 1 DC ta được N = (p a : p b : p c). Dễ kiểm tra được: p p a + a 2(N+I) = 2 2p : : = 4p : : = S 2 1 b c và 3I= 3p + 3p : : = 3 : : = G 3N + 1 2 p a a 1 Tức là ta có phương trình đường Nagel: ( N + I = 2S . N+2I = 3G Định lý 3.1. (Hệ thức Sterwartz) Trong tam giác ABC với mọi M thuộc cạnh BC ta có MB:AC 2 + MC:AB 2 = BC:MA 2 + MB:MC:MA: Chứng minh. Khi M thuộc cạnh BC thì dễ thấy M [MB:B; MC:C], áp dụng hệ thức Jacobi với hệ 2 điểm ta có: : :BC 2 MB MC MB:AC 2 MC:AB 2 = (MB MC)AM 2 MB MC () MBAC 2 MC:AB 2 = CB:AM 2 + MB:MC:BC: ! Chọn véc tơ đơn vị chỉ phương cùng phương với BC ta suy ra:
  • 70. Tải tài liệu tại sividoc.com Viết đề tài giá sinh viên – ZALO:0973.287.149-TEAMLUANVAN.COM 45 MB:AC 2 + MC:AB 2 = BC:MA 2 + MB:MC:BC Từ đồng nhất thức Jacobi đối với hệ 3 điểm với I [a:A;b :B;g:C], P là điểm tùy ý: a:PA2 + b :PB2 + g:PC2 = (a + b + g)PI2 + b g:a2 + ga:b2 + ab :c2 : a + b + g Trừ vế với vế cho (a + b + g)R 2 , theo tính chất của phương tích ta có: a:PA=(O) +b :PB=(O) +g:PC=(O) = (a +b +g)PI=(O) + b g:a2 + ga:b2 + ab :c2 a + b + g () P I=(O) = aP A=(O) + b P B=(O) + gP C=(O) b g:a 2 + ga:b 2 + ab :c 2 : (a + b + g) (a + b + g) 2 (3.2) Đây là công thức tính phương tích của tâm tỷ cự I [a:A;b :B;g:C] đối với đường tròn tùy ý tâm O. Các trường hợp đặc biệt cho ta các hệ quả sau: Hệ quả 3.1. Cho tam giác ABC và đường tròn ngoại tiếp (O), phương tích của điểm I [a:A;b :B;g:C] đối với (O) được tính theo công thức: b g:a 2 + ga:b 2 + ab :c 2 (3.3) P I=(O) = : (a + b + g) 2 Hệ quả 3.2. Điểm I [a:A;b :B;g:C] thuộc đường tròn ngoại tiếp (O) khi và chỉ khi b g:a 2 + ga:b 2 + ab :c 2 = 0: Hệ quả 3.3. (Hệ thức Euler) Tam giác ABC nội tiếp đường tròn tâm O, bán kính R, ngoại tiếp đường tròn tâm I, bán kính r. Khi đó OI 2 = R 2 2Rr. Chứng minh. Vì I [aA;bB;cC] nên áp dụng (3.3) P I=(O) = bc:a 2 + ca:b 2 + ab:c 2 = abc : (a + b + c) 2 (a + b + c) Vế trái bằng OI 2 R 2 . Vế phải bằng 2Rr với lưu ý: SABC =abc = p:r. Vậy 4R đẳng thức trên tương đương với OI 2 = R 2 2Rr:
  • 71. Tải tài liệu tại sividoc.com Viết đề tài giá sinh viên – ZALO:0973.287.149-TEAMLUANVAN.COM Tương tự đối với các đường tròn bàng tiếp (Ia;ra);(Ib;rb);(Ic;rc)
  • 72. Tải tài liệu tại sividoc.com Viết đề tài giá sinh viên – ZALO:0973.287.149-TEAMLUANVAN.COM 46 Hệ quả 3.4. OI 2 a = R 2 + 2Rra; OI 2 b = R 2 + 2Rrb; OI 2 c = R 2 + 2Rrc: Bài toán 3.1. Cho tam giác ABC. Các điểm O, G và H lần lượt là tâm ngoại tiếp, trọng tâm và trực tâm tam giác. Khi đó ta có: a. OG 2 = R 2 a 2 + b 2 + c 2 ; 9 b. PH=(O) = 8R 2 cos A cos B cos C: c. cos 2 A + cos 2 B + cos 2 C + 2 cos A cos B cos C = 1: Hệ quả 3.5. (Định lý Ptolemy) Cho tam giác ABC nội tiếp đường tròn (O) và D là điểm nằm trên đường tròn. Khi đó trong ba số sau có một số bằng tổng hai số kia aDA;bDB;cDC . Cụ thể: Nếu D thuộc cung nhỏ BC thì aDA = bDB + cDC Nếu D thuộc cung nhỏ CA thì bDB = cDC + aDA Nếu D thuộc cung nhỏ AB thì cDC = aDA + bDB: Chứng minh. Khi D thuộc cung nhỏ BC dễ thấy D [ a :A; b :B; c :C]: DA DB DC Áp dụng (3.3) ta có điều phải chứng minh. Khi D ở các vị trí còn lại làm tương tự. Hệ quả 3.6. Cho tam giác ABC với đường tròn ngoại tiếp (O) và đường tròn nội tiếp (I). Tâm tỷ cự P [a:A;b :B;g:C]. Khi đó a(p a) 2 + b (p b) 2 + g(p c) 2 (3.4) P P P=(O) = : P=(I) a + b + g Chứng minh. Gọi tiếp điểm của đường tròn (I) với các cạnh BC, CA, AB là D,E,F tương ứng. Theo công thức phương tích và tiếp tuyến ta có: PA=(O) = AE 2 = AF 2 = (p a) 2 ; P = BD 2 = BF 2 = (p b) 2 ; P = CE 2 = CD 2 = B=(O) C=(O)
  • 73. Tải tài liệu tại sividoc.com Viết đề tài giá sinh viên – ZALO:0973.287.149-TEAMLUANVAN.COM (p c)2 : Sau khi áp dụng công thức 3.2 ta thu được điều phải chứng minh.
  • 74. Tải tài liệu tại sividoc.com Viết đề tài giá sinh viên – ZALO:0973.287.149-TEAMLUANVAN.COM 47 Bài toán 3.2. Tam giác ABC ngoại tiếp đường tròn (I) có G là trọng tâm tam giác. Khi đó: p2 4r(4R + r) (3.5) P G=(I) = : 9 Chứng minh. Áp dụng công thức tính phương tích của G [1A;1B;1C] đối với đường tròn (I) ta có (p a) 2 + (p b) 2 + (p c) 2 a 2 + b 2 + c 2 P G=(I) = + : 3 9 Bằng phép biến đổi thích hợp ta suy ra điều phải chứng minh. Từ (3.5), GI 2 r 2 p 2 4r(4R + r) () GI 2 = p 2 16Rr + 5r 2 = . Suy ra 9 9 Hệ quả 3.7. (Bất đẳng thức Gerretsen thứ nhất) p 2 16Rr 5r 2 : Bài toán 3.3. Cho tam giác ABC nội tiếp đường tròn tâm (O) bán kính R, ngoai tiếp đường tròn tâm I, bán kính r. Gọi H là trực tâm tam giác. Khi đó: P = 4R 2 + 4Rr + 2r 2 p 2 (3.6) H=(O) p2 16Rr + 5r 2 Chứng minh. Ta đã có các công thức GI 2 = ; OI 2 = R 2 2Rr 9 9R2 (a2 + b2 + c2 ) và OG 2 . Từ hệ thức Euler ! ! ~ , áp dụng hệ = = 0 3HG 2HO 9 2IO 2 = IH 2 6OG 2 , bằng các tính toán thích hợp thức Jacobi ta suy ra: 3IG 2 ta suy ra điều phải chứng minh. Từ (3.6) dễ chỉ ra 0 IH 2 = 4R 2 + 4Rr + 3r 2 p 2 : Từ đó thu được Hệ quả 3.8. (Bất đẳng thức Gerretsen thứ hai) p 2 4R 2 + 4Rr + 3r 2 : Ví dụ 3.2. Cho tam giác ABC và (O9) là đường tròn Euler hay đường tròn chín
  • 75. Tải tài liệu tại sividoc.com Viết đề tài giá sinh viên – ZALO:0973.287.149-TEAMLUANVAN.COM 48 điểm của tam giác. Tâm tỷ cự P [a:A;b :B;g:C]. Khi đó abc cos A + b ca cos B + gab cos C b g:a 2 + ga:b 2 + ab :c 2 P P=(O9) = 2(a + b + g) (a + b + g)2 (3.7) Chứng minh. Ký hiệu A 0 ;B 0 ;C 0 là trung điểm các cạnh BC, CA, AB và D, E, F là chân các đường cao hạ từ A, B, C tương ứng. Ta chú ý rằng đường tròn Euler (O9) đi qua các điểm A 0 ;B 0 ;C 0 và D,E,F. Theo công thức phương tích: P A=(O9) = : = bc cos A. Tương tự, PB=(O9) = ca cos B;P C=(O9) = ab cos C . AO9 AB 0 2 2 2 Áp dụng 3.2 ta được P P=(O9) = abc cos A + b ca cos B + gab cos C b g:a 2 + ga:b 2 + ab :c 2 : (a + b + g) 2 2(a + b + g) Đó là điều phải chứng minh. Ta đã có kết quả sau: Cho tam giác ABC. Khi đó nếu PA=(GBC) = PB=(GCA) = PC=(GAB) thì G là trọng tâm tam giác . Phần đảo lại cũng đúng nhưng chứng minh khó hơn (T8/399/THTT): Nếu điểm G không nằm trên cạnh, không nằm trên đường tròn ngoại tiếp và thỏa mãn hệ thức trên thì G là trọng tâm tam giác. Ở đây ta sẽ chứng minh một kết quả tổng quát hơn: Ví dụ 3.3. Cho tam giác ABC và điểm P = (a : b : g) bất kỳ không nằm trên cạnh, không nằm trên đường tròn nội tiếp tam giác. Chứng minh rằng aP A=(PBC) = aP B=(PCA) = aP C=(PAB) = (a + b + g)P A=(PBC) : (3.8) Chứng minh. Vì A,B,C,P là các điểm tùy ý trong mặt phẳng nên ta có thể tìm PA + ~ PB + PC = 0, với tọa độ tỷ cự của A đối với tam giac cơ sở PBC. Từ a ! b! !g mọi M ta có: )MP + + = ( + b + ! aMA ! bMB ! gMC a g ! AB + AC=( a + b + )AP hay ( + b + )AP + AB + Cho M A được b ! g ! g ! a g ! b ! ~ . Như vậy, A [ ( + b + g )P : b B : g C] AC=0 g ! a . Theo hệ thức (3.3) áp dụng vào điểm A và (PBC): b ga 2 b (a + b + g)PB 2 (a + b + g)PC 2 P A=(PBC) = =
  • 76. Tải tài liệu tại sividoc.com Viết đề tài giá sinh viên – ZALO:0973.287.149-TEAMLUANVAN.COM [ (a + b + g) + b + g] 2
  • 77. Tải tài liệu tại sividoc.com Viết đề tài giá sinh viên – ZALO:0973.287.149-TEAMLUANVAN.COM 49 = b ga 2 (a + b + g)(b PB 2 + gPC 2 ) : a 2 Từ hệ thức Leibniz cho moi M ta có: aMA 2 + b MB 2 + gMC 2 = (a + b + g)MP 2 +aPA 2 +b PB 2 +gPC 2 hay aMA 2 +b MB 2 +gMC 2 = (a +b +g)MP 2 = (a + b + g)b ga 2 + gab 2 + ab c 2 . a + b + g Nếu cho M A ta được: b c 2 + gb 2 = (a + b + g)AP 2 b ga 2 + gab 2 + ab c 2 + : a + b + g Do đó PA 2 = (b + g)(b c 2 + gb 2 ) b ga 2 . Tương tự, (a + b + g) 2 PB 2 = (g + a)(gb 2 + ac 2 ) gab 2 ; PC 2 = (a + b )(ab 2 + b c 2 ) ab c 2 . Lại (a + b + g) 2 (a + b + g) 2 theo công thức b ga 2 + gab 2 + ab c 2 b PB 2 + gPC 2 = aPA 2 + b PB 2 + gPC 2 aPA 2 = a + b + g a (b + g)(b c 2 + gb 2 ) b ga 2 = (a + b + g)b ga 2 + a(b ga 2 + gab 2 + ab c 2 ): (a + b + g) 2 (a + b + g) 2 Thay vào đẳng thức trên ta được b ga 2 (a + b + g)b ga 2 + a(b ga 2 + gab 2 + ab c 2 ) a + b + g P A=(PBC) = = a 2 b ga 2 + gab 2 + ab c 2 : Mặt khác cũng từ bổ đề ta có: a(a + b + g) P P=(ABC) = b ga 2 + gab 2 + ab c 2 . Từ hai công thức trên, PA=(PBC) = (a + (a + b + g)2 b+ g)P P=(ABC) . Làm tương tự với các đỉnh B;C ta có điều phải chứng minh. Các trường hợp đặc biệt: Bài toán 3.4. Cho DABC và điểm P bất kỳ không thuộc cạnh, không thuộc đường tròn ngoại tiếp tam giác. Nếu PA=(PBC) = PB=(PCA) = PC=(PAB) thì P là trọng tâm DABC.
  • 78. Tải tài liệu tại sividoc.com Viết đề tài giá sinh viên – ZALO:0973.287.149-TEAMLUANVAN.COM 50 Bài toán 3.5. Cho DABC và điểm P bất kỳ không thuộc cạnh, không thuộc đường tròn ngoại tiếp tam giác. Nếu a:PA=(PBC) = b :PB=(PCA) = g:PC=(PAB) thì P [aA;b B;gC] hay P = (a : b : g). Bài toán 3.6. Cho DABC và điểm P bất kỳ không thuộc cạnh, không thuộc đường tròn ngoại tiếp tam giác. Nếu a:PA=(PBC) = b:PB=(PCA) = c:PC=(PAB) thì P [aA : bB : cC] hay P là tâm đường tròn nội tiếp. Các bài toán khác Bài toán 3.7. (Công thức Euler) Cho DABC nội tiếp trong đường tròn tâm O, bán kính R. M là một điểm bất kỳ trong mặt phẳng tam giác. Gọi A1; B1; C1 lần lượt là hình chiếu vuông góc của M lên BC; CA;AB. Chứng minh rằng A 1 B 1 C 1 = P M=(O) : ABC 4R 2 Bài toán 3.8. Cho DABC trọng tâm G. Gọi A 0 ;B 0 ;C 0 là giao của GA;GB;GC với đường tròn ngoại tiếp (O). Hãy tính 1 + 1 + 1 : GA 02 GB 02 GC 02 Bài toán 3.9. Cho DABC và điểm I [aA;b B;gC]. Gọi A 0 ;B 0 ;C 0 là giao của IA;IB;IC với đường tròn ngoại tiếp (O). Hãy tính a + b + g : IA 02 IB 02 IC 02 Bài toán 3.10. Cho DABC nội tiếp trong đường tròn (O), hai phân giác trong là AE;BF, đường thẳng EF cắt (O) tại I: Chứng minh rằng 1 = 1 + 1 : IA IB IC Bài toán 3.11. Cho DABC nội tiếp trong đường tròn (O). Giả sử I [aA : b B : gC]. Các đường thẳng AI;BI cắt BC;CA tại E;F. Tia EF cắt (O) tại I. Chứng minh rằng a = b + g : a:IA b :IB g:IC 3.2 Một số bài toán thi học sinh giỏi và thi Olympic
  • 79. Tải tài liệu tại sividoc.com Viết đề tài giá sinh viên – ZALO:0973.287.149-TEAMLUANVAN.COM Ta sẽ bổ sung thêm một số kỹ thuật cơ bản trước khi áp dụng các phương pháp nói trên vào giải các bài toán ở mức độ khó: Các bài toán thi học sinh giỏi và thi Olympic.
  • 80. Tải tài liệu tại sividoc.com Viết đề tài giá sinh viên – ZALO:0973.287.149-TEAMLUANVAN.COM 51 3.2.1 Véc tơ chuyển chỗ Véc tơ chuyển chỗ của hai điểm (tọa độ chuẩn hóa) P = (p1 : p2 : p3); Q = (q 1: q 2: q 3 ) PQ và bằng (p 1 q 1 ; p 2 q 2 ; p 3 q 3 ): Chú ý rằng được ký hiệu là! tổng các tọa độ của véc tơ chuyển chỗ p1 q1 + p2 q2 + p3 q3 = 0: Trong các phép chứng minh tiếp theo ta sẽ dẫn ra một số tính chất sau với quy ước DABC là tam giác cơ sở và O là tâm đường tròn ngoại tiếp tam giác ABC, bán kính đường tròn ngoại tiếp là R: ! ! ! ! O = 0 ;tức OO = 0 ! ! ! ! A : A = R2 ;tức OA:OA = R2 c 2 c 2 ! ! ! ! A:B =R 2 ;tức OA:OB= R 2 : 2 2 3.2.2 Đường thẳng vuông góc Bổ đề sau được chứng minh nhờ định lý Pythagorean Bổ đề 3.1. ! MN 1 1 1 ! 2 2 2 ): Khi Xét các véc tơ chuyển chỗ = (x ;y ;z ) và PQ = (x ;y ;z đó MN ? PQ , 0 = a 2 (z1y2 + y1z2) + b 2 (x1z2 + z1x2) + c 2 (y1x2 + x1y2): Lấy ~ = ~ khi đó: Chứng minh. O 0 ! ! ! ! ! ! MN ? PQ , (x1 A + y1 B + z1 C )(x2 A + y2 B + z2 C ) = 0: Khai triển vế trái ta có c 2 2 å(x1x2R 2 ) + å(x1y2 + x2y1) R 2 = 0: ,R 2 å(x1x2) + å(x1y2 + x2y1) ! = 1 å(x1y2 + x2y1):(c2 )) 2 1 ,R 2 (x1 + x2 + x3)(y1 + y2 + y3) = å(x1y2 + x2y1):(c2 )) 2 ,R 2 0:0 = 1 å(x1y2 + x2y1):(c2 )) , 0 = 1 å(x1y2 + x2y1):(c2 )) 2 2
  • 81. Tải tài liệu tại sividoc.com Viết đề tài giá sinh viên – ZALO:0973.287.149-TEAMLUANVAN.COM Trong các biểu thức trên là ký hiệu hoán vị vòng tròn.
  • 82. Tải tài liệu tại sividoc.com Viết đề tài giá sinh viên – ZALO:0973.287.149-TEAMLUANVAN.COM 52 Bổ đề trên có hai hệ quả sau: Hệ quả 3.9. ! 1 1 1 ) là véc tơ chuyển chỗ. Khi đó PQ ? BC khi và chỉ PQ = (x ;y ;z khi 0 = a 2 (z1 y1) + x1(c 2 b 2 ): Hệ quả 3.10. Trung trực của cạnh BC trong tam giác ABC có phương trình 0 = a 2 (z y) + x(c 2 b 2 ): 3.2.3 Phương trình đường tròn Giả sử DABC là tam giác cơ sở, độ dài các cạnh là a; b; c. Khi đó phương trình tổng quát của đường tròn là a 2 yz b 2 zx c 2 xy + (ux + vy + wz)(x + y + z) = 0: Chứng minh. Giả sử đường tròn có tâm ( j : k : l) và bán kính r. Điều đó có nghĩa là a 2 (y k)(z l) b 2 (z l)(x j) c 2 (x j)(y k) = r 2 : Khai triển vế trái a 2 yz b 2 zx c 2 xy + D1x + D2y + D3z = D với các hằng số Di; D nào đó. Vì x + y + z = 1 nên ta có thể viết lại đẳng thức đó là a2yz a2yz b2zx b2zx c2xy + ux + vy + wz = 0 , c2xy + (ux + vy + wz)(x + y + z) = 0: Trường hợp đặc biệt: Đường tròn ngoại tiếp tam giác ABC có phương trình a 2 yz + b 2 zx + c 2 xy = 0: Tất cả các bài toán sau đây đều đã có lời giải theo phương pháp truyền thống. Khi áp dụng tọa độ tâm tỷ cự ta nhận được cách giải độc đáo, thể hiện ưu thế của phương pháp tâm tỷ cự. Chúng ta bắt đầu với các bài toán USAMO, được giải bằng cách áp dụng tâm tỷ cự với các kỹ thuật đã nói trong chương 2. Phần này có tham khảo cách giải của Evan Chen trong[6].
  • 83. Tải tài liệu tại sividoc.com Viết đề tài giá sinh viên – ZALO:0973.287.149-TEAMLUANVAN.COM 53 Ví dụ 3.4. (MOP 2006) Tam giác ABC nội tiếp trong đường tròn w. Điểm P nằm trên đường thẳng BC sao cho PA tiếp xúc với w: Phân giác góc APBd cắt các cạnh AB; AC tương ứng tại D; E, các cạnh BE; CD cắt nhau tại Q. Cho đường thẳng PQ đi qua tâm của w; hãy tính góc BACd: Lời giải. Hình 3.2: MOP 2006 Chứng minh. Từ sự đồng dạng của các tam giác PBA;PAC ta có PB = PA = c nên BD = PA PC b DA c và AE = c . Điều đó cho D(c : b : 0);E(b : 0 : c);DQ(bc : b2 : c2 ): Điểm b EC b P 2 BC có dạng P(0 : x : y) với x;y nào đó và vì P, D và E thẳng hàng nên ta có 0 x y = 0, tức làx = b 2 . Do đó, P = (0 : b 2 : c 2 ). c b 0 2 y c b 0 c Cuối cùng, vì P;Q và O = (a cos a : : ) thẳng hàng ta tìm được 0 b 2 bc b 2 a cos a b cos b 2 = bc; 0 b c = 0: c2 bc b c c c cos g a cos a cos b cos g Điều đó kéo theo 2abc cos a = bc2 cos b + b2 cos g = bc(c cos b + b cos g) = abc; tức là cos a = 1 ;a = 600 : 2
  • 84. Tải tài liệu tại sividoc.com Viết đề tài giá sinh viên – ZALO:0973.287.149-TEAMLUANVAN.COM 54 Ví dụ 3.5. (USAMO 2001 #2) Giả sử ABC là một tam giác có w là tâm đường tròn nội tiếp. Ký hiệu D1;E1 là các tiếp điểm của (w) với các cạnh BC;AC (tương ứng). D2;E2 là các điểm trên cạnh BC;AC (tương ứng) sao cho CD2 = BD1;CE2 = AE1. Ngoài ra P = AD2 BE2. Đường tròn (w) giao với AD2 tại hai điểm, điểm gần với A được ký hiệu là Q. Chứng minh rằng AQ = D2P. Hình 3.3: USAMO 2001 #2 Chứng minh. Ta có thể xác định tọa độ tỷ cự của các điểm w = 1(a : b : c); D1 =1 (0 : 1 1 2p a p c : p b); D2 = (0 : p b : p c); E2 = (p a : 0 : p c); P Na = a b 1(p a : p p : p c) (Na là điểm Nagel). Để tìm tọa độ điểm Q ta chú ý rằng a phép vị tự tâm A biến đường tròn bàng tiếp góc A thành đường tròn nội tiếp sẽ biến D2 thành Q. Nghĩa là bán kính wQ song song với waD2 k wD1 hay Q là điểm xuyên tâm đối của D1: Vì vậy Q = 2w D1 = ( a : b p c :c p b ). a p a p p Đó là tất cả những gì cần để chứng minh AQ = PD2. AQ=PD , tức là: Ta sẽ chứng minh ! ! 2 p 1 : pa : p a = p : a p : a p : a b p c c p b p a p b p b p c p c Có thể kiểm tra đẳng thức đó trực tiếp: Các tọa độ thứ nhất bằng nhau, các tọa độ thứ hai cũng vậy do 2p = a + b + c và tương tự đối với các tọa độ thứ ba.
  • 85. Tải tài liệu tại sividoc.com Viết đề tài giá sinh viên – ZALO:0973.287.149-TEAMLUANVAN.COM 55 Phương pháp khác là đi chứng minh AQB = PD2B. Điều đó chính là a b 1 p p 0 tức là: p b c = a p đã được kiểm tra. p c c p b p p p p b p c = a p ; p b p c 0 0 a p a 0 a a 0 1 0 1 0 p a p c 2 p a () a + b + c = 2 p . Đẳng thức ! ! : AQ=PD Ví dụ 3.6. (USAMO, 2008, #2) Giả sử ABC là một tam giác nhọn, không cân. M, N, P là trung điểm các cạnh BC;CA;AB tương ứng. Trung trực của AB và AC cắt tia AM tương ứng ở D và E và giả sử các đường thẳng BD và CE giao nhau tại F ở trong tam giác ABC. Chứng minh rằng A;N;F;P nằm trên một đường tròn. Hình 3.4: USAMO 2008 Chứng minh. 1 1 Giả sử A = (1 : 0 : 0);B = (0 : 1 : 0);C = (0 : 0 : 1). Hiển nhiên, F = (: : 0),...Kiểm tra được phương trình của AM là y = z. Bây giờ ta sẽ tính tọa độ của D. Vì D 2 AM nên có thể viết D = (1 2t;t;t). Áp dụng điều kiện vuông góc DP ? AB: